◎正当な理由による書き込みの削除について:      生島英之とみられる方へ:

初心者質問スレ 153 YouTube動画>13本 ->画像>53枚


動画、画像抽出 || この掲示板へ 類似スレ 掲示板一覧 人気スレ 動画人気順

このスレへの固定リンク: http://5chb.net/r/denki/1651030631/
ヒント:5chスレのurlに http://xxxx.5chb.net/xxxx のようにbを入れるだけでここでスレ保存、閲覧できます。

0001774ワット発電中さん2022/04/27(水) 12:37:11.11ID:fGyHgmFC
      /゙ミヽ、,,___,,/゙ヽ
      i ノ   川 `ヽ'
      / ` ・  . ・ i、   初心者発 質問スレッドです。
     彡,   ミ(_,人_)彡ミ
 ∩,  / ヽ、,      ノ    スレのルールをよく読んで
 丶ニ|    '"''''''''"´ ノ     みんな仲良く教え合いましょう
    ∪⌒∪" ̄ ̄∪

  初心者質問スレのルール
  ・回答者のルール 初心者を笑うな。回答者にも同じ時期があったはず。彼らの気持になれ。
           真意をうまく聞き出すのも先輩の能力だ。
  ・質問者のルール 他人にわかりやすい説明を心がけて。ここには「超人エスパー」は居ません。
           回答をもらったら「ありがとう」と謝礼せよ。

  × 華麗に放置される質問
     ・自分で努力していない ・「実は...」(後出し説明) ・「回路図をお願いします」(丸投げ)
     ・「宿題の解答が欲しい」(自分でやれ) ・マルチポスト(複数スレに同質問)
     ・専門用語や変な省略語の使用 (馬鹿丸出し) ・違法なニオイぷんぷんの質問 (犯罪はダメ)
   こんな質問には、回答しません。全力放置されます。

  ◎ 必ず解答が得られる質問
     1) 何がしたいのか、はっきり書いてある質問
     2) まず自分でググって調べてあって、 グーグル先生→ http://www.google.co.jp/
     3) 回路図や写真がUPされていて、
        アップローダ→ http://imgur.com/  http://www.wazamono.jp/img/pc/
     4) そして、精一杯の説明がされていて、
     5) あなた自身の予想が書いてある、
   そんな質問なら必ずレスがあります。
   質問者は、質問逃げするな。ちゃんと礼を言って終わりにしろ。
   回答者は、仲良くやれ。煽るな、ケンカするな。偉そうにするな。

  過去スレ
その148 http://2chb.net/r/denki/1626833004/
初心者質問スレ 151
http://2chb.net/r/denki/1642773462/

  それでは、質問どぞ〜

前スレ
初心者質問スレ 152
http://2chb.net/r/denki/1646659234/
0002774ワット発電中さん2022/04/27(水) 13:13:13.97ID:y31jVrZM
いっちょつ!!
0003774ワット発電中さん2022/04/28(木) 15:30:02.70ID:mX7agLrH
しつもんです・・・
1)一次コイルの電圧
2)一次電流、磁束密度B
3)磁場H
4)励磁電流I
励磁電流は正弦波とはだいぶ異なる形に歪んでいますが、
家庭用コンセントの波形はこんなに歪んでいない印象です。
柱上変圧器は鉄心を用いていないのでしょうか??
それとも、同様に歪んではいるが、ヒステリシスカーブが狭くなるよう
工夫してあるので、歪みが軽微で済んでいる、ということでしょうか?
c.f. 電気機器学 P.11 電気学会 オーム社
0004774ワット発電中さん2022/04/28(木) 16:27:34.45ID:G/iWTtOl
コンセントの波形って電圧波形見たんでしょ
000532022/04/28(木) 16:33:36.42ID:mX7agLrH
はい そうです
0006774ワット発電中さん2022/04/28(木) 16:36:16.18ID:G/iWTtOl
>>励磁電流は正弦波とはだいぶ異なる形に歪んでいますが、
>>家庭用コンセントの波形はこんなに歪んでいない印象です。

電流波形をみたいのに、電圧波形を見ちゃってるのが間違いかな
0007774ワット発電中さん2022/04/28(木) 16:39:56.35ID:pPrbqqD8
電気機器学 P.11 電気学会 オーム社 の著者に謝れ
自分に都合の良いところだけ読んで、ずさんな引用もどきをして誤解を撒き散らすな

顔文字、つけてもらえませんか?
0008774ワット発電中さん2022/04/28(木) 16:46:11.46ID:mX7agLrH
ええー。
電流がもしも正弦波なら負荷が抗って現れる電圧も正弦波でしょうけど
電流の時点で正弦波から逸脱してるのに電圧が正弦波になるという理解なの??
まいったな、意味わからん(^p^q^;)
0009774ワット発電中さん2022/04/28(木) 16:49:15.76ID:G/iWTtOl
>>8
交流回路では、電圧と電流の波形が一致しないのが普通
一致するのは負荷が純抵抗な場合くらい
0010774ワット発電中さん2022/04/28(木) 16:58:20.37ID:mX7agLrH
えぇえー(・д・`
容量性負荷、誘導性負荷とが相殺し合って残った成分側に、
位相が進むか遅れるかはするだろうし、抵抗成分で振幅は変わるでしょうけれども、
「波形」迄変わっちゃうんですか?まいったな、ますます意味わからん(^q^p^;
0011774ワット発電中さん2022/04/28(木) 16:59:29.59ID:G/iWTtOl
勉強すればわかるようになるよ
0012コテハン2022/04/28(木) 17:19:23.37ID:mX7agLrH
ぐぎぎ  納得はできませんでしたぐゎ、とりま レスをありがとうございました
0013774ワット発電中さん2022/04/28(木) 17:34:21.62ID:G/iWTtOl
勉強中なんでしょ、ある日突然、納得いかなかった部分が、ぐるっと回ってつながって
なんだそういうことかっていう日が来るから、がんがれ
0014774ワット発電中さん2022/04/28(木) 18:43:52.87ID:Qbj9t4qq
交流はねー ひとによっちゃぁ死ぬまで理解できないな
0015774ワット発電中さん2022/04/28(木) 18:56:01.45ID:AIPF0ECK
ヒステリシス損はHとBに囲まれた面積だから
ほんとうに図に示されたとおりなら
キロワットの機器つなげば
トランスの鉄心が一瞬で爆死するヨカン
0016774ワット発電中さん2022/04/28(木) 19:06:46.91ID:Elv7piSC
ベクトル図は絵のふいんきでイメージOK
三角関数で挫折
ボード/ナイキスト、取り残されはるか先へ
0017コテハン2022/04/28(木) 19:14:18.27ID:fxlhz6qx
あ、あれか。正弦波からの逸脱は、異なる周波数成分の重畳を意味していて、
より高い周波数成分に対してはより強く働いて、
正弦波に近い電圧波形に整えられるってことなのかなぁ・・・(´〜`;

交流理論の議論では数式表現する都合上なのか、
議論を簡略化する為なのか、正弦波を「前提」とした段階で
捨象されてたんやろうな。。。

>>15
つっても、キロワット機器自体が爆発してないわけだし
鉄損が大きいタイプのトランスの発熱は
放熱フィンで雰囲気中に捨ててるんじゃね? しらんけど(^p^)
0018774ワット発電中さん2022/04/28(木) 19:49:02.34ID:pPrbqqD8
>>3
つけてくれてありがとう

励磁電流の具体的な作図方法のPDFがあったので御紹介
http://hirachi.cocolog-nifty.com/kh/files/20071011-1.pdf

ところで
>>励磁電流は正弦波とはだいぶ異なる形に歪んでいますが、

歪み波形が、>>3の図とPDFでちがって見えると思いますが
それは、>>3の図が鉄心を磁気飽和させている場合で (教科書がそうなっているかは知りません)
PDFは解説にあるように磁気飽和しない範囲にしているからです

>家庭用コンセントの波形はこんなに歪んでいない印象です。

すでに言われているように、電圧波形と電流波形の違いですが、
そもそもコンセントで励磁電流の波形を見られると思ったのが
たんなる勘違いか、「励磁電流」を知らない、電圧と電流でこんがらがっているか、そのあたりです

>>柱上変圧器は〜
もちろん鉄芯を使っています
柱上変圧器にかぎらず、ヒステリシス損、過電流損をできるだけ小さく
できるだけ安価に、できるだけ軽量になるように、
ついでに銅損もできるだけ小さくなるように、いろいろ工夫されています

蛇足
ヒステリシス損、渦電流損には周波数特性があります
波形が歪むといささか面倒
0019774ワット発電中さん2022/04/28(木) 20:41:13.84ID:LG0t80oT
励磁電流はコイルトランスを駆動する時の、お駄賃電流ともいう。
本当は、トランスの励磁インダクタンス等価回路の導出を理解してないといけない。
この導出方法は、ほとんどの教科書に書かれていない。秘伝の方法か?
実は、スイッチング電源の教科書にトランスの等価回路の導出方法が書いてある。

オーム社のBHカーブとトランス電圧電流図は、大げさに書いたもので、実働電圧
範囲をオーバーして書いたもの。実際の使用時はBHカーブ線形範囲で使用する。
励磁電流が急上昇し磁気飽和レベルまで使用する用途は大昔のスイッチング電源
というかロイヤー回路とかジェンセン回路で使用された、更に真空管発明前の
火花式送信回路でも使われたが、こんな昔の100年前の使用例を出しても。
0020コテハン2022/04/28(木) 20:48:33.22ID:fxlhz6qx
俺の画力が低いだけでリンク先のカーブな形でしたサーセン

励磁電流、励磁なんだから二次側じゃなくて
一次側の電流の「成分」なんですね。 電力技術便覧にあたったら、
一次電流I1 = 励磁電流I0 + 補償電流I’1 なのだそうで;保証電流ってなんじゃらほい
勉強し直しますさーせんした;
0021774ワット発電中さん2022/04/28(木) 21:59:14.53ID:eO5SuPNV
ヒステリシスのために歪んだ波形になるのか?
ヒステリシスが無かったらB-Hが曲がってても歪まないのか?
0022774ワット発電中さん2022/04/28(木) 23:24:20.87ID:LG0t80oT
ヒステリシスのために歪んだ波形になるのか?・・・・yes
Bは、Φ=nφ=nBS=L*i だから Bが歪めばiも歪む

ヒステリシスが無かったらB-Hが曲がってても歪まないのか?・・・???
BHが曲がることをヒステリシスと言う。だからBHが曲がらなければヒステリシス
はない。だれか、空芯以外で、曲がらない磁性体を発明して欲しいものだ。
・・・村田製作所でも、TDKでもダメか?
0023774ワット発電中さん2022/04/28(木) 23:43:17.12ID:5jQsmTxu
>>3
ちょっとWebを見て調べた範囲だけど
参考にした資料ではヒステリシス損失を説明する為に大げさなカーブで書いてるのでは?
実際には素材の関係でヒステリシス損失が少なければ歪みも少ない状態になるのでは?
あと普及率は知らなが鉄心には合金が使われていてヒステリシス損失を減らす工夫がされてるみたい
なんかググったらアモルファス磁性材なるものが出てきた

あとオシロスレで美少女のタイツを被るアイデア参考になりました~この場を借りてお礼申し上げます
ですがタイツを提供してくれる美少女が見つかりません\(^o^)/

あと我が家の100Vは正弦波に比べれば歪んでますね
初心者質問スレ 153 YouTube動画>13本 ->画像>53枚
0024774ワット発電中さん2022/04/29(金) 00:40:26.23ID:OTUMSS+O
電気は謎が多いな… 惹かれるぜ…
0025774ワット発電中さん2022/04/29(金) 01:24:04.02ID:RMC2uL/6
>>4
オシロスコープって電圧を見る装置だと思ってました。

電圧波形はどう見たら良いのでしょうか?
0026774ワット発電中さん2022/04/29(金) 03:13:00.77ID:Ka/XrCl6
>>8
顔文字、やめてください
0027774ワット発電中さん2022/04/29(金) 03:14:40.64ID:Ka/XrCl6
>>10
落ち着け。
顔文字、やめて下さい
0028774ワット発電中さん2022/04/29(金) 03:16:02.33ID:Ka/XrCl6
>>17
顔文字やめてください
0029774ワット発電中さん2022/04/29(金) 04:59:55.11ID:BRfYktMX
ヒステリシスって行きと帰りで通る道が違うことをいうのだと思ってたが
曲がるのがヒステリシスとは強電は良く分からんな
0030774ワット発電中さん2022/04/29(金) 05:06:41.30ID:5WTHNZqN
>>26
お前が顔文字をNGに入れりゃ済む話だろ
いちいち相手にするな
0031774ワット発電中さん2022/04/29(金) 06:16:02.33ID:3rNMI298
なぜ顔文字がダメなの(ノ_・、)
0032774ワット発電中さん2022/04/29(金) 06:17:22.04ID:3rNMI298
>>29
ヒステリーの顔文字 (*`Д´*)
0033774ワット発電中さん2022/04/29(金) 07:37:09.23ID:qw6Pou+i
「顔文字やめてください」をNGワードにすれば。
0034774ワット発電中さん2022/04/29(金) 07:41:20.26ID:qipUQBQf
>>25
電流プローブっていう伝説の、、以下略
0035774ワット発電中さん2022/04/29(金) 07:54:00.78ID:3rNMI298
>>26-28

(/´△`\)

やめてください、の顔文字
0036774ワット発電中さん2022/04/29(金) 07:59:00.82ID:v3Lye1Zd
>>29
ヒステリシスが「履歴的な性質」という解釈は誤解で、本来は、「変化が遅れる」という意味らしい
Xの変化にYの変化が遅れてしまうから、(X,Y)をプロットするとカーブを描く

en.wikipedia "Hysteresis"よりgoogle翻訳
「ヒステリシス」という用語は、「欠乏」または「遅れ」を意味する古代ギリシャ語のὑστέρησιςに由来します。
1881年にジェームズアルフレッドユーイング卿によって造られました。[1]

『磁性の基礎からスピントロニクスまで(2)』佐藤勝昭 東京農工大学名誉教授 科学技術振興機構
https://www.kg-nanotech.jp/data/doc/1627467268_doc_6_0.pdf
”hysteresis”の語源は、ギリシャ語で「遅れ」を表すことばで、外界の変化に対して応答が遅れることを
意味しています。磁気ヒステリシスを磁気履歴ということがありますが、これは、hysteresis
とhistoryを混同した誤訳に基づくものだといわれています。(p.27の囲み)
0037774ワット発電中さん2022/04/29(金) 09:28:28.41ID:g5P1e0P/
>>33
3連投して3回とも微妙に書き方を変えるんだから
もう荒らしだよ
0038774ワット発電中さん2022/04/29(金) 12:04:38.18ID:4DiNxTA+
あのー、BHカーブのヒステリシス現象は単に、磁芯に損失があるからなんだけど。
そもそもBHカーブは、コイルに交流電圧を掛け、電圧積分波形(磁束量)と電流を
測ったリサージュ波形だよ。電流流して電圧積分波形取ってもいいけど。要するに
インダクタと抵抗を並列なり直列にした回路のリサージュ波形。BHカーブは磁性体
だけど、誘電体にも損失、飽和現象があり、これはDEカーブという。

BHカーブは二つの意味があり、一つは損失によるヒステリシス現象、二つ目は
磁気飽和による飽和現象。だから本当はBHヒステリシス磁気飽和カーブと言うべき
なんだけど、略して言ってしまう。この辺を混同してはいけない。
0039774ワット発電中さん2022/04/29(金) 13:12:48.46ID:4DiNxTA+
あ、でも残留磁化、保持力ともいうか?
0040774ワット発電中さん2022/04/29(金) 22:27:32.71ID:rzaEFdFj
>>23
>提供美少女不在
そっかー。ざんねんだなー。

どきどき! 悩殺下着生放送回!(付図)とか
美受肉だらけの水着大会ポロリ(※)もあるよ! (※きゃんたま袋) とか
おもしろそうだと思ったんだけどなぁー・・・(・д・`
0041774ワット発電中さん2022/04/30(土) 10:43:25.02ID:1a19FFmE
外部から5Vの入力があるときに、1.8VのLDO(以下LDOA)と、1.5VのLDO(以下LDOB)を使って、
3.3Vの電源って作ることはできますか?
素人考えだと、LDOA.VCCに5V、LCOA.GNDに外部のGNDを接続して1.8Vを生成、
LDOB.GNDにLDOA.OUTの出力1.8Vを接続し、LDOB.VCCに外部の5Vを接続すれば、
LDOBの出力は外部GNDに対して3.3Vになると思うのですが、こんな使い方ってできるんですか?
LDOB単体で見た時の入力電圧が0→5V→3.2V(GND端子が1.8Vになって、VCCが5Vになるので)
って変な遷移をしてしまうことになりますが、大丈夫なんでしょうか?

すいませんが、よろしくお願いします。
0042774ワット発電中さん2022/04/30(土) 11:21:06.30ID:MlGRFKdF
>>41
やったことないですが。

LDOのGND端子からは電流が流れだしています。
でも、多くLDOが電流の吸い込みはできないはず。LDOのGNDをほかのLDOの出力に単純に接続するとそこが問題になりそう。
下にLDOA.OUTに抵抗などで十分な出力電流をながしておけばいいかも。

リニアレギュレータのGNDにツェナーを入れて電圧の嵩ましをするのはあるし。
0043774ワット発電中さん2022/04/30(土) 11:26:28.36ID:A53PKOWN
>>41
絵をかいてみればわかるよ、電位差を生じさせるにはつながってないとならない
0044774ワット発電中さん2022/04/30(土) 12:21:27.99ID:MlGRFKdF
>>42に補足

https://imgur.com/a/W1fKmWQ

元は(A)という話ですね。抵抗を付けて下のレギュレータの出力の電流を流してるけど。
上にも書いた通り、やったことはないですが、できそうな気がします。
ただ、LDOはいろいろな種類があって、発振しやすいタイプもあります。ものは選ぶかもしれません。

(B)みたいに、LEDとかツェナーとかを入れて電圧の嵩ましをするのは、78シリーズでは定番でした。

(C)はLM317の接続ですけど、自分の出力で自分のGNDを上げて電圧の嵩ましをしているとも言えます。
78シリーズでもできなくはなかったです。

LDOの1117には固定電圧タイプとアジャスタブルタイプがありますが、固定タイプは抵抗を内蔵しているものです。
https://www.onsemi.com/pdf/datasheet/ncp1117-d.pdf
0045774ワット発電中さん2022/04/30(土) 12:33:57.02ID:Tw4SAGAi
Aliで安かったLDO、XC6206(3.3V最大200mA)が100個あります。
これを5つ 並列にしたら1A取れますか?ESP32とか動かせますか?
0046774ワット発電中さん2022/04/30(土) 12:54:37.23ID:MlGRFKdF
>>45
LEDの並列接続と同じで定電圧の性質を持つ素子を単純に並列にすると、ばらつきの影響を避けられません。
特別にそれができることを標ぼうしているものでない限り、並列接続で電流を増やすことはできないと考えて良いと思います。
個々の出力に抵抗を入れて束ねるという方法もありますけど、おすすめできないです。
0047774ワット発電中さん2022/04/30(土) 13:09:36.67ID:ML9mwv6w
バランサーが必要になるんじゃない?
しらんけど

LM1117は0.8Aだから足りないな
ドロップ0.2V 1.5A 70円はどうだろう
https://akizukidenshi.com/catalog/g/gI-09261/
0048774ワット発電中さん2022/04/30(土) 13:43:54.70ID:JYTww55Y
やはりそう単純なものじゃないんですね。
調整も大変そうなので素直に大容量のものを探してみます。リンクの商品も検討します。
ありがとうございました。
0049774ワット発電中さん2022/04/30(土) 13:55:05.35ID:9UgBbvGt
>>45
データシート見てきたけど
並列が難しい事は他の方が書いてるので他の事を
XC6206は入力と出力の電圧差が少ない状態で使用することを前提にしてるみたい
なので5Vから3.3Vとかの変換は定格を超えてしまうので出来ない
あとAliで売られてるXC6206(に限らず半導体の大半)は本物ではないと思った方が良い
(本物より性能が低い可能性を十分に留意してください)
0050774ワット発電中さん2022/04/30(土) 13:56:55.66ID:rB4N0Z8Q
(C)なら一個で出来るし良いね
0051774ワット発電中さん2022/04/30(土) 14:29:07.37ID:MlGRFKdF
>XC6206は入力と出力の電圧差が少ない状態で使用することを前提にしてるみたい
>なので5Vから3.3Vとかの変換は定格を超えてしまうので出来ない

https://www.torex.co.jp/file/xc6206/XC6206-j.pdf
↑データシートはこれかな
(本物でないかもしれない、というのは別として)
このデータシートを見る限りだと入力は6VまでOKです。

ただ、入力電圧が高ければ発熱も増えます。
SOT23パッケージだと単体で(おそらく周囲温度25℃で)250mWってことですので、
250/(5-3.3)=147
147mAが安全に使える電流になります。
データシートに書かれているような基板に実装した場合は500mWですし、この倍になります。
0052492022/04/30(土) 15:05:57.11ID:9UgBbvGt
>>51
すまん、色々勘違いしてた
首吊ってくる
0053774ワット発電中さん2022/04/30(土) 20:18:58.21ID:NMZBm/M+
>>44
いろいろありがとうございます!

デジタル屋なので、電源は全くわからないので、B/C等の定番を
知識として知れただけでもありがたいです。

はい、Aの図を考えています。
正確には、LDOB.OUTの3.3Vが必要なだけではなく、
LDOA.OUTで生成した1.8Vも、ICの電源として使用するので(よくある
3.3V/1.8VのVDDが必要なICへ供給を考えてます)
LDOA.OUT=LDOB.GNDから、実GNDへの流れだしパスはあるはずです。

情報ありがとうございました!
0054774ワット発電中さん2022/04/30(土) 23:51:30.84ID:bAvbT5ua
しつもんおねがいします
https://monoist.itmedia.co.jp/mn/articles/1601/28/news005_3.html
> 次にGPIOの電圧が決まれば、ベースにつながった抵抗値を決定することができます。
GPIOの電圧を5Vとすると4.5mA流れるように抵抗値を求める式はオームの法則より、
抵抗=電圧/電流=5/0.0045=1111となり、Rの抵抗値は1111Ω(オーム)となります。

5Vからベースーエミッタ間電圧を除いてから必要な電流値で割るのではないでしょうか?
0055774ワット発電中さん2022/05/01(日) 00:22:29.30ID:oTMcDdT3
>>54
>5Vからベースーエミッタ間電圧を除いてから必要な電流値で割るのではないでしょうか?
Yes

それ以外も色々と間違ってる気がする
0056774ワット発電中さん2022/05/01(日) 00:32:50.63ID:Ibdi5kBm
コンセントのAC100Vって片方の端子がGNDでもう片方が±141Vで振れてるってことですか?
それとも浮いてて両端子間が141Vってことですか?
0057774ワット発電中さん2022/05/01(日) 00:33:00.63ID:W1jMkjNi
FA-130RAのデータシートってこれかな?
https://www.mabuchi-motor.co.jp/motorize/branch/motor/pdf/fa_130ra.pdf
500mAがどこから出てきたのかわからない。
DCモーターって起動時とか、外部要因で止まったときに大きい電流が流れます。
データシートではSTALL時2.1Aって書いてあるし。
0058774ワット発電中さん2022/05/01(日) 00:36:16.14ID:oTMcDdT3
>>54
URLの内容見てきたけどおかしな点が複数
・モーターの始動時の電流値がトランジスタの最大定格を超えてる
・モーターはインダクタンスが高い(誘導性負荷)なのにフリーホイールダイオードの設置なし
 (これはON>OFF時にトランジスタが降伏状態になり故障する可能性が高い)
・増幅率は温度や個体差での差が大きいのでベース電流はもっと余裕が必要
 (そもそもグラフの読み方間違ってる・・・)

個人のブログより酷い \(^o^)/
0059774ワット発電中さん2022/05/01(日) 00:49:30.15ID:KnU+Fa2Q
>>56
単相100VのVp-pは282.8V
0060774ワット発電中さん2022/05/01(日) 00:58:09.86ID:Ibdi5kBm
>>59
浮いてるかはともかく両極間は282Vなんですねありがとうございます
0061774ワット発電中さん2022/05/01(日) 01:04:24.84ID:oTMcDdT3
>>56
コンセントの100Vはニュートラル・ライブが有って
ニュートラルは外で地面にアースされてる
計測するとニュートラルとGND(アース)は近い電圧になる
0062774ワット発電中さん2022/05/01(日) 01:13:50.54ID:Ibdi5kBm
>>61
なるほど浮いてるかどうかはどちらでもいいんですね
0063774ワット発電中さん2022/05/01(日) 03:37:31.20ID:Qklv1hNz
>>58
顔文字やめてください
0064774ワット発電中さん2022/05/01(日) 06:35:32.77ID:Ss2jN57l
>>62
浮くという表現は何に対してということが重要になりますよ
この場合大地に対してですが( ゚∀゚)
0065774ワット発電中さん2022/05/01(日) 07:02:08.79ID:r7fSLPen
>>63


   ____∧∧  / ̄ ̄ ̄ ̄ ̄ ̄ ̄ ̄
 ~' ____(,,゚Д゚)< 逝ってよし!
   UU    U U   \________
0066774ワット発電中さん2022/05/01(日) 07:07:52.65ID:1eVrG6Pe
>>65
   ∧_∧  / ̄ ̄ ̄ ̄ ̄
  ( ´∀`)< オマエモナー
  (    )  \_____
  | | |
  (__)_)
0067774ワット発電中さん2022/05/01(日) 07:13:46.06ID:uSnctXcm
「顔文字やめてください」って言うのやめてください
0068774ワット発電中さん2022/05/01(日) 10:41:52.32ID:W1jMkjNi
>>60
>浮いてるかはともかく両極間は282Vなんですねありがとうございます
違う。両極間のピークは±141V。だからVp-pなら282V。
で、一方がニュートラルで接地側です。
https://faq.fa.omron.co.jp/tech/s/article/faq04154
0069774ワット発電中さん2022/05/01(日) 11:03:36.09ID:rapkMDWj
>>54
そもそもスイッチング用途でベース電流を計算するのに必要なのは増幅率hfe。
0070774ワット発電中さん2022/05/01(日) 11:05:03.22ID:P9ZI4NMH
>>56
>コンセントのAC100Vって片方の端子がGNDでもう片方が±141Vで振れてるってことですか?

これで合ってる
0071774ワット発電中さん2022/05/01(日) 11:10:35.26ID:W1jMkjNi
>>69
500mAのでどころがいい加減なのでもういいか、って思ったけど、これも変ですね。
>今回モーターを駆動する電圧は3Vとしますので、横軸の3Vのところから上に目をやり
>縦軸のコレクタ電流の500mAの横軸と交わるところを探します。
>その時のベース電流(IB)を読みとります。グラフからだと4mAから5mAの間の様に読み取れます。ここでは仮に4.5mAとします。
500mA流しているときには、コレクタエミッタ間電圧は、Vcesatになっているのに、0mA流しているときの3Vで考えています。なにこれ。
0072774ワット発電中さん2022/05/01(日) 11:38:14.42ID:W1jMkjNi
同じライターの記事で
https://monoist.itmedia.co.jp/mn/articles/1608/25/news004_2.html
>今度はステッピングモーターの駆動に必要な電力を計算してみましょう。
>1A×12Vで12Wとなります。最大定格は12.5Wですので、これはギリギリOKなのですが
ICの許容損失と駆動電力を混同しています。

今岡通博さんは、ソフトや論理寄りの人っぽい。
0073774ワット発電中さん2022/05/01(日) 11:45:39.13ID:8B1qTHHD
>>57
昨日検索してたらFA-130RAのバリエーションで500mAの物もあるみたいです
(まあ、500mAだとしても問題はそこじゃないんですが・・・)
https://jp.misumi-ec.com/vona2/detail/223012118886/
https://www.smartschool.jp/products/detail.php?product_id=38110
0074774ワット発電中さん2022/05/01(日) 11:58:35.12ID:W1jMkjNi
>>73
ありがとうございます。巻き線か何か違いでいくつかのバージョンが出回っているのか。
0075774ワット発電中さん2022/05/01(日) 11:59:56.74ID:8B1qTHHD
>>72
そのライターさんの記事で逆起電力の説明があるんだけど
https://monoist.itmedia.co.jp/mn/articles/1603/04/news042.html
間違いだらけ〜\(^o^)/

これ、大手出版社が運営するサイトみたいなんですけど・・・・
0076774ワット発電中さん2022/05/01(日) 12:17:21.49ID:W1jMkjNi
>電流を流していたコイルの電流を切ると、逆方向に電流が流れます。これを逆起電力と呼び…
とか図2とか。
ちょっとまずいな。なまじ真面目な講座っぽい外装だけに。
0077774ワット発電中さん2022/05/01(日) 12:34:05.98ID:Ss2jN57l
>>76
どこを電流が流れるの
0078774ワット発電中さん2022/05/01(日) 12:36:03.74ID:W1jMkjNi
>>77
どういう意図で俺に尋ねてますか?
0079774ワット発電中さん2022/05/01(日) 12:36:33.11ID:Ss2jN57l
>>78
失礼しました
0080774ワット発電中さん2022/05/01(日) 12:39:54.38ID:W1jMkjNi
>電流を流していたコイルの電流を切ると、逆方向に電流が流れます。
↑これが、俺の意見だと思ったのかな? 引用符は共通認識じゃないし、仕方ないですね。
0081774ワット発電中さん2022/05/01(日) 12:40:38.80ID:+pg8gjrC
ジロー チェインジー ゴーゴゴーゴー
0082774ワット発電中さん2022/05/01(日) 12:53:38.70ID:199LKnJq
ユーチューブだって適当な事言っている人は少なくないしそんなもんだろ
0083774ワット発電中さん2022/05/01(日) 13:15:00.06ID:W1jMkjNi
>>82
そういう意見もあって良いと思います。

俺としては、上にも書きましたけど、なまじ真面目な講座っぽい外装なので飲み込んでしまう入門者が多そうなのは気になります。
そのへんのにいちゃんねえちゃんが
「はーいお元気ですか。今日はモーターをまわしてみましょう。なんかよくわからないけどこんな接続でまわりました。てへ。いかがでしたか。では」
みたいな感じなら良いのですけど。
0084774ワット発電中さん2022/05/01(日) 13:24:46.78ID:uSnctXcm
たしかにひで~なw

コイルに直流を流すと突入電流が発生するって言い張ってた奴を思い出したw
0085774ワット発電中さん2022/05/01(日) 13:50:22.20ID:199LKnJq
ガチっぽい解説動画で適当な事言っているケースも珍しくないと思いますが
0086774ワット発電中さん2022/05/01(日) 13:54:27.88ID:LIIc4wkb
>>85
コメントで突っ込まれてないの?それ
まあ、わかってる人はそもそも見てないのかもしれないけど
0087774ワット発電中さん2022/05/01(日) 14:09:18.29ID:7PasIm4r
ヘッドフォンの断線修理をやっていますが、テスターで断線部分を見つけて被膜を剥いて中は2本線でした。半田付け前にライターで炙ったのですが導線がぼろぼろになってしまいます。ネジネジした線をばらしても導線が見つかりません。服の繊維っぼいのしか見当たらなくてお手上げです。細い線はライターで炙ると記憶していましたが、安物は構造が違うのでしょうか?よろしくお願いいたします。
0088774ワット発電中さん2022/05/01(日) 14:13:26.66ID:8B1qTHHD
>>85
youtubeでその様な動画に出会ったときは黙ってBadボタンを押して立ち去ってる
あまりにも酷い時は「報告」から「有害または危険な行為」や「スパムまたは誤解を招く内容」を選んで立ち去る
良いと思った動画にはGoodボタン押してる\(^o^)/

ITmediaの記事に対してはどうしてよいやら(;´・ω・)
0089774ワット発電中さん2022/05/01(日) 14:14:13.93ID:XtZJLRkF
ちょっと写真貼ってみ
0090774ワット発電中さん2022/05/01(日) 14:14:42.79ID:LIIc4wkb
>>87
ライターであぶるには細すぎるから燃えてしまってる
絶縁剥がすには紙やすりで丁寧に、銅が出てくれば繊維と区別がつくはず
0091774ワット発電中さん2022/05/01(日) 14:19:57.18ID:APGNUIdO
スマートスピーカーにはLEDが付いていて、声を聞いている時、会話する時などに連動してチカチカと光ります。
これをフォトダイオードなどで取り込んで、USBから5Vを取り出した別の回路に繋いだ砲弾型LED3~5つが光る、という回路を作りたいと考えています。

https://stocker.jp/500W/2015/12/12/njl7502l/
こちらのページを参考にパーツを揃え組んで色々試したのですがLED1つであればそれなりに光るまでは再現できたのですが、複数繋ぐと電流値が一個分の電流を分け合っているのか光量が弱くて足りなくて困っています。元々の入力電流を上げたほうがいいのか?トランジスタの出力電流が高いものなど?に交換すれば良いのでしょうか?
0092774ワット発電中さん2022/05/01(日) 14:28:22.49ID:W1jMkjNi
>>87
白っぽい化学繊維っぽいものと緑と赤の細線が使われているような電線はわりとみかけるけど、
わざわざ被覆を剥かなくても、ちょっと高めの温度設定のはんだごてで、はんだメッキできないかな?
0093872022/05/01(日) 14:29:51.15ID:7PasIm4r
>>90
さつそくヤスリ買いにいってきます。
ヘッドフォン修理2台目なので、麹。回は成功したb「です!アドレャXありがとうごbエいました
0094774ワット発電中さん2022/05/01(日) 14:31:22.46ID:LIIc4wkb
>>92
それでもできる、繊維がじゃまだけど
0095774ワット発電中さん2022/05/01(日) 14:36:55.24ID:W1jMkjNi
>>91
>こちらのページを参考にパーツを揃え組んで色々試したのですがLED1つであればそれなりに光るまでは再現できたのですが、
それだとあなたがどんな回路にしたのかわからない。
何が原因でうまくいかないのかは、あなたが実験した回路が定数もふくめてわからないと正確なコメントも難しい。
正式な回路図でなくても、接続がわかる絵でもいい。写真に撮って>>1で紹介されているアップローダに載せて。

あと、そのページからリンクされている資料は↓だけど、これを参考にしたとしても
http://cms.db.tokushima-u.ac.jp/DAV//person/S10723/LEDを使いこなそう/公開講座06-2012SS.pdf
このどの回路なのかもわからない。何かの値が変わっているならちゃんと全部書いて。

だいじなことなのでくどくど書きました。
0096872022/05/01(日) 14:38:04.83ID:7PasIm4r
>>92
それもいけそうですね😄化学繊維が半田で溶けて導線を薄くコーティング して絶縁しないか不安なところです。あれどういう役割なんですかね?緩衝材なんすかね
0097774ワット発電中さん2022/05/01(日) 14:38:43.40ID:W1jMkjNi
>>94
たぶん、その機器を製造している業者さんは面倒なことはしてないでしょうしね。
0098774ワット発電中さん2022/05/01(日) 14:41:17.91ID:LIIc4wkb
>>97
業者はジグ作って、ガッシャンポンでしょう
0099774ワット発電中さん2022/05/01(日) 14:41:42.11ID:APGNUIdO
>>95
どうも
またちゃんと資料整えてまた来るよ
0100774ワット発電中さん2022/05/01(日) 14:52:44.37ID:LIIc4wkb
>>96
ひっぱったときにちぎれないように、テンションメンバーになってんじゃないかな
0101774ワット発電中さん2022/05/01(日) 14:59:50.99ID:199LKnJq
>>86
自分より詳しくない素人を囲ってアフィ稼ぐのが昨今のトレンドだろ
0102774ワット発電中さん2022/05/01(日) 15:13:25.25ID:W1jMkjNi
>>99
よろしく。俺にわかることなら書きます。
0103774ワット発電中さん2022/05/01(日) 16:00:36.01ID:LIIc4wkb
>>101
そうなん?ニッチすぎて儲かりそうもないんだけど
0104774ワット発電中さん2022/05/01(日) 16:08:28.74ID:8B1qTHHD
今日も暇なんだな〜\(^o^)/
https://monoist.itmedia.co.jp/mn/articles/1601/28/news005_3.html
ちょっと作ってみた(笑)
初心者質問スレ 153 YouTube動画>13本 ->画像>53枚
GPIO HIGH-->LOWの時にC<-->E間の電圧64V辺りで降伏されられてる
GPIO LOW-->HIGHの時はCE間約2.6Vで約650mAに制限されてるので瞬間的だが約1.69W位の損失が・・・
トランジスタさんかわいそう( ;∀;)
0105774ワット発電中さん2022/05/01(日) 16:34:50.27ID:voaEHKyv
2SC2120Yってw
0106774ワット発電中さん2022/05/01(日) 16:36:27.58ID:sfJTHHs8
っコンデンサ
0107774ワット発電中さん2022/05/01(日) 16:49:46.64ID:voaEHKyv
>>75
すいません、何が間違いなのか良く分からないので教えてください。
0108774ワット発電中さん2022/05/01(日) 16:51:27.73ID:LIIc4wkb
>>107
>>電流を流していたコイルの電流を切ると、逆方向に電流が流れます

このあたりw
0109774ワット発電中さん2022/05/01(日) 16:53:32.93ID:voaEHKyv
電流は流れていないので、正解は「逆起電力が発生する」でしょうか?
0110774ワット発電中さん2022/05/01(日) 16:59:31.25ID:LIIc4wkb
>>109
コイルの逆起電力というのは、電流変化を妨害する方向に出ます
ブチっと切ってしまって、電流が流れないときは、電圧となってあらわれます
0111774ワット発電中さん2022/05/01(日) 17:12:31.56ID:LIIc4wkb
マクロな目でコイルとしてではなくモーターとしてみると
モーターが発電機として働き、逆電流を流す方向に働くので
モーターの発電電圧を逆起電力と言ってるかもしれませんが
コイルとして説明してるので逆のことを言ってしまってますね
0112774ワット発電中さん2022/05/01(日) 19:23:53.33ID:e5MIC8B1
還流ダイオードとか
フライバックダイオード
っていうのが無いのがアレなんじゃね?しらんけど
0113774ワット発電中さん2022/05/01(日) 19:34:06.98ID:55Nvy6t0
逆起電力ということばがきらい(古人の乾燥です)
順方向の起電力が何だかわからないのに、いきなり逆起電力というのが一番きらい

コイルの場合は、オンのときと同じだけの電流が流れ続けようとするだけで
電圧が観察されるケースもあるけど
だれかが電力を作り出そうとしているのではないと思うのです

電流が流れている抵抗の両端電圧を測って、「これは抵抗の起電力です」と言う人が
まだ出現していないのは救いでありましょう
0114774ワット発電中さん2022/05/01(日) 20:02:08.11ID:+pg8gjrC
文系の奴だと言いそう
0115774ワット発電中さん2022/05/01(日) 20:12:31.21ID:W1jMkjNi
インダクタの電流を増やそうとしたらそれに抗する方向に
インダクタの電流を減らそうとしたらそれに抗する方向に発生するから、「逆」なんじゃないですかね。
定着した言葉なんで、今さら抗しても仕方がないとおもってます。
0116774ワット発電中さん2022/05/01(日) 20:55:10.29ID:5HvDIJTp
反日って言いますしおすし
0117774ワット発電中さん2022/05/01(日) 22:02:07.33ID:55Nvy6t0
>>115
その文脈では、起電力と逆起電力との釣り合いで説明するのが良いと思います
表皮効果、近接効果で中央の導体の電流がへる理由も逆起電力で説明できます

コイル外部からの起電力をゼロにしたとたんに、
コイル内部の逆起電力の方向が逆転するのは乾いた脳みそではついていけません
0118774ワット発電中さん2022/05/01(日) 23:21:52.52ID:ePqSiEQ2
質問です
74HC04や74HC14などのインバーターで1回路しか使わない時、残りの未使用回路の入力は
A)Hi入力にしたほうが消費電力が少ない
B)Low入力にしたほうが消費電力が少ない
C)同じ
どちらでになりますか?
0119774ワット発電中さん2022/05/01(日) 23:26:58.03ID:BzgHKl4U
同じだった気がする:-)
0120774ワット発電中さん2022/05/02(月) 00:55:14.72ID:4iiVjj0P
>>115
なぜ、逆らう方向の電流が流れるのでしょうか?
0121774ワット発電中さん2022/05/02(月) 05:40:47.99ID:9N2LTcDA
中二病だから
0122774ワット発電中さん2022/05/02(月) 08:11:24.45ID:XtgvKNQB
>>118
そんなもんやってみればわかる、DIPだったら空中配線で良いんだし
0123774ワット発電中さん2022/05/02(月) 09:17:39.17ID:C36dxlT9
>>120
「逆らう方向の電流が流れる」とは俺は書いてないよ。
0124774ワット発電中さん2022/05/02(月) 09:29:56.28ID:RoxbjpJo
どうなんだろう・・・
・強烈なノイズに晒されて解放側入力端子がバタバタ切り替わるばやいも同じと言えるのか
・小さい抵抗でプルアップ
・小さい抵抗でGND側に落とす
・大きい抵抗で(以下略
・大きい抵抗で引っ張った上に、キャパシタも追加しておく
節電の為に望ましい組み合わせはあるのかもしれんですな。しらんけど
0125774ワット発電中さん2022/05/02(月) 12:22:59.82ID:irrCRK0l
入力特性が Hi-Z だとプルアップしてもプルダウンしても消費電力は同じじゃね
抵抗挟む必要も無いんじゃね
開放がだめなだけで
0126774ワット発電中さん2022/05/02(月) 12:32:25.28ID:uAY08uvk
CMOSはインピーダンス高いんで
静電気とかで中間電位に留まられるのが一番消費電力増えるし面倒だから
どっちかに固定しろと
0127774ワット発電中さん2022/05/02(月) 12:36:11.27ID:irrCRK0l
>>113
その抵抗の意味じゃなくて
コイルが嫌々抵抗してる訳だから抵抗起電力はアリだと思います!
0128774ワット発電中さん2022/05/02(月) 13:17:33.96ID:r5eBsFb1
この回路って動くんですか?
二次側が1方向にしか流れないからひたすら片側に励起されて飽和しないんですか?
初心者質問スレ 153 YouTube動画>13本 ->画像>53枚
0129774ワット発電中さん2022/05/02(月) 13:17:57.21ID:RoxbjpJo
>>127
コイル「いやー><;やめてー」
悪電源「嫌よ嫌よも 好きの内ってねぇ・・・グヘへ!(^p^」

>>126
しーもすでばいしずのThrough Currentは
突貫電流とも訳されますな・・・
倫理だと偏見は悪で、中庸の徳が肝要とか説くのにネ!
0130774ワット発電中さん2022/05/02(月) 13:24:01.74ID:JibGGnhY
>>128
そういう場合は直流分で飽和しないだけの容量を持つトランスを使うんだよ
0131774ワット発電中さん2022/05/02(月) 13:27:31.60ID:JibGGnhY
おまいら、おちつけ
CMOSの貫通電流は入力オープンにしたときのはなしだ
>>118はオープンにするとは一言も言ってないぞ
0132774ワット発電中さん2022/05/02(月) 13:39:11.26ID:jCvB4e4s
なんでこれって同じ線で繋がってるのに電圧が5Vに減るの?

初心者質問スレ 153 YouTube動画>13本 ->画像>53枚
0133774ワット発電中さん2022/05/02(月) 13:46:09.88ID:Q2BeTKMA
ツェナーを抵抗に置き換えて味噌
0134774ワット発電中さん2022/05/02(月) 13:47:44.19ID:RoxbjpJo
>>131 ほんまや! さーせん(^p^;
>>132 ツェナーダイオードが電流を流して、
その電流は抵抗器も通るから、
抵抗器でE=IRの電圧降下を生じるから、じゃね? しらんけど
0135774ワット発電中さん2022/05/02(月) 15:44:14.74ID:irrCRK0l
>>128
そんなことよりその図は正弦波もどきで正弦波じゃないものが描かれてるので気持ち悪い
0136774ワット発電中さん2022/05/02(月) 16:13:23.83ID:RoxbjpJo
>>128
なるほどこういうことか・・・
0137774ワット発電中さん2022/05/02(月) 16:28:12.45ID:tuBNfWSD
>>136
それショートしてるし
0138774ワット発電中さん2022/05/02(月) 16:43:03.32ID:HgQ9H0T/
逆起電力ですね判ります
0139774ワット発電中さん2022/05/02(月) 16:55:01.30ID:C36dxlT9
>>136
コンデンサを1つ入れれば倍圧整流ですね。
0140774ワット発電中さん2022/05/02(月) 17:11:22.35ID:RoxbjpJo
>>137
あれ? そうなるのか・・・
オーナンテコッタ<(^p^)>

電位差をほぼダイオードが引き受け
負荷部の緑部がH側の等電位になるだけで
電流自体はごく微小ながら 流れるけどそれでオッケーなのかな
いやバランスせんだろうし ぐぎぎ ようわからん
0141774ワット発電中さん2022/05/02(月) 17:14:07.91ID:RoxbjpJo
ああそっか。
ちょうどさっきの逆で、還流ダイオードが一見ショートに見てたとしても、
「直流のショートとは違って」むげんに流せるわけじゃないんじゃね?
それこそ、飽和しちまうべ。
0142774ワット発電中さん2022/05/02(月) 17:16:16.28ID:tuBNfWSD
>>140
だから、半波整流だからバランスはしないよ
バランスせず動いてるから、それで飽和しない余力のあるトランスが必要なだけ
0143774ワット発電中さん2022/05/02(月) 17:23:09.18ID:RoxbjpJo
いや、煽りじゃなくマジで いみわからん どんな余力じゃじゃじゃい(^p^;
0144774ワット発電中さん2022/05/02(月) 17:30:28.18ID:tuBNfWSD
>>143
バランスしてればBHカーブの中央から対象に使ってるというのはイメージできる?
半波整流だとそれの半分に直流分が重畳されるから飽和する側に動作点がシフトする
だから飽和点がちょっくら大きくなきゃだめよってこと
0145774ワット発電中さん2022/05/02(月) 17:31:14.67ID:RRC5JZB/
>>132
電流で考えるんだ
0146774ワット発電中さん2022/05/02(月) 17:54:47.03ID:tuBNfWSD
あ~、直流分云々は余計だったかもしれない

両波だったら、両側から半分ずつ取り出してる電力を
半波だと片側から取り出さなきゃならんから、その分大電流が必要で
大電流を流すと飽和点が近づくから、飽和天の大きなトランスが必要
こうかな
0147774ワット発電中さん2022/05/02(月) 19:28:08.97ID:cnkOx8Vg
ああ、なるほど!
水色部分の面積と
肌色部分の面積とでバランスするわけね!!(^p^)目からウロコ!!

一次コイルにも半端整流波形しか流れない、というのが盲点でした・・・
0148774ワット発電中さん2022/05/02(月) 19:33:21.04ID:cnkOx8Vg
>>104
新ジャンル
電子(部品 破壊)工作
誕生の瞬間であった・・・
0149774ワット発電中さん2022/05/02(月) 19:57:17.02ID:tuBNfWSD
いや、だから、バランスしないんだけどw
0150(・∀・;バランスするんじゃね?2022/05/02(月) 20:21:31.48ID:cnkOx8Vg
あれ? 二次コイルにぶら下がってる負荷は、変圧比aを用いて
一次換算することができて、トランスを消して「直結」されている等価回路に表現することが出来るけど
電流が1/a、電圧がa倍するとしてもダイオードで一方通行なのは
いかんともしがたいわけで一次側コイルにも半波波形しか流れないと思うんですが
違いました?
0151774ワット発電中さん2022/05/02(月) 20:23:33.37ID:tuBNfWSD
>>150
半波しか流れないからバランスしようがないでしょ
0152774ワット発電中さん2022/05/02(月) 20:54:14.17ID:cnkOx8Vg
あれ? 147は歪んでて伝わらなかったのかな・・・

面積等分になるところでバランスできますよね?
0153774ワット発電中さん2022/05/02(月) 20:56:38.07ID:tuBNfWSD
>>152
その図だと赤も青もどっちもプラス足したらプラス

バランスしてるっていうのは差し引き0
0154774ワット発電中さん2022/05/02(月) 21:02:00.76ID:tuBNfWSD
てか Nの部分の電流はどっから来たの?
0155解説しよう!(^p^)2022/05/02(月) 21:51:33.75ID:cnkOx8Vg
話はいったん飛ぶけど、潮汐力。太陽や月の方向と一致する大潮は納得しやすいけど、
小潮は太陽の逆方向だから一見不可解に見えるんよな。
 あれは、地球と月との合成重心があるんだけど、それは地球の重心とは一致していない。
月ー地球の連星状態に視野を広げると、実は、地球も合成重心を中心に回転運動している。
その遠心力が働いて、重心から最も遠い月の反対側に、小潮は形成されるんやで。

 同様に、半波整流トランスも電流の平均値を基準に見れば、負の電流が発見できて、
それがN極を形成しているのだと思うず(^p^)
0156774ワット発電中さん2022/05/03(火) 00:51:28.19ID:Q2cEAdNM
>>128
2次側に流れる電流に見合った(巻き数比で)電流が1次側に流れるから励磁は増えも減りもしない。
トランスに直流が流れるわけではないからまったく問題なし。
0157774ワット発電中さん2022/05/03(火) 01:15:13.39ID:M1sUTcU3
バランスってなんのことだかわからない
だれか等式を書いてくれないかな
あるいは馬鹿に釣られるのを自粛して欲しい
0158774ワット発電中さん2022/05/03(火) 02:14:39.65ID:M1sUTcU3
バランスがなんのことかわからないのに夢でみて困ったのでへんなことを書きますが
質問者は
1原理的にバランスするはず
2バランスさせなければならない
3バランスさせたい
のどれかだと思われますが、

かみのおつげによれば
原理的にバランスしない。あるていどのアンバランスなら、実用上はゆるされている。
0159774ワット発電中さん2022/05/03(火) 03:40:48.02ID:TmBpwMfK
マイナス側いっぱいに張り付いて励起されててプラス側は素通りってイメージ?
0160774ワット発電中さん2022/05/03(火) 07:59:26.18ID:YTWSSHKT
だれか絵描いてやって
0161774ワット発電中さん2022/05/03(火) 08:04:26.68ID:xa7TQV2K
IGBTって結局GNDは共通なのに
高電圧かけても大丈夫なの?

https://engineer.fabcross.jp/archeive/190529_nedo.html
0162774ワット発電中さん2022/05/03(火) 08:08:16.06ID:YTWSSHKT
>>161
コレクタとゲート間の耐圧のことか?
それは素子の耐圧次第

制御側GNDが高圧回路に接続されてしまうことなら
そうならない回路を採用する
0163774ワット発電中さん2022/05/03(火) 11:01:45.55ID:yAoWk29Z
D級アンプって出力にLCフィルタを繋ぎますが、Cを通ってGNDに逃げる高周波成分は完全にロスですか?
Lを積んでCを削ったほうがロスは減りますか?
0164774ワット発電中さん2022/05/03(火) 12:29:20.80ID:M6j/MCAT
>>161
エミッター<-->ゲート間の電圧が耐圧を超えない様に使う
0165774ワット発電中さん2022/05/03(火) 15:26:47.03ID:juCoCyB9
A級伏見amp
0166774ワット発電中さん2022/05/03(火) 19:37:31.43ID:M6j/MCAT
>>163
自信ないので上手く説明できんかも (;´・ω・)

LC filterのコンデンサに溜まった電荷(エネルギー)がどの様に動くかを考えてみればいいと思う
初心者質問スレ 153 YouTube動画>13本 ->画像>53枚

電源の代わりに電荷がたまった電源側をコンデンサに置き換えて考えるとわかりやすいと思う
出力側のコンデンサに電荷がたまった状態でハイサイドOFF・ローサイドONにするとコイルに電流が流れる
コイルの電流が反時計回りに流れてる状態でハイサイドON・ローサイドOFFにすると電荷が電源側のコンデンサに戻る

つまりDクラスアンプの出力のLC filterは理論的にはロスにならない?
(現実の回路ではコイルの抵抗成分やFETの抵抗成分やら配線の抵抗やらその他やらロスする)

たぶん\(^o^)/
0167774ワット発電中さん2022/05/03(火) 19:44:11.27ID:bP4KXQtp
>>128
教科書読み直しました。

トランスの等価回路ってこうなるんですと。
励磁電流i0 と、補償電流 i'1

負荷に追随するのは保証電流の方で、
ヒステリシス損の励磁電流は 二次巻き線側の負荷とは独立に存在するみたい。
だから、励磁電流はダイオードに関係ないみたいです、サーセン(^p^;)
0168774ワット発電中さん2022/05/03(火) 20:19:57.61ID:98QeuKnr
>>166
無音時もちょこちょこ電流が流れるわけでやっぱりロスではないですか?
0169774ワット発電中さん2022/05/04(水) 01:13:19.96ID:iGDBzhPF
>>167
だから落ち着け。いつも言ってるだろ。

顔文字やめろって。
0170774ワット発電中さん2022/05/04(水) 09:38:07.42ID:DqrrvmcQ
降圧型のDCDCコンバーターで入力12V、デューティー50%の場合、出力電圧の最大値は6Vですか?
0171774ワット発電中さん2022/05/04(水) 10:46:43.86ID:lsY3KZ3M
そんなことはない 切れてる期間に出力がどう下がるかはdutyで決まるわけじゃない
0172774ワット発電中さん2022/05/04(水) 12:19:26.55ID:fgsQ99xh
>>170
コイルに流れている電流が連続モードのときはほぼそうなります。
(コイルをはじめとする電源回路の抵抗値がゼロで、整流ダイオードのVFを0Vとみなしたとき)

でも、軽負荷で電流断続モードにあるときにデューティ50%にしたら高い電圧になってしまいます。
0173774ワット発電中さん2022/05/04(水) 16:12:55.20ID:lUYi43zz
この基板に付いているdelphiのコネクタに刺せる相方を探しています
https://ux.getuploader.com/kajimerental/download/113
https://ux.getuploader.com/kajimerental/download/114
基板素人の自分が探してみても全く検討がつきません
何卒よろしくお願いします
0174774ワット発電中さん2022/05/04(水) 16:49:57.38ID:ZYah6FWX
>>168
それは現実的な回路での話ですか?理屈上(理論)の話ですか?
0175774ワット発電中さん2022/05/04(水) 17:08:05.45ID:3Cnekz35
>>170
降圧型DCDCで、デューティーを50%に制御、あるいは固定したとき
インダクタのインダクタンス、出力キャパシタのキャパシタンス、
スイッチング周期の長さ、負荷に流れる電流によって、12V以上の電圧も生じる
はずだが、やったことないので確証は無い
(1H, 1pF, 1秒, 0Aのときなど)
0176774ワット発電中さん2022/05/04(水) 17:11:51.42ID:3Cnekz35
>>168
無音時にちょこちょこ電流が流れるのは、どういう理屈ですか
0177774ワット発電中さん2022/05/04(水) 17:12:29.66ID:gxBEEPoy
宇宙的な波動
0178774ワット発電中さん2022/05/04(水) 17:41:35.39ID:P12NxenC
ドトール来た。勉強のふりをしてる女の子二人。
勉強は全然せずに、しゃべってばっかり。
そんなんで勉強になるのか。
横にいて、イライラする。
コーヒー半分残して出た。むかついた。
0179774ワット発電中さん2022/05/04(水) 17:50:15.96ID:ZYah6FWX
>>175
コンデンサ0Vで最初のスイッチングONの時間がLC共振の周波数より長ければ理論上は電源電圧の倍まで行きますね
負荷が無い状態で最初の1回は その後は知らん\(^o^)/
0180774ワット発電中さん2022/05/04(水) 17:51:02.43ID:ZYah6FWX
>>178
女の子の写真うp 話はそれからだ\(^o^)/
0181774ワット発電中さん2022/05/04(水) 18:45:09.60ID:CcCmbIcJ
>>173
ここに貼って
http://imgur.com/
0182774ワット発電中さん2022/05/04(水) 18:49:25.90ID:44MNchV+
>>178
ドトールて勉強するとこなんか?
0183774ワット発電中さん2022/05/04(水) 18:50:10.66ID:2DMiae3o
>>181
俺は173ではないけど
初心者質問スレ 153 YouTube動画>13本 ->画像>53枚
初心者質問スレ 153 YouTube動画>13本 ->画像>53枚
0184774ワット発電中さん2022/05/04(水) 18:50:47.45ID:44MNchV+
>>178
ドトールて、勉強は全然せずにしゃべってばっかりだと
隣のやつからイライラされるとこなんか?
0185774ワット発電中さん2022/05/04(水) 18:52:38.70ID:2DMiae3o
今日日喋ってばっかの奴はどこでもイライラされる可能性がある
マスク外してるなら特に
0186774ワット発電中さん2022/05/04(水) 19:04:29.02ID:L+3LztK6
>>184
落ち着くならコメダ珈琲一択
0187774ワット発電中さん2022/05/04(水) 19:09:49.85ID:DItSLmUY
Raspberry PiとArduinoで一つのRTCとOLEDを共有ってできる?
Raspberry Piの電源がOFFの時でもボタンを押したときだけArduinoで時計を表示できるようにしたい
0188774ワット発電中さん2022/05/04(水) 19:21:52.01ID:CfCVBeMj
Arduinoにつないでおいてシリアル通信で共有するとか
0189774ワット発電中さん2022/05/04(水) 19:25:11.57ID:ZYah6FWX
>>187
RTC,OLEDはI2C接続だよね?やろうと思えばできる
raspberryPiがOFFの時のみArduinoから表示するだけなら
raspberryPiの電源状態でI2Cをアナログマルチプレクサで切り替える
Arduinoから初期化コード含めて通信するだけで出来ると思う
本当の意味で共有するならもっと複雑な方法になると思うけど
0190774ワット発電中さん2022/05/04(水) 23:23:14.03ID:P12NxenC
>>179
顔文字やめたら写真送る
0191774ワット発電中さん2022/05/04(水) 23:24:55.64ID:P12NxenC
>>186
コメダでマイコンのデバッグしたら、
何か言われるかな?
0192774ワット発電中さん2022/05/04(水) 23:41:28.32ID:iaIcHUhY
>>181
初心者質問スレ 153 YouTube動画>13本 ->画像>53枚
初心者質問スレ 153 YouTube動画>13本 ->画像>53枚
01931732022/05/05(木) 01:28:50.78ID:drnKaxaW
>>181
すまねぇ・・・すまねぇ・・・
>>183
ありがてぇ・・・ありがてぇ・・・
0194774ワット発電中さん2022/05/05(木) 01:47:18.83ID:lJsh5ml1
>>192
何これ、怪しいから踏まん
0195774ワット発電中さん2022/05/05(木) 04:26:04.79ID:a2HSX0Iy
>>191
逆にドトールで大丈夫なの?元レスの話ではドトールでどうこうって話だったので
0196774ワット発電中さん2022/05/05(木) 06:51:12.70ID:pEdkRauP
OPアンプ使った増幅回路(定電圧制御)が発振してしまいました。
対策は簡単にできたのですが、「なぜ」発振するのかの説明を探しても
いまいちピンとくるものが見つかりません。

いわく「フィードバックとは位相を180度遅らせるわけだから、そのときの
ゲインが。。。。」とかいわれても、位相を「遅らせ」てるわけじゃなくて
単に信号を「マイナス倍」してるだけであって、進み遅れとは関係ない
じゃんとか思ってしまうのです。

なんかうまい説明は無いものでしょうか?
0197774ワット発電中さん2022/05/05(木) 08:21:27.68ID:I+qiTUJm
本当に一文字も違わずに
「フィードバックとは位相を180度遅らせるわけだから、そのときのゲインが」
と書いてあるなら、その本は仕舞っておいて別の本を買ったほうが良い
0198774ワット発電中さん2022/05/05(木) 08:43:42.46ID:shmOMw6E
>>196
その時の発振周波数は? どういう対策をしたの?
0199774ワット発電中さん2022/05/05(木) 08:48:33.40ID:wik5dWO8
>>196
こんな説明はどうだろう。
オペアンプで直流を扱っているつもりでも、実はいくらでもノイズ源があって、意図していなくても交流増幅を
並行して行っている。

ノイズといっても、無数の正弦波の集まりとも考えられるのでとりあえずひとつひとつの正弦波で考えてみる。
正弦波をマイナス倍にした波形は180度遅らせた波形と同じになる。

ということで、使う人は、正出力を負入力に戻すことで「180度遅らせたものを戻しているつもり」でいる。
でも実際にはオペアンプ自体の遅れがあって「180度の時間+遅れ時間」の遅れがある。

こんな単純なものではないけれど…
あるオペアンプ自体の遅れが1μ秒だとする。
ノイズを作っているたくさんの正弦波のうちの500kHz成分を考えてみる。
500kHzの180度は1μ秒だけど、「180度の時間+遅れ時間」は2μ秒になって1周回って0度の帰還をすることになる。
0200774ワット発電中さん2022/05/05(木) 10:14:03.12ID:Y4CcLdX1
遅れた位相に進み補償をかけて元に戻す。実はタイムマシーンなんだ。
これは、有名な映画にもなった。名付けて、

 ”バック・トゥザ・フェイザァー”

このバック・トゥザ・フェイザァー現象を発するには、スーパーOPアンプ、
デロリンに1ジゴワットの電力エネルギを印加して発振させる必要がある。
0201774ワット発電中さん2022/05/05(木) 12:46:51.66ID:jN8Lx9QP
回路構成がわからないから何とも言えないが
単に信号を「マイナス倍」して戻すだけなら何も変わらないだろ
信号に何かしらの手を加えてフィードバックするのだから
その過程で位相に変化が生じてるってことでないの
0202774ワット発電中さん2022/05/05(木) 15:59:22.86ID:GclVWQy8
>>191
似て非なるか?w
 入浴中はフローやアルゴリズムなどを検討する程度にしているが、
この前、浴槽で温まっているときに、つい夢中になって
サブルーチンを1ステップづつ組んでいた。(もちろん頭の中で)
すると突然、カミサンが風呂場の扉を開けて
「ずっと何の音もしないから心配で見に来た」
だって。
反省ッ!
0203774ワット発電中さん2022/05/05(木) 16:09:55.53ID:q+dxxExp
近所の老人が最近風呂の中で煮物になってたな
一か月くらい浸かってたとか
0204774ワット発電中さん2022/05/05(木) 16:13:00.02ID:v9NBzOxx
低温調理かよ
0205774ワット発電中さん2022/05/05(木) 17:04:32.15ID:LFogtpNw
美味しそう(エクスカニバー
0206774ワット発電中さん2022/05/05(木) 18:24:07.47ID:aOC/cKDw
>>196
入力端子の差分が常にない状態に保てるなら、それはフィードフォワード制御
フィードバックは差分が発生してから制御するので必ず遅れが発生する

その遅れ時間が180度分になる周波数が発振のピーク周波数
たぶん
0207774ワット発電中さん2022/05/05(木) 20:03:07.73ID:kk8rXIqS
古いパソコンやゲーム機などで使われているRGB映像信号の規格に関する資料ってどこにあるのでしょうか
ググってもRGB信号の仕様も同期信号の仕様もよく判りませんでした
0208774ワット発電中さん2022/05/05(木) 21:19:05.34ID:anyNwgR+
ケースに電池ボックスを埋め込む方法を色々探してたら、
フタにネオジム磁石を使った方法が見つかりました。これ良いアイデアですね。
3Dプリンター持ってないんで、これを真似しようと思ってます。

@YouTube



タカチで埋め込み用電池ボックスの完成品が売ってるんですが、
https://www.takachi-el.co.jp/products/LD
Aliで買った電池ボックスが結構余ってて消費したいので、他に良いやり方とかあればお教えいただきたいです。
0209774ワット発電中さん2022/05/05(木) 23:56:36.34ID:AgqyUbsM
>>207
詳しくないけど足がかりにどうぞ
https://ja.m.wikipedia.org/wiki/RGB21%E3%83%94%E3%83%B3
0210774ワット発電中さん2022/05/06(金) 02:51:00.11ID:y5eAIsC4
こういう入力増やしても増えるか減るか分からない系ってどうやって制御するんですか?
デジタルでififしながら総当たりしかないんですか?
0211774ワット発電中さん2022/05/06(金) 07:29:10.05ID:FrwYovLz
微分
0212774ワット発電中さん2022/05/06(金) 07:30:27.70ID:xW+lX+Fl
>>208
電池の消費次第、あそこまでやらなくても普通にケースに入れれば良いかも
年に一回、数年に一回程度ならねじを外すのは苦にならないのでは?
>>207
http://ele-tech.net/vga-doc1/
https://download.tek.com/document/SD_HD_ApplicationNote_25Z-14700-05.pdf
0213774ワット発電中さん2022/05/06(金) 07:41:13.71ID:xW+lX+Fl
>>210
簡単なのは上り下りを判断して下り始めたら戻る、最大値をメモリーしておきそこに戻る
ないしは登りと下りの同じ値になるところの中間を最大とする
参考:カメラレンズの場合
http://tokkyoj.com/data/tk2008-242226.shtm
http://tokkyoj.com/data/tk2007-78908.shtml
0214774ワット発電中さん2022/05/06(金) 08:27:45.25ID:eh+0WMAG
>>210
その曲線を得る近似式はないの?
0215774ワット発電中さん2022/05/06(金) 10:50:05.31ID:mo6lNkbU
>>196
これ読んで理解できなきゃ伝達関数、フーリエ変換やラプラス変換を勉強することだな
https://www.tij.co.jp/jp/lit/an/jaja130/jaja130.pdf
0216774ワット発電中さん2022/05/06(金) 11:44:24.88ID:YsCBWuQ3
ナイキスとサンキスト、線図と千擦りの違いが分かりません。
0217774ワット発電中さん2022/05/06(金) 12:44:58.47ID:cjMgHsrv
>>215
それの執筆担当者も「岡本廸夫:定本OPアンプ回路の設計 1993」を読んで勉強したんだろうな
0218774ワット発電中さん2022/05/06(金) 13:30:02.69ID:YsCBWuQ3
しかし、津波の影響は防げなかった。
0219774ワット発電中さん2022/05/06(金) 19:44:32.11ID:dG8GN/gF
いまこういう低周波治療器アダルトグッズ的なのを作ろうと思っています
初心者質問スレ 153 YouTube動画>13本 ->画像>53枚

周りはおゆまるで作成して中に電極としてアルミテープを埋め込もうと考えています
このように中に埋め込んだ場合、低周波治療器として動作しますよね

下のサイトによると低周波治療器のパッドの部分は絶縁体だからこのような等価回路になるようです
https://www2.hamajima.co.jp/~tenjin/labo/lowfreq.htm
初心者質問スレ 153 YouTube動画>13本 ->画像>53枚

また、おゆまるの中に針金を入れた場合人体の方に電流は流れますか?流れない?
1000円くらいで作れるんでとりあえずやってみようと思ってますが
初心者質問スレ 153 YouTube動画>13本 ->画像>53枚

ちなみに流す電流は10mAくらい?オムロンのエレパレス使おうと思ってます
0220774ワット発電中さん2022/05/06(金) 19:58:39.35ID:FrwYovLz
電極が露出してないと流れないよね。
0221774ワット発電中さん2022/05/06(金) 20:11:39.57ID:DVwlH+ur
>>196
外乱に対処しようと遅れて大きく操作すると、偏差がいつまでも収束せずに
時には発散して大変なことが起きる

 死亡事故例:フェデックス80便着陸失敗事故

系全体として、フィードバックゲインが高く、操作が遅れると振動的になり、
ゲインが1になる周波数でいつまでも振動し続けることになる。

振動させないためには、すばやく(進み補償を追加する)、小さめのフィードバック
が肝要。
0222774ワット発電中さん2022/05/06(金) 21:11:12.35ID:JuFkDcNO
アナログスイッチを作りたいんですがボディーダイオードの無いMOS-FETまたはエンハンスメントのJ-FETは無いですか
0223774ワット発電中さん2022/05/06(金) 21:15:11.99ID:dG8GN/gF
>>220
こういうパッドの接触部は数MΩらしい
初心者質問スレ 153 YouTube動画>13本 ->画像>53枚
0224774ワット発電中さん2022/05/06(金) 21:21:05.56ID:CCRW8UpF
スキャンレート?が10kHzくらいあるジャイロと加速度計ってありませんかね
秋月とかでも9軸ジャイロとか売っていますが遅いと数百Hz、速くても数kHzのような
0225774ワット発電中さん2022/05/06(金) 21:37:19.09ID:uX7sydTh
単にスキャンレートを上げたいならアナログセンサを使って、その出力を適切なAD変換器で処理すればよい
まぁセンサそのものの応答が遅い場合はあまり意味はないと思うけど
0226774ワット発電中さん2022/05/06(金) 21:54:16.64ID:4PRqU6lZ
>>222
CD4007の中にそういうのが入っていた。
0227774ワット発電中さん2022/05/06(金) 22:14:08.33ID:JuFkDcNO
>>226
ディスクリートで作ってみたいなと思ったんですが無いみたいですね
0228774ワット発電中さん2022/05/06(金) 23:13:30.57ID:4PRqU6lZ
CD4007は中の一部のFETをディスクリートで使えるよ。
0229774ワット発電中さん2022/05/07(土) 01:11:15.78ID:/c+Q2NmS
出力5Vのソーラーパネル(USB出力付き)から
モバイルバッテリーに充電しっぱなしにしてたらやっぱマズいですか?
過充電になってしまいますかね
モノは↓です
https://www.sparkfun.com/products/16835
0230774ワット発電中さん2022/05/07(土) 02:21:38.32ID:5Sje/HB4
モバイルバッテリー側に制御回路があるから大丈夫。勝手に止まる
制御回路が糞なら知らん
0231774ワット発電中さん2022/05/07(土) 10:27:58.68ID:D3E8y93p
>>222
バックトゥバックMOSFETじゃだめ?
0232774ワット発電中さん2022/05/07(土) 10:40:27.87ID:etFIa4Np
>>231
たしかにそう。
アナログスイッチの出来合いのものではなくて、自分で作るのはどういう目的なんだろうって気はする。
性能の観点だとなかなか既製品には勝てないし。
0233774ワット発電中さん2022/05/07(土) 14:22:12.20ID:8oU+nG6l
出来合いのアナログスイッチ
今の定番てなに?
0234774ワット発電中さん2022/05/07(土) 14:52:29.31ID:dM5AOZZt
ボディーダイオードは構造上の話だから。
0235774ワット発電中さん2022/05/07(土) 15:49:13.13ID:etFIa4Np
>>233
昔は4066とか4051〜3が定番だったけどそれは品種も少なかったから。
今はON抵抗が低いとか、チャージインジェクションがとか、スイッチ間容量がとか、スイッチスピードだとか、OFFリークとか、
いろいろ得意なものがあるし、これといった定番なんてないと思う。カタログを見て悩めばいいと思うんだ。
0236774ワット発電中さん2022/05/07(土) 16:03:00.67ID:etFIa4Np
思い出した。2N4351。4端子で、ソースとボディがつながっていない。
0237774ワット発電中さん2022/05/07(土) 16:05:58.54ID:8oU+nG6l
>>235
そのいろいろ厳しくない用途で使える万能選手みたいのないかな
いちいち選定するのめんどくてw
0238774ワット発電中さん2022/05/07(土) 16:14:22.22ID:etFIa4Np
>>237
部品のストックを考えるなら、妥協を前提に品種は絞れますね。

4066も4051〜3もロジック電圧以外で使うときには面倒なことがあるけれど、
電圧条件さえOKなら、いろいろ使える。昔はこれでなんでもやってきたし。
たいして難しくなくて、すこしぐらい抵抗が大きくてもいいや、っていうことなら
スイッチ構成、アナログ電圧範囲が決まればある程度は絞れるのでは。
あと、人によってはDIPじゃないと嫌だとかあるか。
0239774ワット発電中さん2022/05/07(土) 16:18:22.59ID:8oU+nG6l
>>238
4051時代によく壊れてから、頑丈なのがいいなあw
0240774ワット発電中さん2022/05/07(土) 16:20:24.92ID:etFIa4Np
それは使い方が悪いのでは。
データシートに書かれてあることを守らないことを前提に部品選定するのは無理。
0241774ワット発電中さん2022/05/07(土) 16:22:25.09ID:8oU+nG6l
>>240
設計は他だからね~、うちは修理屋、
修理に入ってきた基板にそれのってたら、真っ先にそれ調べて、ああやっぱりてなことが
すごくよくあった
0242774ワット発電中さん2022/05/07(土) 17:32:29.73ID:TKkW1bJk
4051〜4053とか、74HCであるんじゃないかな。
74HC4053 とか。
0243774ワット発電中さん2022/05/07(土) 18:02:49.61ID:NAgM1HvH
多CHデータロガーなんかはフォトMOS使っているね。
0244774ワット発電中さん2022/05/07(土) 18:06:46.51ID:GL7lpJrs
>エンハンスメントのJ-FET
って存在するの?
0245774ワット発電中さん2022/05/07(土) 19:39:58.74ID:YM3LosgK
初心者質問スレの筈なのに
真正電子工作初心者の俺様から見ると
アウェー感半端ねえぞ(^p^;)お前らナニモンだよw
0246774ワット発電中さん2022/05/07(土) 21:01:08.05ID:etFIa4Np
>>243
遅くていいなら、とか、端子間静電容量が大きくていいなら、とか制約はあるしね。万能はないな。

>>244
構造的に無理そう。
0247774ワット発電中さん2022/05/08(日) 01:40:29.39ID:994x8/gS
>>245
顔文字やめような。
0248774ワット発電中さん2022/05/08(日) 12:16:36.27ID:XwB6D+M7
Dフリップフロップに関する質問です。

https://akizukidenshi.com/catalog/g/gI-10879/

上のICを買ったのですが端子を

CLR(クリア):+
PRE(プリセット):+
CLK(クロック):プルダウン-
D(データ入力):オープン(不定)

の様に配線した状態で電源ONにすると必ず Q(出力) は LOW で開始されます。
てっきり電源オン直後は、Qは不定でHIGHだったりLOWだったりして最初にCLRをLOWにしてQを強制的にLOWにする
作業がいるかと思っていました。

これってICが自動で電源投入直後に CLR(クリア) をLOWにするような仕組みになっているからでしょうか?
0249774ワット発電中さん2022/05/08(日) 12:55:34.49ID:vZVYVDbz
>>248
https://toshiba.semicon-storage.com/jp/semiconductor/knowledge/faq/logic_cmos/logic_cmos_01.html
0250774ワット発電中さん2022/05/08(日) 13:07:36.67ID:XwB6D+M7
>>249
そこを読むと電源投入後の出力状態は決まっていないとかいてありますが、何回ON、OFFを繰り返しても
>>248の状態だと必ず初期の出力はLOWになります。
なぜでしょうか?
0251774ワット発電中さん2022/05/08(日) 13:29:41.79ID:j6mAJrq+
Qにブローブやテスタリードを当てているなら負荷容量の関係で立ち上がりが遅いからでは
メーカにもよりますがD-FFはRSラッチ2段構成とトランスファゲート2段構成とがあります
0252774ワット発電中さん2022/05/08(日) 13:45:42.47ID:mfLmD3P8
普通は学校で習う

初心者質問スレ 153 YouTube動画>13本 ->画像>53枚
0253774ワット発電中さん2022/05/08(日) 14:01:42.83ID:+wMGoRgv
>>252
みんながみんな工業高校に行ってる訳じゃないんだよ。
0254774ワット発電中さん2022/05/08(日) 14:14:58.06ID:OJOScc8I
「工業」高校が「普通」とはこれ如何にw
0255774ワット発電中さん2022/05/08(日) 14:32:00.01ID:994x8/gS
>>248
>これってICが自動で電源投入直後に CLR(クリア) をLOWにするような仕組みになって
たまたま、そのICの個性だと思います。
電源on時にLowにする回路は入っていません。

Dataピンを、openでなく指で触りながら電源onしたときは、Hも出ませんか?
0256774ワット発電中さん2022/05/08(日) 16:28:10.56ID:XwB6D+M7
>>255
Dataピンを+に繋ぐ、-に繋ぐ、指で摘む、を試しましたが全て出力はLOWでした。

https://imgur.com/X7tJCTl

上のデータシートにある回路図に

CLR(クリア):1
PRE(プリセット):1
CLK(クロック):0
D(データ入力):不定

を書き込んで出力が決まるかやってみようと思ったのですが三角形が4つ集まった記号や
インバータの先にφと書かれた記号が出てきてよくわかりません。
何を表す記号でしょうか?
また上の値を図に書いていって出力Qは一意に決定されるでしょうか?
0257774ワット発電中さん2022/05/08(日) 16:49:20.93ID:994x8/gS
>>256
そうでしたか、変化無しですか。
もともとD-FFは、使用前にユーザーがCLRまたはPRをするのが前提の素子ですので、
電源on時は不定(Hばっかり、H/Lf混合、Lばっかり)のどれでも、製品としてはOKなのだと思います。

そもそもD-FFは、
CKの↑の瞬間(エッジ)しか動かない(出力Qに伝達されない)素子ですので、
図のCK端子から入ってきてφと/φを作って、
そのφの変化の瞬間だけ、スイッチの左と右が「接続」され、それ以外のときは「断」になって繋がりません。

なので、φの↑(/φの↓)変化の直前の電圧が瞬間的に導通される、と読んで下さい。

一方、AND, ORなどは、組み合わせ論理回路と言って、1, 0 1,0 と追っていける回路です。
0258774ワット発電中さん2022/05/08(日) 17:08:17.14ID:994x8/gS
>>256
ダイヤのような記号は、φで操作される「スイッチ」です。

図の「...の時に導通?」の示している所をよく見ると、NOT物がお互いに向き合って、
リング上の回路になっています。このリングにφからの信号でNOT記号の○スイッチが動作すると、
Dからの信号が繋がって、リングに読み込まれ、次のφのときまで維持されます。
後段にももう一つリングがありますね。
こんなイメージです
0259774ワット発電中さん2022/05/08(日) 17:08:29.90ID:CS7suDNP
>>256
完全に均等に製造できればパワーオンで0になったり1になったりするけど、少しでも不均等
があればほぼ同じ値になる。
マイコンのRAMなんかも同じ。
0260774ワット発電中さん2022/05/08(日) 18:17:54.78ID:YmRBXCM9
>>245
ニューカマーを生暖かい目で見守る、ジイサンズ
0261774ワット発電中さん2022/05/08(日) 18:37:24.28ID:J7lTqyDm
カニカマで天津飯作ろう
0262774ワット発電中さん2022/05/08(日) 19:52:18.60ID:sE6ig52F
RTCで使うLIR2032ってBluetoothイヤホンとかにあるリポじゃ駄目なんかな
0263774ワット発電中さん2022/05/08(日) 21:34:58.46ID:zEf2EmER
>>262
充電電圧が4.2以下ならいいんでない?
0264774ワット発電中さん2022/05/08(日) 22:07:34.49ID:XwB6D+M7
>>258,259
同じICを4個買っていたので4個とも同じ配線で電源ON、OFFを繰り返しても
やはり一番最初の出力は必ずLOWになりました。

しかし>>249のリンク先でメーカーが初期状態は不定と言っている以上、確実にクリアをかけるには
CLR端子に抵抗+コンデンサの遅延回路を作って電源オン時にオートリセットするようにした方がいいんですかね?
0265774ワット発電中さん2022/05/08(日) 23:24:33.86ID:isqYmAFo
好きにすればいい
定格外の電源電圧で試したら動いたから使って良いですか?ってのと同じ
0266774ワット発電中さん2022/05/08(日) 23:35:23.05ID:jMoq61D+
今晩は
12Vのリレーをスマートホンのマイク端子につないで電気毛布の上に置いてみると
繰り返し周期50Hzのデータを取得できました。
もう少し大きな入力が欲しいと思いリレーを二つつないでみたけど大きくなりません。
なぜでしょうか?。
0267774ワット発電中さん2022/05/08(日) 23:42:59.98ID:K4OmQVo1
位相があってないから
0268774ワット発電中さん2022/05/09(月) 01:38:37.36ID:CXTUo8r5
ラ製の入門工作にジャンケンポンマシーンや電子サイコロがあったな
ランダムに動作させるのは結構難しい
0269774ワット発電中さん2022/05/09(月) 06:57:10.58ID:7an/zC9U
ソーラーパネルからヒ素が出ることってあるの?
0270774ワット発電中さん2022/05/09(月) 07:00:09.26ID:hsnYYo3j
>>266
2つのリレーをどうつないだのか? どう配置したのか?
0271774ワット発電中さん2022/05/09(月) 08:11:14.89ID:iC+oLvtV
ROHMの5V1A出力の三端子DCDCコンバータですが、無負荷時は5.06Vですがオペアンプ1つ繋ぐと4.36Vまで下がり22Ωを繋ぐと1.63Vまで下がるのですがこんなもんですかね?
あまりにもひどすぎるスペックだと思うのですがやはり安定度を求めるなら普通の三端子レギュレータが良いですか
0272774ワット発電中さん2022/05/09(月) 08:13:27.92ID:xipTsLBR
>>266
二個のリレーは直列に繋ぐ
二個のリレーは同じ姿勢でできるだけ近づけて設置する
これでどうだろう
0273774ワット発電中さん2022/05/09(月) 08:37:34.40ID:SuXtJB2I
>>271
それは使い方を間違っている可能性が高いです。
DC-DCコンバータの型式を含めて、周辺の回路図、元電源のスペックを出してみたらどうでしょう。
0274774ワット発電中さん2022/05/09(月) 08:51:16.97ID:iC+oLvtV
>>273
すいません入力に100uF繋いだら安定しました
入出力コンデンサ不要と書いてあったのですが必要なんですね…
0275774ワット発電中さん2022/05/09(月) 09:05:42.63ID:JbPt542s
入力側につないだ電源の都合だったりして・・・しらんけど
0276774ワット発電中さん2022/05/09(月) 12:00:25.03ID:wg91fvKf
>>265
同感
0277774ワット発電中さん2022/05/09(月) 12:08:46.08ID:wg91fvKf
>>274
入力側の回路は?
まさかダイオード整流だけして平滑して無いとかないよね?
0278774ワット発電中さん2022/05/09(月) 13:44:07.83ID:4XvpYCHO
あの、テスターでバイクのバッテリーはかってたらなんか測れなくなって、
それはヒューズが飛んでるせいだってわかったから買ってきて付け替えたら測れるようになったんだけど、
20vでテスターの棒当てても最初測れなくて、なんでかなーと思いつつも
200vに合わせて測ったら本来でる13v付近じゃなく26vとか表示されたんだ
要するに倍ぐらいの値が表示されるようになっちゃったんだけど
これなんのせいなの?
0279774ワット発電中さん2022/05/09(月) 13:46:08.28ID:J26cIEwl
気のせい
0280774ワット発電中さん2022/05/09(月) 13:46:59.71ID:nO49UgQU
>>278
壊してしまったか、読み間違えてるかのどっちか
0281774ワット発電中さん2022/05/09(月) 13:51:42.18ID:J26cIEwl
真面目な話、テスターを買い足して、疑いあるテスターとバイクを調べる
そんな面倒な、と思うならバイク屋行くのがいい
0282774ワット発電中さん2022/05/09(月) 13:53:34.16ID:J26cIEwl
計器なんて、適正に測った上でその値を疑った時点で価値はゼロ
0283774ワット発電中さん2022/05/09(月) 14:46:34.93ID:T8JO7j+k
間違えて交流レンジ使ったらそうなったことがあったな
0284774ワット発電中さん2022/05/09(月) 15:12:11.43ID:lkxUwnje
>>278
少しエンジン吹かして更に電圧上がるようなら
ボルテージレギュレータ死んでるんじゃね?
0285774ワット発電中さん2022/05/09(月) 15:24:14.26ID:9shr9UjO
>>283が当たってるんじゃないかな?
直流で計らないと
0286774ワット発電中さん2022/05/09(月) 15:27:26.49ID:xRZgBczq
DCDCコンバータのインダクタに流して良い最大電流はどう計算すればいいんですか
例えばFT-114-77(飽和磁束密度4900ガウス)のコアに9回巻いた(100uH)インダクタの場合は50kHzのDUTY50%のパルスを何Aまで流せるんでしょうか

初心者質問スレ 153 YouTube動画>13本 ->画像>53枚
https://www.amidoncorp.com/product_images/specifications/77_Material.pdf
0287774ワット発電中さん2022/05/09(月) 15:42:56.12ID:DseuHzCp
総磁束量は大文字でΦと書く で、もって Φ=nφ
小文字のφは磁芯の磁束量 でもって、 φ=BS

更に、 Φ=nφ=Li   でもって、Φ=L*i=Vi*ton  だから 100uHの
インダクタには何Vかかるかが決め手となるニダ それに、インダクタは毎回
励磁をリセットする必要がある場合とない場合がある。この場合、印加電圧が
±かまたは、+電圧だけかで条件が異なるね。   
0288774ワット発電中さん2022/05/09(月) 20:01:52.69ID:25rIHi0C
>>286
インダクタにパルス電圧をかけるとノコギリ歯状の電流になるのでピーク電流で考える必要がある
参考
https://cc.%63qpub.co.jp/system/contents/1714/
0289774ワット発電中さん2022/05/09(月) 20:40:04.55ID:EkCMDxZC
ノイズについて質問です

ブレッドボード上でDフリップフロップ(TC74HC74AP)の出力にLEDを繋ぎON、OFFさせていたのですが
そばにある電気スタンド(LEDではなく蛍光灯)を切ると、必ずフリップフロップに繋いで点灯していたLEDが消灯します。
(フリップフロップのデータ入力をHIGHにしながら電気スタンドを切った場合はLEDが消灯→点灯となる)

どうも、電気スタンドのオフに合わせてノイズが出ていてそれがフリップフロップのクロック端子に入っているようなので
オシロスコープで確認したところ電源電圧が3.3Vに対しノイズは約2V出ていました。

ちなみに理屈はわからないですが電源電圧を5Vに上げるとなぜかノイズの測定結果も約3~4Vに上がりました。

そこで以下のようなノイズ対策をしてみました。

効果のあった対策
・クロック端子に繋いでいるジャンパ線を短くする
・GNDから金属トレーにアースする
・電気スタンドから3メートル以上離れる

効果がなかった、あまりなかった対策
・金属トレーで覆う
・470μFのコンデンサをクロック線と並列接続
・GNDから人体へアース

こういった場合どういったノイズ対策が有効でしょうか?
0290774ワット発電中さん2022/05/09(月) 20:50:10.54ID:pUWO9WTB
>>289
回路図、基板写真、電源と接続ライン
情報が足りない
適当でよければclkラインをシールド
0291774ワット発電中さん2022/05/09(月) 21:26:35.98ID:EkCMDxZC
>>290
https://imgur.com/bbo6RzT
ゴチャゴチャしていてすいません、ブレッドボードの写真です。

・ラズベリーパイ(電源、クロック入力、データ入力) ─ Dフリップフロップ

・安定化電源 ─ Dフリップフロップ(手動でクロック、データ入力)

上の2パターンで試しましたがどちらもノイズでLEDが消えました。
0292774ワット発電中さん2022/05/09(月) 21:36:40.84ID:pUWO9WTB
>>291
エスパーするとバスコン
0293774ワット発電中さん2022/05/09(月) 22:44:41.85ID:dH4NHonr
>>291
コモンモードなのでクロックとグラウンド線をピッチリ平行にする。その上でコモンモードフィルタつなぐ。
クロックをシールド、クロックへの結線を短く、ダメもとでクロックにLPF
でしょうね
0294774ワット発電中さん2022/05/09(月) 23:25:55.58ID:1+PnFWY+
>>289
クロックSWのpull-downに不具合が有る
・ジャンプワイヤの不良(か接触不良)
・pull-down抵抗の接触不良
・このタイプのブレッドボードは接触不良が多い
・このタイプのジャンプワイヤは断線・接触不良が多い
たぶん\(^o^)/
0295774ワット発電中さん2022/05/09(月) 23:49:12.33ID:f9LiXk8U
>>269
ガリウム砒素なら
0296774ワット発電中さん2022/05/10(火) 00:05:59.61ID:xk6/40iD
>>294
顔文字、やめてください
0297774ワット発電中さん2022/05/10(火) 01:09:49.66ID:NrU/3LmV
>>296
大抵こういうのは1年持たないから放っておいたらいいよ
夏まで持たないかもな
0298774ワット発電中さん2022/05/10(火) 06:09:47.99ID:qijeO+vx
>>287
>>288
結局最大電流ぶんのコアを用意しないといけないってことですか?
スイッチング電源の利点はコアを小さくできることだと思うのですがその理屈だと小さくならないと思うのですが
0299774ワット発電中さん2022/05/10(火) 06:36:44.63ID:a9gQRqqe
>>298
ACアダプタのトランスサイズと勘違いしてない?
0300774ワット発電中さん2022/05/10(火) 06:45:48.28ID:+E87HMUs
だから、電流はコアが磁束飽和しないよう巻き数が必要ということ。
コアサイズは別途、鉄損などの損失発熱量で決めるんだよ。
0301774ワット発電中さん2022/05/10(火) 07:33:42.63ID:pZA54o5z
>>298
「スイッチング電源の利点は(周波数を高くすることで)コアを小さくできること」です。

元の
FT-114-77(飽和磁束密度4900ガウス)のコアに9回巻いた(100uH)インダクタ
と、
FT-114-77(飽和磁束密度4900ガウス)のコアに3回巻いた((100÷9)uH)インダクタ
だと、後者の方が高いピーク電流を流すことができることにならないですか。
もちろん、後者の方が高い周波数でのスイッチングが必要になります。
0302774ワット発電中さん2022/05/10(火) 08:07:04.76ID:qijeO+vx
>>301
なるほどインダクタンスを削れるんですね
理解できましたありがとうございます
0303774ワット発電中さん2022/05/10(火) 12:44:36.75ID:M3yLnUSL
>>292-294
ありがとうございます、シールド線を買ってきてそれでジャンパ線を作ろうと思います。
0304774ワット発電中さん2022/05/10(火) 15:52:11.97ID:K3J3xGRO
スイッチが一瞬ONになったら3分くらいモーターを動かせるようなタイマーみたいなキット
ってありませんか?

具体的には地震が起きて鉄球の玉が揺れて鉄の板と当たると、通電してモーターが起動するっていうものを
作りたいなと思っています。
ただ回路的な知識はほぼなくて、はんだ付けとかトランスミッターの秋月のキットを作った事あるくらいのレベルですので
一からは無理だと思うのでキットがあればと思ったのですが…

もしくはキットはないとして、簡単に自作できるものなのでしょうか?
0305774ワット発電中さん2022/05/10(火) 17:24:48.93ID:uWIlZPeJ
どんな(種類やサイズの)モーターを想定してるん?
0306774ワット発電中さん2022/05/10(火) 18:15:36.48ID:RNk5FvNQ
https://akizukidenshi.com/catalog/g/gK-11000/
0307774ワット発電中さん2022/05/10(火) 18:51:21.46ID:OfZITNHC
地震計で鋼球式というがあるよ
(例)
オムロン(omron) 感震装置 D7H 鋼球式感震装置
0308774ワット発電中さん2022/05/10(火) 19:13:04.49ID:vxtaX/xF
555爺はよ
0309774ワット発電中さん2022/05/10(火) 19:45:31.46ID:zqUdKXtX
揺れたら動くだけでいいなら水銀スイッチとか
0310774ワット発電中さん2022/05/10(火) 19:51:45.43ID:aRBPYjI7
ラッチリレーとプログラムタイマ使った方が早い気が
0311774ワット発電中さん2022/05/10(火) 21:14:18.31ID:u/SvDDqD
https://aliexpress.com/item/32770099729.html
https://aliexpress.com/item/1005004052806508.html
0312774ワット発電中さん2022/05/10(火) 21:15:34.23ID:u/SvDDqD
https://aliexpress.com/item/10000003359075.html
0313774ワット発電中さん2022/05/10(火) 21:20:03.98ID:mSBd3a3Q
>>308
https://akizukidenshi.com/catalog/g/gK-11000/
やっぱりこれだよな。
0314774ワット発電中さん2022/05/10(火) 22:20:47.96ID:nrm55R2V
>>289
対策は2つあるよ。
1. ブレッドボードの使用をやめる。接触不良続発なので、不燃物として出す。
   半田湖で一式とユニバーサル基板を用意して、ハンダ付けで実験する。

2. HC74のCK端子とGND間に10k程度の抵抗を付ける。
  (またはCK端子とVDD間)
   入力インピーダンスを下げるのが目的。これでほぼ直る。

CK端子はどこから信号をもらっているのか書かないと、適切な回答はない。
まさかオープンコレクタの出力をそのまま繋いでいるだけじゃないだろうね?
0315774ワット発電中さん2022/05/10(火) 22:33:12.08ID:Ow17l8b5
>>308
もうレスされてる >>306
0316774ワット発電中さん2022/05/11(水) 00:01:51.02ID:Yevm/5LB
>>305
タミヤの工作のギヤキットとタイヤが動く電池で動くミニ四駆のモーターみたいなセットのやつを動かしたいです。
0317774ワット発電中さん2022/05/11(水) 00:37:54.34ID:8QKWWHw0
>>314
信号入力は手動とラズパイで試してブレッドボードの写真は>>291です。

クロックへのジャンパ線をシールド線に換えようと思ったのですがその前にジャンパ線にアルミ箔を巻くのを試したら効果絶大でした。

・アルミ箔をジャンパ線に巻くだけ→効果0
・アルミ拍から金属トレーへアース→効果ほぼなし
・アルミ箔からGNDへ接続→一切ノイズの影響を受けなくなった
0318774ワット発電中さん2022/05/11(水) 06:56:35.80ID:wqLebhJ/
>>317
マイコンのプログラムの実行と違って、素のフリップフロップは時間的に微小な信号に敏感だしね。
>>314はソルダレスブレッドボードをやめよう、って勧めている。
俺もフリップフロップで遊ぶのには、ブレッドボードは難度の高い気づかいが必要になると思う。
用途によっては有益なツールであることは理解してるつもり。
あと、手書きでもいいから回路図は書くようにしよう。
0319774ワット発電中さん2022/05/11(水) 07:05:36.69ID:MAtBs+e4
>>317
74HCに限らずICのVcc-GND間にはおまじないで0.1uF付けるというのを見たことがあると思う
これ入れてる?
特にFFはこの効果が絶大で入れたら動作おかしかったのがピタリと治まった事があった
ああ、このおまじないやっぱし必要なんだなと再確認できた
0320774ワット発電中さん2022/05/11(水) 07:19:52.77ID:wqLebhJ/
C-MOS(マイコンでも)は、内部外部でH、Lが入れ替わるときにスパイク状の電流が流れます。
一般的に電源の配線は抵抗値が低いように見えていても、インピーダンス(交流的な抵抗値。スパイク電流は交流成分です)が高く、
なにもしない電源線は、C-MOS ICの動作にともなうスパイク状の電流変化により、スパイク状に電圧が変化します。
よく使われる0.1uFのコンデンサはこのスパイク状の電流を吸収する働きをします。おまじないじゃありません。
0321774ワット発電中さん2022/05/11(水) 07:34:03.38ID:Vs83fjDB
>>317
解決おめでとう
でもシールドが効いたという事は
やはりCKのラインが通常の3倍以上ハイインピーダンスになっているんだと思うよ
0322774ワット発電中さん2022/05/11(水) 08:36:57.07ID:Zbn7YndX
ハイインピーダンスという字面にはすでに意味というか役割がある言葉だから、
通常の三倍ぐらいではちと足りないんじゃね?しらんけど;^p^)
0323774ワット発電中さん2022/05/11(水) 09:48:25.32ID:wqLebhJ/
「3倍」がどこからきたのか興味はあるな。根拠のない数字は却ってオカルト。
0324774ワット発電中さん2022/05/11(水) 11:05:30.40ID:+tdw8175
>>317
○○したら直ったのは、よくわかりました。
それよの、なぜそのようになったかを知った方が、今後に役立ちますよ。
74HC74のCK線は、何に繋いでいますか? (何から信号をもらっていますか?)

紙に回路図を書いて写真を撮って、upしてみませんか?
0325774ワット発電中さん2022/05/11(水) 11:10:39.09ID:+tdw8175
誤記訂正
× それよの、
○ それより、
0326774ワット発電中さん2022/05/11(水) 20:31:27.78ID:GuVcXacq
シミュレータで分かったつもりになってるより実際に動かすとなかなかうまくいかないと分かるから
ブレッドボードでも蛇の目基板でも動かしてみるのはいいことだよね
0327774ワット発電中さん2022/05/11(水) 21:24:00.73ID:ZpdqmMA8
楽しめてればなんでも良いぞ
理解を深めるためには、オシロだったりシミュレータだったり色々有った方が良いぞ
03282892022/05/11(水) 21:34:23.79ID:8QKWWHw0
>>318-321
すいません、>>289でクロック線に470μFのバイパスコンデンサを入れるのが効果がなかったと書きましたが
今試すとアルミホイル巻くのと同じくらい無茶苦茶効果ありました。
どうも前はコンデンサを繋ぐ位置を間違えていたみたい。

パスコンの容量は

470μF~15pF→効果あり、ノイズで全く誤動作せず。
10pF以下→効果なし

でした。ただしパスコンがない時はラズパイから信号を送る時

GPIO.output(14, GPIO.HIGH) #クロック信号オン
GPIO.output(14, GPIO.LOW) #クロック信号オフ

とPythonで書いて問題無くクロック信号を入力できましたが、パスコンを付けると信号が短すぎて、
ノイズと同じように信号がパスコンに吸収されてしまうようで

GPIO.output(14, GPIO.HIGH) #クロック信号オン
time.sleep(0.1) #クロック信号を0.1秒流し続ける
GPIO.output(14, GPIO.LOW) #クロック信号オフ

のように間にsleep入れる必要が出てきました。
0329774ワット発電中さん2022/05/11(水) 21:44:34.49ID:933Q/jFK
>>328
個人的には>>314氏の2が簡単で効果的と思うけど試した?
0330774ワット発電中さん2022/05/11(水) 22:40:27.14ID:Jf++UAEG
プルダウンは入ってるぞ
これ以上の対策望むなら回路図出せ
0331774ワット発電中さん2022/05/11(水) 23:40:38.34ID:wm9rPgZJ
>>328
>クロック線に470μFのバイパスコンデンサを入れ
ちょっと異常だね。ドライバが壊れそう。
そんなに立ち上がり遅くするならシュミットトリガで受けないと
かえってノイズに弱くなるよ。
0332774ワット発電中さん2022/05/12(木) 01:39:25.22ID:088E05YF
なんか 人の言うこと無視なんだな。
インピーダンスが高いことが気にならないのかな。
あるいは、こないだの定電流のお兄さんのように、
GNDが接続されていないとか。
0333774ワット発電中さん2022/05/12(木) 02:58:20.00ID:1Dy2T9Wr
マイコンの用語に「ハイインピーダンス」ってあるよね
インピーダンスが高いのはあたりまえなので
動けばいいのではないでしょうか
と思う
0334774ワット発電中さん2022/05/12(木) 03:24:15.32ID:SJpGif78
>>333
クロック線がハイインピーダンスの時ってある?
前段の出力はハイインピーダンスのときは無いんだよ。
常にゼロインピーダンス。
だからそれにつながるクロックの線も、ハイインピーダンスではなく、
ゼロインピーダンス。

あと、動けばいいと言う 結果オーライ は、ダメ。

ヒューズのよく飛ぶ家電品の対策として、
100本入りのヒューズを用意するようなもの。

少し覚えれば、何の問題もなく回路が扱えるのに、
動いたからいいじゃんと言う子供みたいなこと言ってはダメです。
0335774ワット発電中さん2022/05/12(木) 05:51:02.43ID:1Dy2T9Wr
なんというか
「マイコンの端子を入力に使うときはハイインピーダンスに設定します」
ということだから、端子をハイインピーダンスにしているのに
ハイインピーダンスなのがいけない、といわれても困るかもなあ
と思っても説明は面倒くさい
本当はどう思ってシールドしたり、Cをつけたりしてるのかわからないから
なおいっそう、どこから
0336774ワット発電中さん2022/05/12(木) 09:28:16.45ID:awDQ+vtt
>ヒューズのよく飛ぶ家電品の対策として、
>100本入りのヒューズを用意するようなもの。

ぼくたちはなにもかもまちがっていきてきたのかもしれないね

イザナミ「ちくしょー! 毎日千人ぬっ□しちゃる!!(^p^#)」
 ↓
イザナギ「なら毎日1.5k人 産んぢゃるわぃ!(#^p^)」
 ↓
人 口 爆 発 \(^o^)/
0337774ワット発電中さん2022/05/12(木) 11:16:23.08ID:Hom679JS
12vの乾電池っていうものがあったんだな 23aとかいうやつ
9vはラジコンで知ってたけど
これって特に何に使うの? カー用品を電池で動かすとか?
0338774ワット発電中さん2022/05/12(木) 12:10:14.35ID:YG0lrzh4
テスターを開けたら入っていたことがある
0339さて、ここで問題です。↑がテスターを開けたら2022/05/12(木) 13:49:07.48ID:awDQ+vtt
何が入っていたのでしょうか?
・回路図
・ブレッドボード
・470μFのバイパスコンデンサ
・fuse百本
・イザナミ
・(^p^)
・9Vの乾電池
・555爺
0340774ワット発電中さん2022/05/12(木) 13:56:54.71ID:KLwO7trB
AliでLEDライトかなんかのリモコンが12Vの電池使う仕様で
そんな電池手に入れづらいだろって思って買わなかったことがあったな
アマゾンで検索したら普通に12Vの電池売ってたけどw
0341774ワット発電中さん2022/05/12(木) 14:04:46.99ID:EwJ3GbpN
電器屋探してもなくて、何故かコメリに置いてあった
0342774ワット発電中さん2022/05/12(木) 14:27:42.55ID:AQ3L5iBn
近くにパワーがある人がうらやましい。
0343774ワット発電中さん2022/05/12(木) 14:57:40.80ID:K7Q/mxl/
そういえば、単5電池ってどこいっちゃったんだ?
単4電池よりも先に普及していたと思うんだけど
0344774ワット発電中さん2022/05/12(木) 16:04:32.37ID:GU2A7Spu
>>335
入力特性を Hi-Z にすることと
実際に何も繋がっていなくて Hi-Z になってるのを
混同してる人も居るってことか
0345774ワット発電中さん2022/05/12(木) 16:07:21.28ID:GU2A7Spu
>>343
006Pも最近観ないな
0346774ワット発電中さん2022/05/12(木) 16:14:47.99ID:WYXnmpNU
006Pを009Pと言ってしまうま
0347774ワット発電中さん2022/05/12(木) 17:03:24.85ID:JH6c4QS1
>>345
百斤にある
0348774ワット発電中さん2022/05/12(木) 17:21:28.77ID:Dk698Qdu
006Pってチーズですかね?
0349774ワット発電中さん2022/05/12(木) 17:28:51.84ID:3gxFrExI
六個入りだからね。
0350774ワット発電中さん2022/05/12(木) 17:47:58.71ID:MGwuJ3QG
ローソンのロッピーとも関係ある?
0351774ワット発電中さん2022/05/12(木) 19:31:17.06ID:YNdYhj2h
フロッピーディスクは不織布含めた部品が6点だったのでフロッピーと名付けられた
0352774ワット発電中さん2022/05/12(木) 20:43:11.75ID:3VFB9zBp
>>351
面白くない
0353774ワット発電中さん2022/05/13(金) 00:17:17.89ID:OnekJqL0
フロッピーディスクと言えばドクター中松だな
0354774ワット発電中さん2022/05/13(金) 00:37:42.49ID:9Vo3qZ9p
あと麻生だな
0355774ワット発電中さん2022/05/13(金) 00:39:06.45ID:KHBQ3gEP
リコーマイティチャーは?
0356774ワット発電中さん2022/05/13(金) 03:14:52.52ID:Q75fiE0E
4630万円誤振り込みはフロッピーのせいではないよ
0357774ワット発電中さん2022/05/13(金) 06:48:24.93ID:0vbecgQO
>>355
中古の貰い物持ってた
学習目的じゃなくて分解でもして遊べば、と
0358774ワット発電中さん2022/05/13(金) 15:02:25.18ID:W0ROUkfT
倒立振子をなるだけ安く作る方法考えてください
0359774ワット発電中さん2022/05/13(金) 15:30:43.81ID:S+6vt7U/
>>358
ひたすら練習する
0360774ワット発電中さん2022/05/13(金) 15:32:08.41ID:W0ROUkfT
>>359
それは倒立紳士
0361774ワット発電中さん2022/05/13(金) 17:09:39.86ID:PbAFKu4d
>>360
傘を手のひらの上に、、、は置いといて、コマが含まれるならコマ、ダメなら棒に磁石付けて起きあがらせるとか

1軸回転するやつならフィードフォワード制御すればセンサーが無くせるかな?静止は出来ないと思うが
0362774ワット発電中さん2022/05/13(金) 17:16:33.51ID:W0ROUkfT
>>棒に磁石付けて起きあがらせる

あ、それおもろい
フィールドが鉄板なら行けるね
0363774ワット発電中さん2022/05/13(金) 17:21:09.67ID:lunCMtiu
振り子式メトロノームを改造する
電子メトロノームを手土産に音楽教室を巡れば、狂いの出たのが大量に入手できる可能性
0364774ワット発電中さん2022/05/13(金) 17:25:55.32ID:W0ROUkfT
>>363
そんな手がw

音楽教室って成人男性お断りのところが多いって、youtubeの誰かが言ってたけど
なんでだろうね?、ナンパ目的が多いんかな?
0365774ワット発電中さん2022/05/13(金) 18:53:47.65ID:ruvgWMWC
Aliで「倒立振子」で検索すると大人のおもちゃしか出てこないのはなぜなんだぜ?
0366774ワット発電中さん2022/05/13(金) 19:42:32.19ID:FKNCNbf4
>>364
知り合いのおっさんはJK狙いでマックのバイトやってたよ。
しかもモテてたようだ。
0367杜の都の櫻通り2022/05/13(金) 21:54:01.66ID:TS7UyCJl
合成抵抗を求める問題なんですが、各抵抗の逆数の和で出したのですが、どうもきれいな値になりません。誰かといて教えて欲しいです。
初心者質問スレ 153 YouTube動画>13本 ->画像>53枚
0368774ワット発電中さん2022/05/13(金) 21:57:36.93ID:5QVOLjdf
きれいな値である必要はあるのかね
0369杜の都の櫻通り2022/05/13(金) 22:00:57.63ID:TS7UyCJl
各抵抗の逆数の和の逆数で計算したところ、45/29になりましたが、これでよいのでしょうか?
0370774ワット発電中さん2022/05/13(金) 22:06:44.11ID:5QVOLjdf
いいんじゃね?
きれいな値になるはずだというのであればR1,R2,R3の値を再確認するぐらいかな
0371杜の都の櫻通り2022/05/13(金) 22:09:03.94ID:TS7UyCJl
了解です。ありがとうございます!
0372杜の都の櫻通り2022/05/13(金) 22:48:40.54ID:TS7UyCJl
電気回路の質問です。
問9 合成抵抗を求める必要があるのか…
問10 分圧の法則をどのように使えばよいか…
初心者質問スレ 153 YouTube動画>13本 ->画像>53枚
教えて欲しいです。
0373774ワット発電中さん2022/05/13(金) 23:10:35.35ID:PdhLxCZ1
LTC1144でプラスとマイナスの電源を作ろうと思ってます
この説明書の標準的応用例で、5番のVoutの所に10ufの電解コンデンサがついてますが、これは何のためにあるのでしょうか?
また、電源用ノイズフィルターの電解コンデンサやセラミックコンデンサはLTC1144の前後どっちにつけたらいいのでしょうか?
https://akizukidenshi.com/download/ds/analog/ltc1144.pdf
0374774ワット発電中さん2022/05/13(金) 23:42:32.41ID:CvLsE0gH
>>372
両方とも分流の法則そのままでしょ
抵抗値に反比例して分流されるってやつ

問9は電流計側と抵抗側の電流比が15:5
問10はR:R0=1/n:(1-1/n)をRについて解くだけ
0375774ワット発電中さん2022/05/13(金) 23:57:13.62ID:8qgK4D6Z
>>372
合成抵抗を求めて、電圧を出して、内部抵抗値で割っても良いけどね
下の方も、電圧を求めてそれから計算しても良いけどね
Ωの法則をいじっているだけだからどうやっても出来るよ
0376774ワット発電中さん2022/05/14(土) 00:01:07.44ID:9ds/yl39
>>373
仕様書の6ページ辺りを読めば、5番ピンのコンデンサの役割はわかるのでは?
0377杜の都の櫻通り2022/05/14(土) 00:06:07.12ID:uTIgiGxG
374,375
教えてくださり、ありがとうございます。
0378774ワット発電中さん2022/05/14(土) 00:14:21.27ID:yj5NYZd8
>>373
>5番のVoutの所に10ufの電解コンデンサ
これが無いと負圧の出力が矩形になってしまう(;´・ω・)

>電源用ノイズフィルターの電解コンデンサやセラミックコンデンサはLTC1144の前後どっちに
本格的なノイズ対策をするならコンデンサ単独ではなく
コンデンサにインダクタ(もしくは抵抗)を組み合わせたLC(RC)フィルターを入力と出力の両方に欲しい

とろこでLTC1144CN8(秋月)690円って高くね?
TJ7660(少し対応電圧低い)60円じゃ駄目なん?\(^o^)/
03793732022/05/14(土) 00:35:43.09ID:TSxSUOCC
説明ありがとうございます
ちゃんとやるなら両方にフィルターが必要なんですね
ググってたらLTC1144が出てきて他にどんなのがあるのかまだわかってない状態です
TJ7660も調べてみます
0380774ワット発電中さん2022/05/14(土) 00:51:31.45ID:yj5NYZd8
>>379
ノイズ対策にフィルターが必要か?(どの程度のフィルターか)は目的や求める性能で変わると思う
逆に両方にフィルターが無くても問題なく機能する事も多いと思う
消費電流が少ないなら コスト的には入力側>コンデンサのみ 出力側RCフィルターが良いかも
0381774ワット発電中さん2022/05/14(土) 05:48:40.49ID:bJObF9ji
>>365
どんな検索履歴w

inverted pendulumで検索すると普通に出る

…ダウジング用ペンデュラム(水晶とかの振り子)の方が沢山出るけど…
0382杜の都の櫻通り2022/05/14(土) 08:19:48.46ID:uTIgiGxG
電気回路の質問です。
R1=40Ω R2=60Ω R3=120Ω R4=120Ω V=2V のときの端子ab間の電圧V0を求めたいです。
a点の電位とb点がどのように求められるのか教えて欲しいです。
初心者質問スレ 153 YouTube動画>13本 ->画像>53枚
0383杜の都の櫻通り2022/05/14(土) 08:20:42.84ID:uTIgiGxG
訂正→R4=80Ω
0384774ワット発電中さん2022/05/14(土) 08:53:17.03ID:u9cTVEwQ
>>382
R1とR2 をA
R3とR4をB
と考えると AとBにかかる電圧はいくつになるか考える。
あとはA、Bそれぞれの中で2つの抵抗の間の電圧がどうなるか考える
0385774ワット発電中さん2022/05/14(土) 08:54:31.62ID:9ds/yl39
>>382
自分で考えよう、直列抵抗の両端の電圧がわかっているので各々の直列抵抗の電流がわかる
そうすればもうわかるよね、教えて君はいつまでたっても進歩しないよ
前回の質問の答えでやり方はわかったはずだ
>>380
スイッチトキャパシタ型のコンバーターだよ、一般的なノイズ対策じゃないよ
0386杜の都の櫻通り2022/05/14(土) 10:46:48.33ID:o4lC/GED
⏭384.385
ありがとうございます!
教えてもらったことをもとに自分で考えてみます!
0387774ワット発電中さん2022/05/14(土) 11:15:20.90ID:QIUcvBRr
>>378
顔文字、やめてください。
0388774ワット発電中さん2022/05/14(土) 12:44:14.69ID:AVOLe0tV
顔文字は自分が書いたっていう印が欲しいんだろ
顔文字はツイッターのアイコンだと思って諦めろw
0389774ワット発電中さん2022/05/14(土) 12:45:11.38ID:AVOLe0tV
それか、みんな顔文字を書いて誰が誰だか分からなくするっていう作戦もあるw
0390774ワット発電中さん2022/05/14(土) 12:51:36.75ID:mBAeZdpC
>>378=>>387
なのではないかと思う今日この頃
0391774ワット発電中さん2022/05/14(土) 13:29:30.47ID:OlcgA1G8
延々ここで宿題やってんじゃーねよ
0392774ワット発電中さん2022/05/14(土) 14:47:05.23ID:a0VM8kdG
>>382
ヒントだけ書いておくので、計算式など あとは自分で考えてください。
初心者質問スレ 153 YouTube動画>13本 ->画像>53枚
0393杜の都の櫻通り2022/05/14(土) 15:23:36.76ID:o4lC/GED
⏭384
ありがとうございます!
0394774ワット発電中さん2022/05/14(土) 20:16:51.41ID:QIUcvBRr
>>390
そ、そんなこと、ない、ない。
変なこと言わないで。
0395774ワット発電中さん2022/05/14(土) 20:34:29.66ID:mBAeZdpC
>>394
でもさ、>>378の顔文字の告知に一番貢献してるのは>>387だよね
ピンポイントでその告知のためだけにレスしてるものね
物事は、どういう動機でそれをするのか、って考えると真実が見えてくる
0396774ワット発電中さん2022/05/14(土) 23:21:52.04ID:csu334in
>>189
ちょうど手元にあった74HC4066で切り替えできるようにしたら無事にRTCを共有することができました
ありがとうございます
0397774ワット発電中さん2022/05/15(日) 07:46:24.21ID:eXre3XfW
>>395
少し頭のオカシイ人(>>387のことだけど)の動機を考えても意味ないと思うぜ。
なにしろオカシイんだからな。
>>378のは多分、たとえればスタンプラリーのスタンプ、あるいは御朱印帳の御朱印だぜ。
勝手にどうぞ、で他人が気にする必用は無いな。
0398774ワット発電中さん2022/05/15(日) 10:25:42.91ID:n/13Z6p3
>>397
そうは思わない
他人にどう思われてるか>>387に教えてる、気づいてなさそうだから
それより、「意味ない」言うくせにそれを文字に起こしてレスしてる君の方が自己矛盾に満ちてる
その行動は意味あるの?
0399774ワット発電中さん2022/05/15(日) 11:44:17.91ID:o4ie/FMf
顔文字より、口の悪いのやめろ、と言ってみる
0400774ワット発電中さん2022/05/15(日) 12:06:09.39ID:+DOOWhWO
なぜ顔文字をNG登録しないのかといつも思う
わざとやってんだろうし言っても聞かないんだから
淡々粛々と黙って視界から消せばいいのに
0401774ワット発電中さん2022/05/15(日) 12:09:26.11ID:o4ie/FMf
そういう問題じゃなくて、価値観を押し付けてるところがアホ
0402774ワット発電中さん2022/05/15(日) 14:55:56.17ID:cDoTyx5o
>>387
b=(o_o);
これはfuck youじゃ無くてソースコードの一部なんだからね!
0403774ワット発電中さん2022/05/15(日) 17:21:12.16ID:06o+HulB
顔が悪い
口が悪い
頭が悪い
0404774ワット発電中さん2022/05/15(日) 18:06:58.56ID:o4ie/FMf
態度が悪くなければよし
0405774ワット発電中さん2022/05/15(日) 19:06:44.29ID:ht5YV3PZ
身体が良ければ…

いや、健康は大切だよ?
0406774ワット発電中さん2022/05/15(日) 19:38:58.57ID:XdGsXstv
人間、外見じゃない。中身だよ。
ちゃんと人間ドッグ受けろよ。
0407774ワット発電中さん2022/05/15(日) 19:39:47.51ID:XdGsXstv
ドッグじゃなくてドックか?
ホットック?
0408774ワット発電中さん2022/05/15(日) 19:40:43.87ID:HRXz7+qB
自己放電の少ないEDLCってないんですかね?
トーキンのアプリケーションノートを読むと無負荷でも持って1ヶ月くらいに見えます
メモリバックアップで1年くらい保持できるのがあれば・・・
0409774ワット発電中さん2022/05/15(日) 19:56:16.64ID:ht5YV3PZ
ソーラーパネル付けちゃえ
0410774ワット発電中さん2022/05/16(月) 11:00:19.46ID:Pdajf8L8
ledランプで点滅 と 点灯の品があります
それぞれ20mAと表記があります
このばあい点滅のほうが電池が持続するんでしょうか
0411774ワット発電中さん2022/05/16(月) 11:04:39.13ID:Af//YdJS
>>410
0412774ワット発電中さん2022/05/16(月) 11:22:16.43ID:Pdajf8L8
ありがとうございます

amonのクルマ用LED ライト ですね
0413774ワット発電中さん2022/05/16(月) 22:12:43.90ID:bFiPqu9V
家のコンセントのアースについて質問です

昔コンセントの接地側とアース間の抵抗を測ったら0Ωだったのですが
アースと接地の間にある土の抵抗が0Ωと言う事はないだろうから、業者が手抜きして接地とアースをコンセントの裏で
直結しているのだろうと思っていました。

しかし


@YouTube



この動画を見ると土をリード線と考えたら土の断面積=地球 なので接地とアースの抵抗は0Ωになる言われています。

しかし、試しに家の植木鉢の土にテスターのプローブを挿したところたかだか20cmほどの距離で抵抗は無限大でした。
なので土の中の抵抗が0Ωだというのは信じがたいです。

土の抵抗が0Ωなどということは実際あるのでしょうか?
また、皆さんの家のコンセントで接地側とアース間の抵抗も0Ωになっているのでしょうか?
0414774ワット発電中さん2022/05/16(月) 23:21:24.36ID:fVNbpRjf
専用のメガで測れよ
0415774ワット発電中さん2022/05/16(月) 23:54:26.24ID:ScuZzSRJ
>>413
ゼロオームという事はないですよ。

地球自体は断面積がほぼ∞になるので抵抗は小さくなりますが、
接地抵抗といって、埋設電極が
地球に対してあまりに小さすぎることに
起因する抵抗が避けられないですん。

どうやって接地抵抗を小さくするかというのは地質によってはなかなか難しい問題で、
掘れるところなら改質剤添加して疑似的な電極を大きくする様な工夫も使えますが
岩盤質で掘れなかったりすると使えませんし、建築物の躯体・鉄骨を電極としても流用したり
するそうですン。

 接地抵抗の許容される大きさは 用途や環境(漏電遮断器の性能)に応じて色々あって
B種接地とかD種接地とか いろいろありまするが、プロが接地する工事でも
100Ω以下とか 10Ω以下とかそういう基準の世界だったとおもいまするぞぃ(^p^)

>直前レス
メガは絶縁抵抗用途。接地抵抗の測定には専用の測定器があるんよ。
0416774ワット発電中さん2022/05/17(火) 00:13:04.28ID:Z6ECu9xL
テスターみたいな直流で計ろうとすると、抵抗値がどんどん変化するので計れません
0417774ワット発電中さん2022/05/17(火) 00:39:26.09ID:u9ZzwozM
YouTubeのそういう動画は、大した知識のないやつが堂々と
知ったかで語ってるものが多い
どちらかと言うと大した知識のないやつほどそういう動画を作りたがる
0418774ワット発電中さん2022/05/17(火) 00:45:01.96ID:UhpIJnEs
>>416
端的で、分かりやすい説明。
0419774ワット発電中さん2022/05/17(火) 03:51:50.06ID:X4FWMVJV
大電流流すと地面熱くなるのかな
もぐら退治に使えないかな
0420774ワット発電中さん2022/05/17(火) 05:23:12.23ID:z+8sOL1f
>>419
雷ではそれなりに死んでるんだろうな
0421774ワット発電中さん2022/05/17(火) 05:29:18.11ID:hgYxjJG2
>>419
0オームにいくら流しても熱くなりません
0422774ワット発電中さん2022/05/17(火) 06:09:00.24ID:X4FWMVJV
小学生のときの実験
電線の節約にために片側をアースして大地を利用と読んで
トランジスタラジオのスピーカーから線を外して引き出して、その片側を接地として実験
5メートルほどだったけど音聴こえなかった
片側につき5寸釘2本では少なかったかな
0423774ワット発電中さん2022/05/17(火) 06:13:17.63ID:X4FWMVJV
またあるとき読んだ、通信線の断線事故を
垂れて地絡したところまでの電線の抵抗値を
組試験器で測定しておよその位置を調べたと
そんなうまく地絡するのかなと
0424774ワット発電中さん2022/05/17(火) 06:13:33.08ID:hgYxjJG2
>>422
低レベルのアースでも1mはあるような棒を打ち込むよ
0425774ワット発電中さん2022/05/17(火) 06:19:53.13ID:X4FWMVJV
>>424
じゃあ数メートルなら線で繋いだ方が合理的だったね
0426774ワット発電中さん2022/05/17(火) 06:33:28.75ID:hgYxjJG2
>>425
ですね~
これくらいだと100オームレベルだから、スピーカーはむつかしいかも
0427774ワット発電中さん2022/05/17(火) 15:31:32.99ID:UhpIJnEs
>>424
洗濯機でビリビリ来るのは、
抵抗が大きいからビリビリで済んでいるのですか?
0428774ワット発電中さん2022/05/17(火) 15:47:49.12ID:hgYxjJG2
>>427
一概には言えないかど、おおむねそういうこと

体を通る経路の抵抗が大きいということ
0429774ワット発電中さん2022/05/17(火) 17:13:45.53ID:X4FWMVJV
不思議なのは電気風呂はなぜビリビリするんですか
水槽はアースでいう大地と同じような意味合いだと思うのだけど
0430774ワット発電中さん2022/05/17(火) 17:15:02.03ID:hgYxjJG2
>>429
地面とつながってないから
0431774ワット発電中さん2022/05/17(火) 17:24:27.69ID:duqGlgnC
垂直に等電位面が形成されるように、水平方向に電位勾配が形成される方向で
でんあつがいんかされてるんじゃね? しらんけど;^p^)
0432774ワット発電中さん2022/05/17(火) 17:29:07.68ID:X4FWMVJV
>>431
ちんこ大きい
0433774ワット発電中さん2022/05/17(火) 17:30:35.10ID:X4FWMVJV
導体に包まれてるからと思ったけど
その上手な絵を見て考えてみれば
人体のほうが抵抗値低いからかもね
(わからないけど)
0434774ワット発電中さん2022/05/17(火) 17:31:13.49ID:X4FWMVJV
DCだと変なガス発生しそう
0435774ワット発電中さん2022/05/17(火) 17:36:28.70ID:hgYxjJG2
あのビリビリ感は交流だな
0436774ワット発電中さん2022/05/17(火) 19:19:59.09ID:xOcmvcAw
あ、そうか。電極の腐食を防ぐ意味でも交流でしょうなぁ・・・(^p^;

>>433
げに(電流を流しやすいからですな)。

類似の話としては、落雷頻発時には車の中が安心、と俗にいいまするが、
あれは 普通の自動車は車体が導体でファラデーケージを形成し、
人体を経ずに車体を経由して電流を流してくれるからですな。
人体の抵抗Ra と 車体の抵抗Rb とをかんがえると、
厳密に言えば Ra と Rb の両方に 電流は流れているのですが、
並列状態の電流の分配は 分流則にしたがいまするが、
分流則は、総抵抗和分の相手側抵抗値の大きさで電流を担いまする。
Rb≒0,Ra≒∞ と言えるような Ra >> Rb の状況下では
総電流の ほとんどの大部分が 低抵抗である車体側に流れてくれる、というわけですん
0437774ワット発電中さん2022/05/17(火) 21:55:54.01ID:1WiQqWAn
DCだと体内の電解質も影響受けるのかなあ?
0438774ワット発電中さん2022/05/18(水) 01:31:14.07ID:D75D59su
>>431
知らないなら、黙ってたら?
0439774ワット発電中さん2022/05/18(水) 01:40:27.83ID:QkKeQ5hJ
Cdsセルの暗抵抗値について質問させて下さい
型番が分からないCdsセル(Φ5mm)があるので、暗抵抗値を測ってみました
が、部屋を消灯し、更に暗所でテスター測定したところ、
最大30MΩまで値が上がっていきました
(それ以上暗くすると測定不能になりました)
Cdsセルで検索しても、暗抵抗の最大値は10MΩまでしかでてきません
https://akizukidenshi.com/catalog/g/gI-05863/
500kΩとの分圧回路でも抵抗値を計算してみましたが、
同条件で、同じく30MΩ以上まで上がりました

そこで、下記2点を質問させて下さい
1.暗抵抗値も個体差によって幅があるのでしょうか?
2.ひょっとして暗抵抗値50MΩとかのCdsセルが存在する?

どうぞ宜しくお願い致します
ちなみに照度計は所持していないので、暗抵抗くらいしか当てに出来ません...
(スマホの照度計アプリは参考にしてはいけないかと思い、今回は使用していません)
0440774ワット発電中さん2022/05/18(水) 03:07:02.95ID:D75D59su
>>439
>1.暗抵抗値も個体差によって幅があるのでしょうか?
もちろんです。
ただ、そんな「グラフで言うところのまっすぐに立ったような点」で
製品の企画を定義しません。
グラフが立った点なので、ちょっとの明暗で抵抗がガンガン変わってしまって
使いにくくてしょうがありません。
使用領域は、もっと寝たところで100kΩ〜1kΩを使用します。
だいたい、30Mオームとかの高抵抗のところは、回路で使いません。

>2.ひょっとして暗抵抗値50MΩとかのCdsセルが存在する?
1.と同じことです。ちょっとしたことでガンガン変わりますので、
そんなΩ数を競ったところで、大きな意味はありません。
0441774ワット発電中さん2022/05/18(水) 03:29:20.57ID:h+nBEmeU
野菜果物で言うところのヘタの部分!
捨てる所だし食べても美味しくなーい!
0442774ワット発電中さん2022/05/18(水) 03:56:41.14ID:mju/3yap
図鑑でみる細胞のイラストのミトコンドリアみたいで気持ち悪かった
学研の電子ブロックのを見たのが最初
街灯工事のおじさんに交換品を貰ったのが2回目
繋ぎかたも教えてもらって有頂天
でも100V使うのはまだ許してもらえず
電池で試してもうんともすんとも
0443774ワット発電中さん2022/05/18(水) 08:39:04.06ID:ZhPohr5z
子供の頃はエクレアを腹いっぱい食べるのが夢だった・・・

乾電池67個という夢 (^p^)どやぁ

>>438
わかってないなぁ〜
仮説を立てて検証し合うのが楽しいんじゃないか〜
( ^p^)σ)Д`)ツンツン
0444774ワット発電中さん2022/05/18(水) 08:46:34.28ID:GY2BfBQO
韓国とかで使われてるCタイプの電源プラグって売ってないの?
変換じゃなくて電線に付けるやつ
04454392022/05/18(水) 10:07:17.70ID:QkKeQ5hJ
>>440
「グラフの寝たところを使う」
この説明にとても納得がいきました
グラフの直線に惑わされていましたが、そういえば両対数グラフでしたね...
きちんと照度のある場所で、抵抗値を確認しようと思います
とても分かりやすいご説明をありがとうございました!
0446774ワット発電中さん2022/05/18(水) 10:38:19.89ID:O+26fdoj
メガオームとかになるとテスターのインピーダンスの関係で正確に測れないかもな。
0447774ワット発電中さん2022/05/18(水) 12:12:47.82ID:ZhPohr5z
最近のデジタルテスターだと10MΩぐらいありまするな、内部いんぴーダンスとかいう踊り?しらんけど;^p^)
0448774ワット発電中さん2022/05/18(水) 19:19:43.05ID:KdtTZC5n
いんぴーダンスとかいう踊りが得意な、アド美チャンなら知ってるよ。
得意技は、上下逆のヘッドスピンダンス?
0449774ワット発電中さん2022/05/18(水) 20:43:25.67ID:p8uwUz8x
…ハイインピと踊っちまったのさ…
0450774ワット発電中さん2022/05/18(水) 21:39:15.72ID:D75D59su
>>447
顔文字やめろ
0451774ワット発電中さん2022/05/19(木) 00:12:50.09ID:FbbN1FoC
イヤホンジャックをネジ端子台に変換するコネクタがありますけど、これのLRとGNDをオシロで繋いだら音声の電圧波形が見られるんですかね?
※アマゾンのURLが貼れなかった・・・
0452774ワット発電中さん2022/05/19(木) 00:21:29.05ID:0s1ixHqA
余裕
0453774ワット発電中さん2022/05/19(木) 00:48:12.31ID:yQ1GI7z3
URLの「dp/B07BMWRV8Z」この部分だけ貼っていただければ
こちらで検索致しますよ
0454774ワット発電中さん2022/05/19(木) 07:48:19.35ID:FbbN1FoC
>>453
dp/B07FVWPJSL
ありがとうございます。こちらになります。
0455774ワット発電中さん2022/05/19(木) 10:21:09.37ID:Qhp0bEHF
それのGND共通のやつなら3Pのターミナルアダプタにミニジャックを直接はんだ付けして作ったな
0456774ワット発電中さん2022/05/19(木) 10:22:09.06ID:Qhp0bEHF
ターミナルブロックの間違い
0457774ワット発電中さん2022/05/19(木) 11:20:13.66ID:Zv5G6I5s
スイッチングにFETを使う理由としてトランジスタより早いと言いますけど、FETのゲートドライブにトランジスタ使いますよね

???
0458774ワット発電中さん2022/05/19(木) 11:31:33.53ID:Q0lE6h2U
そういう場合は速度を気にしてるわけじゃなくて、トランジスタだとパワーロスが大きくなるからFETを選んでるのでは。
0459774ワット発電中さん2022/05/19(木) 11:34:46.48ID:QdDwDERI
a を %61 にしたら貼れる
0460774ワット発電中さん2022/05/19(木) 11:35:38.02ID:QdDwDERI
FETはトランジスタじゃないんですか
0461774ワット発電中さん2022/05/19(木) 12:00:28.74ID:ECKKQHDG
電界効果トランジスタだからそうだけども、ここでいうトランジスタはバイポーラトランジスタのこと
昼飯にそば(和そば)くいにいこうといわれて中華そばもそばですよねと言われた感じ
0462774ワット発電中さん2022/05/19(木) 12:42:26.75ID:OfLD7iJs
>>452 だがしかし音声の定義を考えるとなかなか難しい話やでぇ・・・
電気を介さない蓄音機の場合、マイクロホンのタイプ、振動板の変位、それは確かに音声の信号なのだろうけれど
スピーカーやイヤホンを前提とした場合、
駆動するコイルは加速度の信号なのだろうけれど
マイクロホンの信号は、振動板と追随して動くコイルが
磁場中を移動し、その変位(移動量)に応じ横断した磁束線の量に
応じた起電力で、その変位に応じた加速信号を得る為には
ただの増幅ではなく演算が必要になるのではないか? しらんけど;^p^)
どの段階の信号を音声の電圧波形と呼んでいいのか・・・
音圧の変化グラフを得るには気体の慣性を加味した変換をしないとあかんのではないかという気も;ぐぎぎ
0463774ワット発電中さん2022/05/19(木) 12:46:16.69ID:po/LOpMS
と、むやみに深く考える傾向はアレなのでしょう、か
0464774ワット発電中さん2022/05/19(木) 12:48:49.81ID:po/LOpMS
>>451を読んで一般には
波形がびよ~んって表示されるかなどうかな
ぐらいの目的に理解するでしょうね
0465774ワット発電中さん2022/05/19(木) 12:49:48.37ID:ELwmrrP0
>>463
ですね。きっとアレですね。
0466774ワット発電中さん2022/05/19(木) 14:20:16.47ID:OfLD7iJs
座右の銘は、石橋(のクラック)を(懸念して打音検査で)叩いて壊す(^p^)
0467774ワット発電中さん2022/05/19(木) 14:22:26.93ID:/0h0tTd/
>>462
>しらんけど;^p^)
そんなこと言うなら、はじめから書かなければいい。

顔文字やめろ
0468774ワット発電中さん2022/05/19(木) 14:51:41.90ID:OfLD7iJs
>かお文字やめろ

か■■■やめますか?
それともにんげんやめますか? ーーー警視庁

ジョジョー! 俺は人間をやめるぞー!! ーーディオ

>>451
NASAは加圧式ボールペンを開発! 一方露西亜は鉛筆を使った・・・ じゃないけれど、
百均でいあほんコード買ってきて、仏陀切ってリッツ線露出させて、
ハンダゴテの小手先に盛った半田に切断部を突っ込みつつ半田を補給(フラックス成分を補給)すれば
半田コートされてプローブの先で摘まめますぞよ。110円。
0469774ワット発電中さん2022/05/19(木) 16:44:02.98ID:jwCTiqjq
「知らんけど」は関西じゃ慣用句「だからな~、まあ、わかっててやってんだろうけど
0470774ワット発電中さん2022/05/19(木) 16:50:28.65ID:Ro0c5ReS
>>461
姫路駅「は?」
0471774ワット発電中さん2022/05/19(木) 19:33:07.13ID:uQeX8UyH
>>468
> NASAは加圧式ボールペンを開発! 一方露西亜は鉛筆を使った・・・ じゃないけれど、

浮遊する黒鉛を吸い込むことになるけどね
一見無駄と思われることでも研究することには意味はある
この違いがGAFAを生み出すことにつながっていると思う
まあ冷戦時代でもGDPとか隔世だったらしいが
今なんでもやってやると考えてるのは中国
北朝鮮も核やICBMを作り上げてしまった
0472774ワット発電中さん2022/05/19(木) 19:37:45.88ID:jwCTiqjq
鉛筆を使ったは、たんなるジョークだと、どっかで聞いたけど
日本だったらどうなるんかな、筆?
0473774ワット発電中さん2022/05/19(木) 20:16:04.18ID:FzBSupcK
芯折れて回路短絡したり
はさまってスイッチとか機器が動かせなくなったらどうするんだと思う

パソコンが貴重だった時代、パソコン室では鉛筆シャーペン使用禁止でした
ちなみに食品工場でも。
0474774ワット発電中さん2022/05/20(金) 06:29:56.02ID:YIaR+SJC
例えば12Vの電源を50%のPWMにして平滑した場合、理想的にはロス0で6Vが得られると思うのですが、降圧チョッパとはどのように違うのでしょうか
0475774ワット発電中さん2022/05/20(金) 06:38:47.18ID:9KLV/Xwi
平滑すれば50%でも1%でも無負荷なら12Vが出ます
0476774ワット発電中さん2022/05/20(金) 06:40:48.09ID:9KLV/Xwi
つまり制御あっての結果6Vが得られます
0477774ワット発電中さん2022/05/20(金) 08:03:03.63ID:UPEMz3dp
>>474
電圧は「PWMスイッチと電源の抵抗値」と「負荷抵抗」次第。
PWMのデューティ50%で電圧が半分になるのは、「PWMスイッチと電源の抵抗値」×2=「負荷抵抗」のとき。
そして、入力電流の平均値は、出力電流の平均値と同じ。電圧差×電流がそのまま損失になります。
降圧チョッパはコイルで電圧を変換しているので理想的にはロスはゼロになります。

PWMで平滑して電圧を変えると損失は小さい、って
「LEDの電流を抵抗だけで制限するより、抵抗+PWMで制限する方がロスが少ない」
と同じような思い込みですね。
コイルを使うか、動的にコンデンサの構成を切り替えるかしないとロス減らないよ。
0478774ワット発電中さん2022/05/20(金) 08:08:33.16ID:UPEMz3dp
>>477に補足。
>PWMのデューティ50%で電圧が半分になるのは、「PWMスイッチと電源の抵抗値」×2=「負荷抵抗」のとき。
これはPWMのデューティがOFFのときに、元電源と負荷側が切り離されていることが前提です。
0479774ワット発電中さん2022/05/20(金) 08:48:32.44ID:4Ks9oQhg
>>473
故障時の被害波及を視野に入れるというのは面白い観点ですね・・・
宇宙での加圧ボールペンの加圧タンク破裂懸念とかは大丈夫なのかしら;^p^)
しかしきちょぱそしつでは、皆さん何でメモされていたのでしょう…マジックとかボールペンとか万年筆?


>>477
素人質問よろしいでしょうか? ご説明を拝読してもよくわかりませんでしたもので。
現実空間では巻線(L) や蓄電器(C)の 透過直列抵抗での発熱がロスにはなるのでしょうけれど、
理想的に抵抗値を持たない巻線や蓄電器を用いた場合、
PWMを用いた降圧と、チョッパ回路を用いた降圧とでは
どちらの損失の方が大きくなるというハナシなのですか?
損失は、どちらもゼロなのですか?PWMのみがゼロなのですか?(・ω・`;
0480774ワット発電中さん2022/05/20(金) 09:16:34.02ID:UPEMz3dp
>>479
477(478)が前提にしているPWM回路だと、スイッチと電源の抵抗が理想的にゼロΩだとすると
負荷がゼロΩなら別として、デューティが小さくても電圧が下がりません。

インダクタを使わないPWMによる降圧は、スイッチ、電源その他どこかに直列抵抗が
存在することが前提で、損失が発生します。

インダクタを使ったBuck降圧回路は理想の状態なら損失ゼロになります。
実際にはゼロは無理ですが、インダクタを使わないPWMが抵抗で電圧降下をさせるのに比べると、
ずっと小さい損失で済みます。
0481774ワット発電中さん2022/05/20(金) 09:20:07.26ID:UPEMz3dp
あれ? なんか違うかな?
>理想的に抵抗値を持たない巻線や蓄電器を用いた場合、
>PWMを用いた降圧と、チョッパ回路を用いた降圧とでは

「PWMを用いた降圧」と「チョッパ回路を用いた降圧」の両方にコイルが入ってるのかな?
だとしたら、両者の回路構成がイメージできないです。
0482ごめん2022/05/20(金) 09:28:50.06ID:4Ks9oQhg
× 透過直列抵抗

○ 等価直列抵抗(ESR)



>減圧できません

どらえもん「はい、だみーろーど〜」
0483774ワット発電中さん2022/05/20(金) 09:36:01.42ID:xDFDY8iO
チョッパ型は電圧差分エネルギーを熱等として捨てる形式のことだから、そこから回答が間違ってる。
0484774ワット発電中さん2022/05/20(金) 09:37:40.08ID:NEBi9UKU
また無駄に電気を捨てるw
0485774ワット発電中さん2022/05/20(金) 09:40:32.91ID:4Ks9oQhg
>>483
まじで? いや、そんな説明、何処で読んだんよ・・・(^p^;  もういちど読み直してみようず
0486774ワット発電中さん2022/05/20(金) 09:54:00.95ID:UPEMz3dp
チョッパってインダクタを使うもの? 使わないもの?
字面だけなら「チョップするもの」だから、コイルの有無はわからない。そこをはっきりしないと。
0487774ワット発電中さん2022/05/20(金) 10:53:07.48ID:MDrC8Jre
今では、スイッチング電源と言う言葉が定着したが、昔はスイッチング電源の
ことをチョッパァー電源と言っていた。だからチョッパーア=スイッチングだね。

力道山の必殺技、空手チョップが、空手スイッチングだと、力が入らない。
スイッチング電源のことを、チョッピング電源と言うと、なんかしまらない。

ワンピースの・・・・・・・
0488なるほどねぇ。。。2022/05/20(金) 11:34:38.24ID:4Ks9oQhg
 12Vの電源を50%のパルス幅変調して平滑した場合
もしそれが12V1000Aの、12kW直流電源なら
50%PWMで電力を6kWにまで抑えられると思いますん。
 PWMの変調周波数は制御可能なのでフィルターで対処しやすいそうですが、
原理上 切り替え時に蓄電器へ向けて突入電流が生じるのは必然で
リプルが載るのは避けられないのではないでしょうか。しらんけど;^p^)

・降圧チョッパとの違い
チョッパはインダクタに磁場として貯める量で電力変換を行う回路のようですから
その制御にPWMをも用いるようです。電流の変化を嫌うインダクタの特性が活かされており
インダクタを伴わないタイプのPWMよりかは安定した出力を得られるみたいですん。

今回ググって拝読して面白かった資料
ocw.nagoya-u.jp/files/422/furuhashi06.pdf
power.nagaokaut.ac.jp/convenience/pdffiles/conv&control.pdf
0489774ワット発電中さん2022/05/20(金) 11:39:52.04ID:UJyv+5Dq
結局平滑するんだからコイルとかスイッチドキャパシタとか併用するでしょ
0490774ワット発電中さん2022/05/20(金) 12:00:02.42ID:wOxmIdQb
モーターのL分を使うって聞いた
0491774ワット発電中さん2022/05/20(金) 13:23:57.84ID:UMth30Rb
>>489
スイッチトキャパシタ分かって書いてる?
0492774ワット発電中さん2022/05/20(金) 13:29:51.42ID:UPEMz3dp
>>488

>12Vの電源を50%のパルス幅変調して平滑した場合
>もしそれが12V1000Aの、12kW直流電源なら
>50%PWMで電力を6kWにまで抑えられると思いますん。

50%でON/OFFして、負荷に電流が流れていても、十分直流になっているとみなせる程度に平滑してるんだよね。
電力が半分になるためには、抵抗負荷なら電圧は71%になっていることになるけど、それはOK?

消費電力は元電源では決まらない。負荷次第なんだけど、100%ONのときに、その負荷に12Vで1000A流れてたってことなんかな?

それと「思います」は「思い込み」であることもあるから、シミュレータで確認してみてはどうでしょう。
LEDでも「PWMは効率良い(コイルなしでも)」って主張している人も計算とかシミュレーションとかしてないよな。
0493774ワット発電中さん2022/05/20(金) 13:42:38.88ID:Lx758Lb3
なんで平滑前提?
パルス駆動で良いでしょ
0494774ワット発電中さん2022/05/20(金) 13:51:20.94ID:UPEMz3dp
>>493
そうそう、平滑していないなら、スイッチと電源の抵抗がゼロなら、すべての電力消費は負荷で発生します。
抵抗負荷で、100%のときに10Wなら、50%のときに5Wですね。
0495774ワット発電中さん2022/05/20(金) 14:32:54.66ID:4Ks9oQhg
>>492 ON状態での消費電力が定格電力を上回るような時は論外としても
平滑回路まるごと負荷側だとみなしてDuty比でオンオフ切り替えてりゃ
負荷次第で自ずとバランスする電位が定まるだけで、
本話題のような降圧目的なら電圧を注視して比率を切り替える制御になるのだろうと思います。
仰る数値が正しいのか知らんけど、仮にそれが正しいとして6V制御をしたいのなら
50%では過大で、もっと少ないON時間にすることになるのでせうなぁ。

>>493 それは >>474 が話題の発端だからです
0496けんざーん 検算〜(^p^2022/05/20(金) 14:49:03.43ID:4Ks9oQhg
Rが一定で仕事率が半分なら P=VxI=IxIxR=VxV/R
平滑後のVsは、連続印加時の電圧Vに対する大きさの比率は
0.5P=0.5 x V x V / R = Vs x Vs /R
V x V = 2 x Vs x Vs
Vs = sqrt(0.5 x V x V)=V/sqrt(2)=0.707V
なるほど抵抗負荷の場合Dutyれしお0.5では電圧0.71倍ですな(^p^;
0497774ワット発電中さん2022/05/20(金) 14:55:03.10ID:4Ks9oQhg
いや、これは十分に大きな平滑能力と、
定格最大の最大負荷の抵抗を繋げていた場合であって、
たとえば12V20W電源に 12MΩの抵抗負荷では元々が1μA、12mWしかないのだから
OFF時もキャパシタなどに貯められたパワーを使い続けて電圧は微動しかせず、ほぼ12Vのままでしょうな。
0.7倍も動くかどうかは平滑化機構次第ですな・・・
0498774ワット発電中さん2022/05/20(金) 15:11:33.09ID:NEBi9UKU
>>491
お前が知らないのはわかった
0499774ワット発電中さん2022/05/20(金) 16:23:44.65ID:1kwfch3P
5Aまで行けるDCジャックが無かったんですがACアダプターのプラグ部分をぶった切って中の線をハンダか何かで繋げても大丈夫でしょうか?
0500774ワット発電中さん2022/05/20(金) 16:28:23.03ID:T4kf6xmv
>>499
だいじょうび
0501774ワット発電中さん2022/05/20(金) 17:54:11.54ID:gXjrULf/
電気初心者です。
点滅する緑色のLEDがあります。
この光のon/offを、20cmくらい離れたところで
on/offを感じるトランジスタはありますでしょうか?
フォトトランジスタという物らしいですが、
緑色でも拾える物はありますでしょうか?
点滅スピードは、1秒間に1000回くらいです。
Cdsでもできるのでしょうか?
0502774ワット発電中さん2022/05/20(金) 18:00:15.89ID:9KLV/Xwi
レンズ
0503774ワット発電中さん2022/05/20(金) 18:30:57.24ID:BgbRunSz
>>501
フォトトランジスタにしてもcdsにしてもまわりとのコントラストの問題。
真っ暗な中に強い明滅ならOK。日光の下だと相当うまくやらないとムリ。
cdsは応答速度が遅いと言われてるが今やってみたら1usでOnOffする緑LEDに追従した。
0504774ワット発電中さん2022/05/20(金) 19:09:09.60ID:UPEMz3dp
変調かけて、同期検波か、受光アンプにバンドパスフィルタ付けないと、背景光にまけそう。
0505774ワット発電中さん2022/05/20(金) 19:15:59.13ID:gXjrULf/
>>501です。
みなさん、ありがとうございます。
>>501さん、実験までしていただいて、ありがとうございました。
500kHzで追従ですか。スゴいですね。テレビ画面に向けて打つ光線銃に入っていたので
それなりの応答とは思いましたが、まさかそこまで反応するとは。
フォトトランジスタはわからないですが、Cdsのほうがゲインは得られそうなので、
実験してみます。
仰るとおり、明暗の差ですので、紙を黒く塗って円筒形に巻いてCdsに巻き付ける
ようにします。
そう言えば、ハマホトのMCD521の代わりに、LED+Cdsでやったことがありました。
正弦波発振器の振幅制限だったかな。

みなさん、ありがとうございました。
0506774ワット発電中さん2022/05/20(金) 19:18:23.63ID:gXjrULf/
>>504
ありがとうございます。
TVリモコンの受光部に付いているフォトダイオードでも行けるかな?
0507774ワット発電中さん2022/05/20(金) 19:53:21.51ID:36znWbXf
カメラで撮影して画像解析
0508774ワット発電中さん2022/05/20(金) 22:05:01.26ID:T4kf6xmv
日光みたいに強い光があるところだとcdsのほうが飽和しなくていいという話を聞いたことがある
0509774ワット発電中さん2022/05/20(金) 22:42:52.84ID:gXjrULf/
自身がAGC動作...
0510774ワット発電中さん2022/05/21(土) 14:35:18.01ID:x7csFSbd
Aで始まりCで終わる困った名前はAGC
0511774ワット発電中さん2022/05/21(土) 15:32:25.81ID:zspmZGzM
君の瞳はAC100ボルト
0512774ワット発電中さん2022/05/21(土) 16:39:28.48ID:OzvwnojQ
コーヒーギフトは
0513774ワット発電中さん2022/05/21(土) 17:28:10.22ID:IGtbSSi3
UCC
0514774ワット発電中さん2022/05/21(土) 17:48:39.71ID:973dBlgh
行け、ピカチュウ!600Vだ!
0515774ワット発電中さん2022/05/22(日) 00:40:37.17ID:kaDDtWW3
フーリエ級数なんて電気電子設計で使いますか?
0516774ワット発電中さん2022/05/22(日) 05:08:57.87ID:YFwPn1Ym
>>515
サイン波の集合体で全ての波形を作れるって事を前提と言うか根幹にして
f得だ何だででの解析をしたりしてる訳だから
ふつーに使ってる、と言える
0517774ワット発電中さん2022/05/22(日) 05:32:05.17ID:fbUE1GQI
>>515
FFTオプションあるオシロ多いだろ…使う奴が多いって事だぞ

って、ここ初心者スレか
0518774ワット発電中さん2022/05/22(日) 06:04:52.42ID:zwEvCDnV
スレタイにはそう書いてあなる
0519774ワット発電中さん2022/05/22(日) 06:29:08.77ID:3N8Rmg8W
フーリエ級数(連続時間)と FFT (離散時間) も一応、別の
ものだぞ、と言っても、ここは初心者スレか。

上記の違いの結果、フーリエ級数の周波数はゼロから±無限大の
直線になるが、FFTのほうはナイキスト周波数で折り返して
円環状になる。
0520774ワット発電中さん2022/05/22(日) 06:38:58.94ID:3N8Rmg8W
フーリエ級数というより、フーリエ変換がらみの一連の知識だが、これを
知らなくても、それなりに電気屋の仕事はできるだろうが、知っていれば、
また別の種類の仕事ができる、ということかな。
0521774ワット発電中さん2022/05/22(日) 08:23:29.94ID:LzX37ml9
FFTって高速フーリエ変換の略、デジタル処理が高速に出来るようにアルゴリズムを考えたもの
0522774ワット発電中さん2022/05/22(日) 08:47:06.12ID:y2Bc7NKs
FFTはどちらかというと、測定や検証で使うとして
>>515の御題である「電気電子設計で使うか」というと
AD/DA、フィルタ、高周波分野だと、Z変換やラプラス変換を
使いまくるんじゃないかと思うけど、どうなんでしょう?
0523774ワット発電中さん2022/05/22(日) 11:02:37.01ID:68J4ZXOz
学生なんですけど、ケースとか作ってるタカチ電機工業から分厚いカタログが届きました
過去に何度かタカチのケースは買ったことあるんですけど、これって何か間違えで法人向けの奴が届いちゃったとかの可能性ありますか?
それともみんな届くのでしょうか?
0524774ワット発電中さん2022/05/22(日) 11:41:03.42ID:CVd1t8bt
普通ケースは小売店から買うからタカチに住所わからんだろ
タカチに直接問い合わせとかなんかしたんじゃね

モノタロウからはぶ厚いのが何冊もくるよ
0525774ワット発電中さん2022/05/22(日) 12:00:35.83ID:3E3dwk9X
費用対効果は悪いと思うのだが何故かカタログ送ってくる会社有るよね
気にせず貰っとけ
0526774ワット発電中さん2022/05/22(日) 12:05:26.70ID:68J4ZXOz
>>524
そういえば過去に一度問い合わせたことがあったので多分それですかね…
ありがとうございます。安心しました。
0527774ワット発電中さん2022/05/22(日) 12:23:53.39ID:CVd1t8bt
>>525
回路構成とか考えなら部品帳パラパラめくって考えをまとめるタイプの人がいる
webやpdfのほうが検索もできるし数万冊持ち歩けて便利だけどやっぱ紙で眺めたいということらしい
0528774ワット発電中さん2022/05/22(日) 12:27:34.45ID:MAxcrHuO
店頭に無い奴は、直販扱いでタカチから送るよ!ってなる
0529774ワット発電中さん2022/05/22(日) 13:10:03.13ID:3E3dwk9X
>>527
MISUMIは紙カタログ好きだったりするよ巻末充実してるし
ただ希望もしてないのにちょっと買ったからってカタログ送られてもね
0530774ワット発電中さん2022/05/22(日) 15:05:42.67ID:Cz6wgB6E
>>527
カタログ数万冊持ち歩くんか。
カタログを探すのが大変そうだ。
0531774ワット発電中さん2022/05/22(日) 16:40:48.75ID:B1BE5tYZ
カタログ作る時にある程度数作らないといけないから、どうしても余ることになる
どうせ捨てるなら、配っちゃえってことで、送ってもらえる、工作少年は狂喜乱舞
0532774ワット発電中さん2022/05/22(日) 17:06:40.10ID:92QTmWLH
並べておいたらインテリアにもなる
0533774ワット発電中さん2022/05/22(日) 18:24:53.02ID:B1BE5tYZ
とか何とか言ってたら家にも来た、タカチカタログ
荷物があるってんで時間空けてまってたらこれかい
こんなんポスト投函でいいやん、ぷんすか
0534774ワット発電中さん2022/05/22(日) 18:35:34.32ID:ilem6JBe
まあカタログ見て電話で注文する町工場のおっちゃんとかがターゲットだからな
0535774ワット発電中さん2022/05/22(日) 18:36:51.78ID:xlu9L5oN
でもPDFより紙面のほうがお宝グッズ発掘とか楽しい
0536774ワット発電中さん2022/05/22(日) 19:10:56.46ID:BvKLxXt2
家にも来たわ
重過ぎ
0537774ワット発電中さん2022/05/22(日) 20:11:35.48ID:6wKUzhh8
なんでタカチがお前らの住所知ってるんだよw
0538774ワット発電中さん2022/05/22(日) 20:31:58.59ID:B1BE5tYZ
>>537
通販で買ったからにきまってんじゃん
0539774ワット発電中さん2022/05/23(月) 00:32:56.29ID:9a1gqQgz
直接孝知てせ買ったの? そんなバカな
0540774ワット発電中さん2022/05/23(月) 00:42:34.51ID:ovMAF4JE
>>539
在庫切れのは、メーカー直送になったりするから、住所はタカチに渡るよ
0541774ワット発電中さん2022/05/23(月) 01:03:37.46ID:3V7g99nS
そうか…
タカチに…
知られてしまった…のか
0542774ワット発電中さん2022/05/23(月) 09:33:46.90ID:ODUUx8Dm
カタログなら便利だし資源ごみなら捨てるのもタダだしいいじゃん!
タキゲンのカタログとかも見ていて楽しいよね!!!

ピコーン) おっきいお友達なのに一度魔が差して(女児向け人形を)買ってしまったが最後、
延々とリカちゃん電話から電話がかかってきて、協賛会員登録して会費を支払い続けている間は
リカちゃんから電話がかかってこないいうオタカラビジネスモデルを思いついたゾ(^p^)

「もしもし? わたしリカちゃん(※但し精神科医の方)」
0543774ワット発電中さん2022/05/23(月) 16:56:22.96ID:9uz5Uzku
すみません、初歩的な質問なんですがなぜか電気代が高いな〜と思ってアース線と繋いでる間の電圧を測ってみたら50v出てたんですが…これって普通なんでしょうか?
計り方が違うのかそれとも地面に電気がダダ漏れなのでしょうか?
初心者質問スレ 153 YouTube動画>13本 ->画像>53枚
0544774ワット発電中さん2022/05/23(月) 17:00:14.06ID:ODUUx8Dm
何処にも接地してなけりゃ中間電位が出るのは不思議じゃないですよ。

ちなみに、エアコン以外の暖房類は電気をバカスカ食いまする。
0545774ワット発電中さん2022/05/23(月) 17:16:37.43ID:aF4G1IMb
ノイズフィルタでの電圧かな
0546774ワット発電中さん2022/05/23(月) 17:43:54.28ID:bcNCHuH/
>>543
テスターの
赤リードをどこにつないで、黒リードをどこにつないでいるのか
はっきり書かないと。
0547774ワット発電中さん2022/05/23(月) 17:59:46.95ID:tFBtrWAB
>>543
地面に駄々洩れしてたら漏電ブレーカー落ちるから、その心配はない
0548774ワット発電中さん2022/05/23(月) 18:05:50.55ID:tFBtrWAB
電気代高い犯人探しは、家の電気メーターの回り方みながら、コンセントを順に抜いてまわる
0549774ワット発電中さん2022/05/23(月) 18:07:13.13ID:KxH5c6Ru
今の電気メーターは回らないよ。
0550774ワット発電中さん2022/05/23(月) 18:22:31.86ID:OolWKWkT
エコワット買えばOK
0551774ワット発電中さん2022/05/23(月) 19:16:13.46ID:9uz5Uzku
>>546
赤リードをファーマンのSS-6B SS6B電源タップに繋げて黒リードをアースに繋げたらこうなりました
0552774ワット発電中さん2022/05/23(月) 19:16:29.45ID:9uz5Uzku
>>547
それが落ちないんですよ
0553774ワット発電中さん2022/05/23(月) 19:23:41.57ID:xbkUZ/09
>>551
>赤リードをファーマンのSS-6B SS6B電源タップに繋げて
その電源タップには端子部が1個しかないのかい?

>黒リードをアースに繋げたら
その「アース」とは具体的にはどこに繋がっている線なんだい?
0554774ワット発電中さん2022/05/23(月) 19:26:30.35ID:tFBtrWAB
>>552
落ちないんだから漏れてないんだよ
0555774ワット発電中さん2022/05/23(月) 19:53:36.56ID:O6ofVG1y
>>551
アースってもしかして機器側のプラグからペロッと出てる緑色の線のことかい?
だとしたら0Vや100Vにならなくても不思議ではない
0556774ワット発電中さん2022/05/23(月) 20:28:57.60ID:1OOpaxws
>>543
計測の理由からして釣りだよね。
0557774ワット発電中さん2022/05/23(月) 20:32:06.66ID:tFBtrWAB
ぐぎぎのひと?
0558774ワット発電中さん2022/05/23(月) 20:36:15.03ID:1OOpaxws
その電源タップの説明に
電磁波、電波が原因のノイズを削減するEMI/RFIノイズフィルター、サーキットブレーカーを搭載したFURMANならではのモデルです。
ってあるな。
無負荷時にホット-コールド間以外に電源のる要因ってあるのか? アース-コールド間はほぼ0Vだよね?
0559774ワット発電中さん2022/05/23(月) 20:36:37.28ID:XsgzbZmO
545で答え出てると思うんだけど…(^p^;
電源ラインにフィルタが入ってる機器の場合、
ノイズ減らす代償にリーク電流として0.1mAとか流すのは故障じゃないよ。
0560774ワット発電中さん2022/05/23(月) 20:43:42.47ID:1OOpaxws
ああ、タップをどこにも接続せずに、タップの電線を他の活線に近づけた場合、誘導でタップのホットとコールドに対地50V位の電圧は普通に出るかもな。
0561774ワット発電中さん2022/05/23(月) 20:49:01.84ID:1OOpaxws
>>559
ふむふむ、その時に50Vってどこ間?
0562774ワット発電中さん2022/05/23(月) 21:07:24.31ID:XsgzbZmO
フィルター付き三極電源インレットの機器を、FG線の接地しないで使うと、
交流電源なんだからキャパシタ経由でユンユン感電するよ!(^p^)まじで
0563774ワット発電中さん2022/05/23(月) 21:36:12.61ID:+e1vyR6W
>>562
ほうほう、だからピッタリ50Vってことね。
回路図分かりやすいね。
0564774ワット発電中さん2022/05/24(火) 00:22:17.26ID:bGtVCJmH
>>561
そのまま交流電流レンジに切り替えればどれだけ漏れてるか即分かるのになぜやらない
0565774ワット発電中さん2022/05/24(火) 00:24:50.04ID:4v+jDPhb
>>564
それは>>543に聞いてくれ
0566774ワット発電中さん2022/05/24(火) 04:24:25.57ID:3ioot72T
昔の機器だが、コンデンサの代わりにMΩクラスのソリッド抵抗を使ったものもあった
そのままシャシに落としているので、まじでビリビリする
アース接地が必要不可欠だと思ったw
ちなみに、80年代に社名変更した無線機オーディオ企業の製品
無線の世界ではおかしくなかったのかもしれないけど
0567774ワット発電中さん2022/05/24(火) 06:06:09.90ID:AnkIsfIR
トランスレス真空管アンプみたいな
コンセントの向き間違えるとシャーシに100Vかかる機械もあるしな
0568774ワット発電中さん2022/05/24(火) 06:25:40.58ID:3ioot72T
木の箱に入った昔のテレビは普通にシャシがホット(フローティング)だったよ
アンテナ(75Ω)のアースターミナルもコンデンサを介していた
0569774ワット発電中さん2022/05/24(火) 08:01:47.56ID:uh003Jhk
昔のラジオ製作記事で、電灯線アンテナとかありましたね
コンセントの端子を片側だけにする、当然L側に繋ぐわけだけど
そのコンセントの中に小さいコンデンサ、これだけが命綱
もしかして耳から感電事故とかあったのかな
屋外とかなら十分ありえる
今ならそんな記事は大炎上になるだろうな
0570774ワット発電中さん2022/05/24(火) 08:03:52.99ID:SvjAu/+g
>>569
なんでL側に繋ぐの?
0571774ワット発電中さん2022/05/24(火) 08:47:48.67ID:3ioot72T
N側じゃアンテナじゃなくてアースになっちゃう
0572774ワット発電中さん2022/05/24(火) 09:15:01.28ID:8RoW0tyy
地球アンテナ~
宇宙からのデムパを拾うのに使う
なおGNDが定まらない模様
0573(^p^)2022/05/24(火) 10:30:12.43ID:oDhqWYpS
ヤッホー知恵袋を紐解いたら
田中パイセンのご教示いわく
L線の時はキャパシタで分離せにゃあかんけど
N線を電灯線アンテナにする時は
キャパシタ不要で直結するものらしいヨ
知らんけど
0574774ワット発電中さん2022/05/24(火) 15:55:03.31ID:8WLZKHOn
挿し間違いは命懸け、ですね

とある現場で機器をオシロ使って動作確認
その機器は動作の都合上GND側がACラインの片側と接続
その担当者、壁のコンセントとオシロの電源線に±書いて極性識別
翌日、機器の電源入れるとオシロのプローブが一瞬にして発煙&断線
朝、掃除のおばちゃん、逆に挿しなおしてた
0575774ワット発電中さん2022/05/24(火) 18:01:26.50ID:3ioot72T
>>574
そういう機器って、3pプラグを使うものじゃないの?
もしくはN側の幅広のプラグとか
0576774ワット発電中さん2022/05/24(火) 18:04:44.42ID:J8jHtWt7
今のは知らんが昔のオシロのプラグって2Pだったな
あえて接地しない使い方ができるようにということだと思ってた
まぁうちの会社じゃそういう使い方はそもそも禁止されてるけど
0577774ワット発電中さん2022/05/24(火) 21:50:56.45ID:8nVsFjns
掃除のおばちゃん言いたいだけの小話によく付き合うよな
0578774ワット発電中さん2022/05/24(火) 21:53:02.82ID:0RIm23dS
この板何でも食いつくぞ
0579774ワット発電中さん2022/05/24(火) 22:01:35.04ID:3m9e1QgB
わたしゃビルのお掃除おばちゃん
0580774ワット発電中さん2022/05/25(水) 11:24:18.90ID:LxGrXIkw
>>574
オリジナルのマーフィーの法則って、
複数の可能性の内、もし致命的になる選択肢があるのならば
そちらが現実になる、でしたっけ? まんまですな(^p^;

>>575 いまどきLANのハブですら黒須ケーブルと直ケーブル間違えても
内部で切りかえくれるというのに・・・ぐぎぎ

>>578 誰ですか硼酸団子を置いたのは!? 吐いてる子もいるんですよ! オェー (^p^)←

>>579 おぜうさん 僕とお茶でもいかがですか??
0581774ワット発電中さん2022/05/25(水) 12:01:05.16ID:fK172V4z
>>579
父ちゃんのパンツはいてますか
0582774ワット発電中さん2022/05/25(水) 12:05:20.92ID:3f3pjnnv
秋葉原行ったらニッカド電池まだ売っていたけど需要あるん?
0583774ワット発電中さん2022/05/25(水) 12:06:22.75ID:/JvQnJqo
>>581
アソコの部分が透けてるの
0584774ワット発電中さん2022/05/25(水) 12:51:11.96ID:LxGrXIkw
あそこ = あんよ !?
0585774ワット発電中さん2022/05/25(水) 12:56:21.94ID:fK172V4z
>>583
それは履いてみたいやつでしょ!
0586774ワット発電中さん2022/05/25(水) 15:41:47.32ID:mnz2JCrB
>>582
大電流を取り出したい人がいるんだよ
0587774ワット発電中さん2022/05/25(水) 15:42:52.54ID:mnz2JCrB
>>547
家庭には漏電ブレーカーは殆ど無いな
0588774ワット発電中さん2022/05/25(水) 16:18:01.12ID:DtCgpaut
>>587
そうかぁ?
今どきの家だと大概ついてるような気がするが、そうでもない?

とりあえずアース取れない場合は設置義務有りだったような
0589774ワット発電中さん2022/05/25(水) 16:52:14.18ID:LxGrXIkw
ていうか、漏電遮断機の無い分電盤・配電盤を見たことあるんけ?
想像できんが 碍子引き配線な古民家にでも住んでいるのだろうか・・・
0590774ワット発電中さん2022/05/25(水) 18:17:31.51ID:qT8QmNAR
うちのボロアパートでも漏電遮断器くらい付いてるが
0591774ワット発電中さん2022/05/25(水) 19:45:51.32ID:PfO+t0L2
日曜日電工2種の試験だよ
漏電ブレーカー、出るよ。
0.5秒以内で遮断だよ。
高速型は0.1秒以内だよ。
0592774ワット発電中さん2022/05/26(木) 09:16:01.30ID:rTKNnaGR
あと少し最後の追い込みか、勉強がばれー G、(^p^)
0593774ワット発電中さん2022/05/26(木) 13:59:27.40ID:uffKjb+V
質問です。
全ての機器にアース接続してるとして、漏れ電流は漏電ブレーカーに反応しない程度で何wか流れてるわけですが、それを蓄積して確認する方法って何かありますか?
いろいろな種類のテスターなどは持ってます。
Amazonで売ってるYB5140DMやデータロガー機能があるDROKのJANコードしか載ってなかったのB098D4HN7K
とか
テスターも持ってるのですが….
0594774ワット発電中さん2022/05/26(木) 14:09:06.58ID:+JpDAuau
>>593
ブレーカーのところで行きと帰りの電流値の差を見る
0595774ワット発電中さん2022/05/26(木) 14:12:24.13ID:rTKNnaGR
電流はワットじゃなくてアンペアな。

ワッカ状のクランプメータ(鎖交させるタイプの電流計)で測ると、
正規の電流は クランプ内で往復して 相殺されて0のハズだけど、
地絡していった電流成分は片道しか通ってない筈だから
感度が良ければ検出できるんじゃね?しらんけど;^p^)
0596774ワット発電中さん2022/05/26(木) 16:25:47.88ID:jWPNg3VO
相殺されて0
waros
0597774ワット発電中さん2022/05/26(木) 16:40:13.83ID:rTKNnaGR
クランプメータは輪の中の電荷の移動量を測るのでしょうから
単相三線を一括で囲めば地絡電流量を測れるんじゃね? マジで(^p^;

漏電してなけりゃ、往路と復路の電流量は、逆向き逆相で 同じ量でそ。
下図では
オレンジ線路 成分が自己相殺しつつ
緑線路成分も 自己相殺。 どっか見落としてます?? (わらわれると不安になるタイプ
0598774ワット発電中さん2022/05/26(木) 17:10:29.60ID:6ObT9aUH
>>全ての機器にアース接続してるとして、
すべての機器に施すとは、工場かオーディオマニアなのかな
いまどきのテレビや炊飯器でも、アース端子はさすがについてないでしょ?
形だけの三極コンセントはあるかもしれないけど

>>漏れ電流は漏電ブレーカーに反応しない程度で
正常なら0ミリアンペアだろうから、ノイズなどで測定は難しそう
0599774ワット発電中さん2022/05/26(木) 17:33:05.34ID:uffKjb+V
>>594
ありがとうございます。
00.06Aの表記が出たり00.12Aだっりとにかく0Aではなかったです。どうなってんやろ…
>>598
アース繋げば同じ他のタップに繋げてるノイズが強い(例えば扇風機やACアダプタ)電子機器の寿命を延ばすことができるのでいちをやってます。
0600774ワット発電中さん2022/05/26(木) 18:10:39.29ID:rFhBHUhL
>しらんけど;^p^)
>(わらわれると不安になるタイプ

だったら、黙ってろ
0601774ワット発電中さん2022/05/26(木) 19:50:31.87ID:117ONeeU
>電子機器の寿命を延ばすことができる

どういう理屈?
0602774ワット発電中さん2022/05/26(木) 19:57:51.95ID:MHlnpH5p
ちなみに+100V相の緑色の電流を測りたいときは
クランプをこの様に挿む。という理解。

結局どこを笑われたのかわからんかった…(^p^;
0603774ワット発電中さん2022/05/26(木) 23:12:30.63ID:iiymKQRT
>>601
別に個人用の電柱作るようなオカルトオーディオオタではないっすけどEMFメーターで測るとアースつけた時とつけない時で電磁波の数値が全然違うよ
検証で近くに停電時のコンセントプラグ近づけたりすると勝手に光ったりした。
同じタップにラジオだったり電波時計だったり何かしら電磁波の影響は受けてたし
オーオタと同じレベルでバカにする根拠ある?
0604774ワット発電中さん2022/05/26(木) 23:34:41.21ID:a/c6CRGA
>>603
マイ電柱には理屈があるよ
費用に対して効果が恐ろしく小さいだけで
0605774ワット発電中さん2022/05/26(木) 23:38:36.20ID:117ONeeU
>>603
いやだからそれらの影響が他の製品の寿命にどう関係する訳?
電磁波が何にどうなって寿命を縮めるの?
0606774ワット発電中さん2022/05/26(木) 23:40:32.16ID:117ONeeU
あ、別に煽ってるとか食ってかかってる訳じゃないからね
純粋に意味不明なもんで
0607774ワット発電中さん2022/05/26(木) 23:46:21.29ID:iiymKQRT
>>605
例えばACアダプタ
アース繋いだ時と繋がない時で熱測ってみろ
0608774ワット発電中さん2022/05/27(金) 00:06:02.87ID:Iia+kvFx
>>607
熱?電磁波がどうこうじゃなかったの?
熱を測れって、アースを取ると電源の熱損失が変わるのかな?
因果関係をすっ飛ばすのはオカルトだよ。
0609774ワット発電中さん2022/05/27(金) 00:30:14.74ID:u5b1V0Cj
>>607
熱云々はともかくとして、ACアダプタのアースを取るって具体的にはどうやってるの?
0610774ワット発電中さん2022/05/27(金) 00:32:18.65ID:ryEIaoOe
>>607
そっ、その「電磁波」って言うヤツの周波数は?
0611774ワット発電中さん2022/05/27(金) 01:04:33.75ID:2Az1Srcz
こわいこわいこわいこわいこわいこわいこわいこわいこわいこわい
0612774ワット発電中さん2022/05/27(金) 01:52:48.16ID:j5atg4jR
めんどくさいから説明するよりやってみれば?
DCコンバーターや直流安定化電源みたいなもん繋いでるそれにさらにオシロを繋げばノイズの観測ができるよ
ノイズはタップに逆流するんだからそれを踏まえて逃せるんじゃないの?
0613774ワット発電中さん2022/05/27(金) 02:07:35.24ID:2Az1Srcz
こわいこわいこわいこわいこわいこわいこわいこわいこわいこわいこわいこわいこわいこわいこわいこわいこわいこわいこわいこわい
0614774ワット発電中さん2022/05/27(金) 02:29:45.93ID:I1/OUaIW
>>604
それを屁理屈という
屁みたいな効果のごとく
0615774ワット発電中さん2022/05/27(金) 04:08:29.14ID:yX/rEnhe
アース繋いだくらいでノイズが逃がせるとか行ってる時点で、でたらめ
そんなんで、逃がせるくらいなら、世の中の技術者は苦労してない
0616774ワット発電中さん2022/05/27(金) 04:45:31.14ID:yX/rEnhe
EMFメーターの数値変わるって、空間分布が変わっただけでしょ
アースに逃げたんなら0になんなきゃね
0617774ワット発電中さん2022/05/27(金) 08:01:21.58ID:VSGDLiwH
やり直しの勉強で過渡現象を学びたいと思ってるんだけど
独学に向いてるお薦めの本って何かありますかね?

微分方程式とかラプラス変換は別途本を買って勉強します
0618774ワット発電中さん2022/05/27(金) 08:34:01.15ID:YYKanqZ+
オーオタは6kV受電するって本当ですか?
0619774ワット発電中さん2022/05/27(金) 09:27:32.37ID:ryEIaoOe
>>617
これとか良さそう

過渡現象の基礎(第2版)
https://www.morikita.co.jp/books/mid/073552
0620774ワット発電中さん2022/05/27(金) 09:36:45.45ID:z161SdPV
500kV受電しなくちゃね
0621774ワット発電中さん2022/05/27(金) 09:37:37.95ID:VSGDLiwH
>>619
試し読み見てみたけど確かによさげ
ありがとう!
0622774ワット発電中さん2022/05/27(金) 10:11:10.22ID:oSFOfBPU
・・・他の需要家の負荷変動の影響を受ける、という意味においては
他の級の契約にすることは何の解決にもならないし
相当好都合な口実がないと契約自体結んでもらえんでそ。

電気的なノイズはフィルタで除去すればいいし
なんなら一旦DC化して、正弦波を合成するのもいい。

機械的に行きたいならモーターと発電機を直結して回転経由で発電すれば、
回転モーメントに均されるんじゃないかな。

もっと凝りたいなら 電気でボイラーを焚いて まいくろ汽力発電で自家発電すれば、
電圧変動はボイラー温度時点で均されて、同系統他需要家の電灯線由来の
ノイズは緩和するかもな、しらんけど;^p^)
0623774ワット発電中さん2022/05/27(金) 10:38:06.03ID:yX/rEnhe
今どきは、バッテリー駆動じゃないっけ?、すでに古い?
0624774ワット発電中さん2022/05/27(金) 12:25:46.83ID:n1uuBwFV
>>622
下二つの案が本当に良いと思うなら高圧受電も有りだよ
発電機の動作音が無いし
0625774ワット発電中さん2022/05/27(金) 12:30:38.06ID:uuMPEWQE
>>618
高圧・特別高圧電気取扱特別教育は受けていないので
600Vまで!
0626774ワット発電中さん2022/05/27(金) 12:37:12.09ID:jixUGf1a
専用電信柱と柱上トランスから100/200Vのオーディオ専用電源線を引き込んでもらうのです。
0627774ワット発電中さん2022/05/27(金) 12:56:55.24ID:x/9HQO76
ソープディスペンサを自作したいのですが、そのためのポンプモーターって売ってますか?

秋月やマルツで調べてみても見つかりません
0628774ワット発電中さん2022/05/27(金) 13:07:29.89ID:yX/rEnhe
>>627
アリエクで探せば、きっとあるw
0629774ワット発電中さん2022/05/27(金) 14:06:38.65ID:cNDdCGyI
>>627
Aliで買ったよ。作りかけて放置中やけどw
こんな感じのやつ
https://aliexpress.com/item/32959584165.html
https://aliexpress.com/item/1005001825387087.html
https://aliexpress.com/item/1005003241646032.html
0630774ワット発電中さん2022/05/27(金) 14:11:48.27ID:cNDdCGyI
あと既知だったらあれだけど、ダイソーで1000円で売ってるけどね
これをハックしてマイコンで排出量とかを制御してもいいかも
https://www.o-uccino.jp/article/posts/81890
0631774ワット発電中さん2022/05/27(金) 14:58:10.90ID:1kZDkAjC
>617
超古い本だが
エレクトロニクスエンジニアのためのラプラス変換
0632774ワット発電中さん2022/05/27(金) 15:14:56.77ID:c8cZdKZk
EMANの物理 ラプラス変換
https://eman-physics.net/math/differential13.html
RL直列回路の例
https://eman-physics.net/circuit/rl.html
0633774ワット発電中さん2022/05/27(金) 15:15:43.07ID:c8cZdKZk
物理のかぎしっぽ ラプラス変換 RC直列回路
http://hooktail.sub.jp/elemag/RCtransient/index.html
0634774ワット発電中さん2022/05/27(金) 15:17:25.88ID:c8cZdKZk
電気技術 解説講座 RL直列回路 過渡現象
https://jeea.or.jp/course/contents/01132/
電験

@YouTube


方形波

@YouTube


三角波

@YouTube

0635774ワット発電中さん2022/05/27(金) 15:31:16.64ID:EwEyyy8d
フーリエ変換とラプラス変換を合わせて、ナポレオン変換っていいますか?
0636774ワット発電中さん2022/05/27(金) 15:39:10.63ID:EwEyyy8d
アニメ世代にはむしろ、ラプラス変身!  とか
フーリエ変身とか言うだろうが?
0637774ワット発電中さん2022/05/27(金) 19:15:12.85ID:Iia+kvFx
>>626
今時太陽光発電(売電)のインバータ由来の汚い正弦波が流れ込んでくるからオーディオは夜間に限る?
オーオタが昼間は音に雑味が出るとか言わないのは何故だろうw
0638774ワット発電中さん2022/05/27(金) 19:24:22.51ID:ZsDP4RwM
柱上変圧器って鉄心のヒステリシスカーブに起因して正弦波が歪むよ?

スタンドアロンシステム用の安物インバータの矩形波の疑似正弦波ならともかく
パワーコンディショナーならむしろ(変圧器由来よりも)綺麗な波形が出てたりして・・・知らんけど;^p^)
0639774ワット発電中さん2022/05/27(金) 19:33:57.43ID:GEQOAEvy
>>629
やっっすくてイイネ
買おうかな

でも、できれば秋月みたいに、まとまってる所から買いたいな
0640774ワット発電中さん2022/05/27(金) 23:57:49.35ID:I1/OUaIW
柱上変圧器って油浸けでヒートシンクも付いてたりもするけど
容量のわりに小さく思えるのだけど
色々とギリギリに設計してるのか
0641774ワット発電中さん2022/05/28(土) 00:35:21.60ID:HgXcZlmD
モバイルバッテリーで12v 1000mAを出力方法はありませんか?ACアダプタの代わりに使いたいです。
0642774ワット発電中さん2022/05/28(土) 00:49:11.18ID:O3i3qPFd
急速充電(USB PD)対応のにすれば5V/9V/15V/20V出てくるから15Vから減圧するのがいいのかな
0643774ワット発電中さん2022/05/28(土) 01:05:41.07ID:62SvHmVJ
PD対応で20Wぐらいを要求すれば
必ず15V以上の電圧になる筈…
だが、対応出来るモバイルバッテリーが少ない

それなら
5V/9Vから12Vに昇圧する事を考えた方が楽かも
5V2.4Aのモバイルバッテリーからだと
12Wだから1Aは無理だろうが0.8Aぐらいならいける…筈
0644774ワット発電中さん2022/05/28(土) 01:21:32.18ID:wTJZLczK
3個直列で3端子レギュレータで落とせ
0645774ワット発電中さん2022/05/28(土) 01:26:41.37ID:O3i3qPFd
チェンジ、ゲッター

合体に失敗すると大変なことになるんじゃ?

タイミングをあわせるんだ!!!
0646774ワット発電中さん2022/05/28(土) 01:36:11.84ID:NQ/1HCVU
コネクタスレって無いですか・・・・

マイクロドットコネクタのマイクをミニプラグに変換するのを自分で見たいのですが
パーツのマイクロドットコネクタって1万円近くするのばっかりで・・・
せめて1000円くらい乗ってないでしょうか
0647774ワット発電中さん2022/05/28(土) 01:47:42.44ID:tkQPXRnf
これだけICが進化してモバイルバッテリーを直列につなぐことも出来ないなんて…
0648774ワット発電中さん2022/05/28(土) 01:51:41.89ID:i+nSFlQh
>>641
新規にモバイルバッテリー購入するなら簡単かも〜
手元のモバイルバッテリー見たら12V2A(PD,QC)の出力に対応してる
PD(12V出力)対応バッテリーなら「トリガーケーブル」使えば12V直接取り出せる〜
たぶん\(^o^)/
0649774ワット発電中さん2022/05/28(土) 02:02:18.86ID:DPEydm5A
ゲッター線を浴びた顔文字博士は常によだれをたらすようになってしまったんだ
0650774ワット発電中さん2022/05/28(土) 02:27:51.98ID:BlNsJGMy
>>646
そんなコネクタ知らないけど、これじゃダメなん?
https://item.rakuten.co.jp/sokutei-ait/ufd-04-03-06/
0651774ワット発電中さん2022/05/28(土) 23:16:21.83ID:8GoOMBxM
>>642
減圧じゃなくて、降圧ね。
0652774ワット発電中さん2022/05/29(日) 08:52:12.28ID:F2D59V4r
12V代案

 1)ノートパソコン用のモバイルバッテリ
12Vと5V USBも付いてるやつ
50000mAHくらいあるのでサイズがやや大きいが
 2)12Vポート付ジャンプスターター
12Vポート付はスマホより二回り大きいが
5Vのモバイルと兼用できる
ちなみに自分はスマホサイズのジャンプスタータ−を
USBでモバイルとして使ってる
 3)単4/単三のニッカド/NiMH電池を10個直列
(4本の)充電器が3つ必要だがw
0653774ワット発電中さん2022/05/29(日) 14:58:41.52ID:GcF1Nmax
マイコン始めたんですが、電子系の部品は秋月とかで揃いますけど、機械系のパーツはどこで買えばいいですか?

タミヤの楽しい工作シリーズとかになりますか?
0654774ワット発電中さん2022/05/29(日) 15:00:13.83ID:axgul4KP
>>653
機械系って言っても色々あるから、具体的に書けば色々教えて貰えると思うよ
0655774ワット発電中さん2022/05/29(日) 15:05:58.80ID:axgul4KP
歯車系はモノタロウがおすすめ
品揃えが多く殆どが日本製。送料も550円、1注文合計が3,500円以上なら送料無料なので買いやすい
0656774ワット発電中さん2022/05/29(日) 15:06:26.26ID:GcF1Nmax
>>654
作りたいものがハッキリしてないので
ストアの一覧を見ながら想像したいんですよね
0657774ワット発電中さん2022/05/29(日) 15:09:12.61ID:uVryKIGZ
モノタロウは1度でも注文するとぶ厚いカタログ4冊も送ってくるぞ
0658774ワット発電中さん2022/05/29(日) 15:36:22.20ID:moIduqzZ
>>653
モジュール0.5位のギアはaliでgear asortで見ると良い。他はほぼ全て3Dプリンタで作ってるな。バネは欲しいことがあったがsus wireで自分で作りゃいいし。
0659774ワット発電中さん2022/05/29(日) 15:48:43.82ID:JP1f0dn8
レゴとか割と色々あって面白い
高いだろうけど
0660774ワット発電中さん2022/05/29(日) 19:03:43.85ID:eMqY3J4L
>>653
秋月に置いてあるタミヤの楽しい工作シリーズじゃ不足ってこと?
0661774ワット発電中さん2022/05/29(日) 19:10:14.18ID:EMzwamWe
>>658
確かに自分で作るという手もありますね
3Dプリンターか…

機械以外は組み合わせを考えなくていいから楽なんですよね

電子→ブレッドボード+DIP
制御→PIC+C言語
0662774ワット発電中さん2022/05/29(日) 19:17:46.35ID:EMzwamWe
>>660
picとタミヤのチェーンを組み合わせて
チェーンが2秒右に動いて、1秒左に動く
っていうのを作ったんですけど
で?って感じです

チェーンスプロケットは面白いから欲しいと思って買いました
モータードライバーの練習用に作ったんですけどトマソンですねこれは
初心者質問スレ 153 YouTube動画>13本 ->画像>53枚
0663774ワット発電中さん2022/05/29(日) 19:24:58.22ID:a+9QsLLy
「で?」って自分にツッコんでんのワロタ
制作の目的が学習以外にないならそれで終わりにすりゃいいんじゃないかな
今後何か実用的なものを作るときには対処すべき課題が自ずと明確になるからそれまで放っておけばいい
0664774ワット発電中さん2022/05/29(日) 19:57:56.22ID:JP1f0dn8
チェーンのかけ方気になっちまう
まあそれは置いといてライントレースロボットとかが教材としてよくあるからそれやってみれば?
0665774ワット発電中さん2022/05/29(日) 20:59:32.84ID:2e17ZIwa
ヒートシンクのこの足の部分が欲しいんですがどこに売ってますか?
初心者質問スレ 153 YouTube動画>13本 ->画像>53枚
0666774ワット発電中さん2022/05/29(日) 21:33:33.68ID:h0IYCPm9
ミスミのノックピン
0667774ワット発電中さん2022/05/29(日) 22:18:22.97ID:2e17ZIwa
>>666
ノックピンというんですね
ありがとうございます探してみます
0668774ワット発電中さん2022/05/30(月) 00:59:15.49ID:MBBZGdbd
使ってるメモリが禿げてこれってハンダで直せないですか?

初心者質問スレ 153 YouTube動画>13本 ->画像>53枚
0669774ワット発電中さん2022/05/30(月) 01:20:05.50ID:OyJgBpmm
ハンダ最強伝説
06706412022/05/30(月) 02:21:09.49ID:bTwFx+Oh
>>648
ありがとうございますm(_ _)m
トリガーケーブルを注文しました。モバイルバッテリーはUSB PD対応みたいなので、これで試してみようとおもいます。動かす物は7インチのモニターなんですよね。野外で有線監視カメラのモニターとして使いたく。
0671774ワット発電中さん2022/05/30(月) 11:08:25.75ID:gOsD3gmW
スイッチング電源からDCDCコンバーター通してオペアンプに電力供給する時って
やっぱりノイズ乗りまくりですか?
0.1u, 4.7u...とかのパスコン以外になんか対策必要ですか
0672774ワット発電中さん2022/05/30(月) 11:21:50.67ID:F4UAWkqE
>>671
パッチンコア
0673774ワット発電中さん2022/05/30(月) 12:41:21.34ID:qN1LY7Mk
>>671
パスコンだけでは不足なので、コイルを入れてください。
こんな感じにすると、スイッチング電源からのノイズはほぼ消えます。
OP AMP、アナログ回路でもOKです。
https://imgur.com/ZjpJ7fG
0674774ワット発電中さん2022/05/30(月) 15:53:17.59ID:BBYCCwLl
>>673
丁寧にお描き頂いてありがとうございます。
参考にして作ろうかと思います。
0675774ワット発電中さん2022/05/30(月) 16:05:21.09ID:qN1LY7Mk
>>674
盲目的に、この回路で作れば、ほとんどのアナログ回路に使えるよ。
0676774ワット発電中さん2022/05/30(月) 16:07:11.34ID:BJgzI8b/
横からだけど保存しましたありがとう
0677774ワット発電中さん2022/05/30(月) 16:14:04.34ID:kqVr0S9O
部品多くてめんどい
0678774ワット発電中さん2022/05/30(月) 16:27:26.31ID:Ccv4oHJa
>>677
では乾電池を使いなさい
0679774ワット発電中さん2022/05/30(月) 17:28:27.10ID:WtASWWeb
部品数が多すぎならDCラインフィルタを買えばいいじゃなーい(^p^)
0680774ワット発電中さん2022/05/30(月) 19:27:07.45ID:xjTLkCWf
じじむさい回路やな
0681774ワット発電中さん2022/05/30(月) 19:37:41.43ID:f1BQbq3W
オーディオ用ならむしろ可聴領域が消えてるであろうスイッチング電源のほうが良いと思ってるけどな
オーオタに言ったら叩かれそうだが
0682774ワット発電中さん2022/05/30(月) 21:33:28.43ID:9U/BH6z9
このスレで聞くのが正しいかわからんのですが、このfpc
初心者質問スレ 153 YouTube動画>13本 ->画像>53枚
を3つほど直接繋いでiPhoneのカメラを外部化しようとしたら画面に何も映らなくなりました。
一つだけをiPhoneのカメラと本体の間につないだ場合は画面も正常に映るのですが、二本以降は画面が真っ暗です。
Amazonで見るとfpcをつなぐコネクタがあるようですが、fpc同士をコネクタで繋げば画面も正常に映るでしょうか?

2本目以降を直接繋ぐと映らなくなる原因と解決策わかる方いたら教えて頂ければ幸いです。
0683774ワット発電中さん2022/05/31(火) 00:57:09.18ID:204ds4a7
>>680
京都の人ですか
0684774ワット発電中さん2022/05/31(火) 07:46:23.51ID:SFO8Z/a9
>>664
チェーンのかけ方変ですか?
一応説明書見ながら作ったんですが
0685774ワット発電中さん2022/05/31(火) 12:09:53.62ID:YuPjwVNk
>>682
長さが長くなると、信号が崩れてしまい、正しく届きません。
接続途中にバッファを入れて、
波形崩れを直せば行けるかもしれません。
0686774ワット発電中さん2022/05/31(火) 12:25:03.07ID:6SEWi6wQ
こういうタイプのスピーカーターミナル、赤黒の押すとこ上下どっちが正解?
初心者質問スレ 153 YouTube動画>13本 ->画像>53枚
初心者質問スレ 153 YouTube動画>13本 ->画像>53枚
0687774ワット発電中さん2022/05/31(火) 12:25:35.85ID:m+Pab29z
0688774ワット発電中さん2022/05/31(火) 12:26:33.28ID:204ds4a7
好きにすればいい
0689774ワット発電中さん2022/05/31(火) 12:28:17.74ID:204ds4a7
>>686
これ色とか極数のシリーズ増やせば使い勝手いいと思うんですけどね
0690774ワット発電中さん2022/05/31(火) 12:29:24.47ID:CjmlqAsT
普通下じゃね?
写真のやつだと上にしたらホコリ入るし
0691774ワット発電中さん2022/05/31(火) 12:30:11.60ID:FFcf9yqj
クロックが速過ぎて 伝送路経路が
クロック波長を超えてる様な場合は無駄かもだけどな;しらんけど
0692774ワット発電中さん2022/05/31(火) 12:33:48.66ID:O1Ev9D1S
>>686
既製品のスピーカー見てみ
線を差し込む先が見えるように押す部分が下になってると思う
0693774ワット発電中さん2022/05/31(火) 12:35:21.03ID:6SEWi6wQ
みなさんありがとう。下にします。
0694774ワット発電中さん2022/05/31(火) 12:36:15.23ID:O1Ev9D1S
初心者質問スレ 153 YouTube動画>13本 ->画像>53枚
0695774ワット発電中さん2022/05/31(火) 13:02:21.04ID:V+hJBAPL
>>617

@YouTube

0696774ワット発電中さん2022/05/31(火) 14:10:30.58ID:lLtagaCG
>>685
返信ありがとうございます。バッファを入れるとは具体的にどうすればいいでしょうか?
0697774ワット発電中さん2022/05/31(火) 16:50:07.23ID:YuPjwVNk
バッファICを延長の途中に入れると言うことです。
そのICをハンダ付けします。波形調査にオシロが必要です。
ICを駆動する電源装置も必要です。
0698774ワット発電中さん2022/05/31(火) 18:34:29.38ID:JIEPRNwD
>>697
それ現実的じゃないと思うの(´;ω;`)
計測するオシロのスペックはどのぐらい必要?
0699774ワット発電中さん2022/06/01(水) 12:26:00.90ID:nJA86bks
ろじあな は兎も角、オシロって最大何チャンネルの迄が市販されているんだろう・・・
USBオシロのスタック機能を使えば256チャンネルとかあるのかしら?
0700774ワット発電中さん2022/06/01(水) 16:23:42.45ID:Rt3IenLn
>>685
通信速度や通信方式はわかって書いてる?
iPhone8の時点で最低でも4k60fpsを転送出来る速度になると思うけど!?
0701774ワット発電中さん2022/06/01(水) 20:24:12.78ID:vwDMp2Y7
15Aくらい流せるやつでオン抵抗のもっとも低いPchFET教えて下さい
自分で調べた限りだと2SJ607の9mΩが一番低そうなのですが
0702774ワット発電中さん2022/06/01(水) 20:33:18.35ID:gL520fbL
>>701
9ミリだったら、かなり低いと思うよ
0703774ワット発電中さん2022/06/01(水) 20:40:51.90ID:k/4nw1Tx
>>701
オン抵抗が問題ならNチャンネルを使う方法を考えたほうがいいとおもう
0704774ワット発電中さん2022/06/01(水) 20:42:11.94ID:vwDMp2Y7
>>702
>>703
ありがとうございます
ではこの9mΩを使います
Nは回路上ちょっと使いにくいのです
0705774ワット発電中さん2022/06/01(水) 20:51:02.18ID:ObTGyCY4
家庭用AC100Vの電気使用料金はWhだけできまるのですか
100Wの消費でも力率が0.5だと2A電流が流れますが
料金は電流に関係なく100W分しかかからないのでしょうか
0706774ワット発電中さん2022/06/01(水) 21:03:50.75ID:hW9thlCK
力率が低いと基本料金が割り増しされたはず
0707774ワット発電中さん2022/06/01(水) 21:04:41.96ID:UV7ivERM
LANケーブルとアンテナ線を同じところに配線したいけど周波数的な問題で干渉したりしない?
0708774ワット発電中さん2022/06/01(水) 21:10:42.78ID:k/4nw1Tx
アンテナ線って、同軸ケーブルのこと?
何メガで使う気?
0709774ワット発電中さん2022/06/01(水) 21:19:00.73ID:k/4nw1Tx
あっと、

仕様周波数より電力のほうが問題かな
大出力で漏れがあると、周波数無関係に干渉する
0710774ワット発電中さん2022/06/01(水) 21:19:43.44ID:UV7ivERM
>>708
テレビで使う予定
10~770Mhzと1032~3224Mhzって書かれているわ
0711774ワット発電中さん2022/06/01(水) 21:24:57.60ID:LQmgyQ0d
>>705
家庭用(低圧電灯契約)なら有効電力のみ考えればよい
0712774ワット発電中さん2022/06/01(水) 21:44:16.48ID:k/4nw1Tx
>>710
TVか、アマ無線のアンテナかと思ったw
TVからLANへの干渉はまずない
問題はLANからTVだけど
tvのチャンネルにドンピシャ重なるような信号が出てるとまずいかもしれんが
TVの同軸ケーブルは同軸ケーブルが電波を受けてしまわないための同軸ケーブルなので
多分無問題
0713774ワット発電中さん2022/06/02(木) 02:22:26.04ID:4Yb5ciqZ
絶縁体とは書いていませんが
体積抵抗率(Ωcm JIS-K6249) : 3.4×10
絶縁破壊強度(kV/mm JIS-K6249) : 21
という記載のあるサーマルパッドって
チップの足に接触してしまってもショートしないのでしょうか?
0714774ワット発電中さん2022/06/02(木) 03:38:13.06ID:du5FWuOM
>>704
出来るだけゲート電圧を入れてね。
マイコンの3.3V出力で駆動したら焼けました、ということのないように、
可能なら10Vくらい入れる。
0715774ワット発電中さん2022/06/02(木) 04:58:50.10ID:qH1S+Rjo
>>705
本のように売れなければ返品
しかも輸送料はロス
そんな感じです
0716774ワット発電中さん2022/06/02(木) 04:59:46.37ID:qH1S+Rjo
>>706
一般家庭(電灯線)で判別できるんですか
0717774ワット発電中さん2022/06/02(木) 06:01:06.00ID:AW9lsr56
できないから関係ないっつってんだ
0718774ワット発電中さん2022/06/02(木) 12:00:22.35ID:/B0PZTLd
昔ながらのアラゴの円板ぐーるぐる、な
電力計なら兎も角、
最近のスマートメーターでも
はたしてわからないと言い切れるのだろうか・・・しらんけど;^p^)
0719774ワット発電中さん2022/06/02(木) 12:38:51.31ID:/B0PZTLd
昔の電力計はさ、なんか 頑張って仕事してるって感じがして、
見ていて飽きないというか がんばれー 電力計がんばれーって
内心応援しちゃいたくなるくらいかわいい印象だったけど、
なんでスマートメーターってあのよーなデザインにしたんだろう?

なんか既視感があると思ったら、あれだわ。スマートメーターって、
はいだしょうこさんの描かれたドラえもん にクリソツなんよな、あれ;^p^)
0720774ワット発電中さん2022/06/02(木) 14:04:19.55ID:crvxsANG
電気のメーターとかなんであんな見ずらい位置に設置すんのかねえ
表示器付いてても見えねえっての
0721774ワット発電中さん2022/06/02(木) 14:30:28.38ID:/B0PZTLd
見やすけりゃいいって思ってた時代が僕にもありました…

@YouTube

0722774ワット発電中さん2022/06/02(木) 15:46:13.35ID:T28Ts9ns
>>720
お前はいちいちチェックしてんのか?
今は検針員すら来ないんだぜ。
0723774ワット発電中さん2022/06/02(木) 15:51:51.51ID:c7/qgwfO
>>720
不在を知りたい人たちから身を守ってるともいえる
0724774ワット発電中さん2022/06/02(木) 16:06:46.62ID:g/lv2VTD
>>712
確かにノイズ対策の為の同軸ですね
実際に配線して駄目だったら対策検討したほうが良さそうですね
0725774ワット発電中さん2022/06/02(木) 16:40:37.91ID:/B0PZTLd
そうなのか? 信号の帯域上やむをえず同軸使ってるだけで
コモンモードノイズに対して同軸はむしろ弱いみたいな話を
むかしノイズ系の蘊蓄本で見かけたような記憶があるが・・・しらんけど;へ○へ)
0726774ワット発電中さん2022/06/02(木) 17:02:49.80ID:crvxsANG
>>725
マッチング取れてないと結構駄々洩れだったりするけど
pc使うとTVに妨害が出るとか、最近聞かなくなったから

たぶん大丈夫、たぶんだけど
0727774ワット発電中さん2022/06/02(木) 19:05:02.06ID:T28Ts9ns
>>726
地デジになったからじゃねぇの?
0728774ワット発電中さん2022/06/02(木) 19:24:04.99ID:crvxsANG
>>727
たぶんそう
0729774ワット発電中さん2022/06/02(木) 19:26:33.60ID:crvxsANG
デジタルになって電波弱くても、マルチパス少々有っても、くっきり映るようになっていいんだが
中身が、さえないので、全然見なくなった
0730774ワット発電中さん2022/06/02(木) 20:39:01.27ID:zojCZrgH
>>729
ただしアナログではかろうじて見えたような弱いのは完全に見えなくなった
0731774ワット発電中さん2022/06/02(木) 21:12:17.67ID:emMPEoU/
うちはTV東京系の電波が弱くて、学校でみんなの話題についていけなくて子供の頃くやしい思いしたなあ
0732774ワット発電中さん2022/06/02(木) 22:27:35.37ID:M4VkTJmR
>>711
だとすると
例えば100Wで力率0.1の暖房装置があるとすると
1KW相当の発熱で電気代は100Wだけ
ということもあるのですね
0733774ワット発電中さん2022/06/02(木) 22:33:09.29ID:yZaqpcl7
>>724
LANの方もシールドするとか(STP+STP用のHUB)
0734774ワット発電中さん2022/06/02(木) 22:33:34.72ID:hi0t6apv
>>732
ないよ
力率0.1ってのは0.9分が発熱するという意味ではないからね
0735774ワット発電中さん2022/06/02(木) 22:50:40.99ID:vDSCES13
>>734
1kW負荷時相当の「送配電線部での」発熱という意味なら
無くもないんんじゃね?(へ〇へ)
0736774ワット発電中さん2022/06/02(木) 23:04:47.42ID:hi0t6apv
>>735
そういうことを言いたいなら
力率1かつ1kW負荷時相当の「送配電線部での」発熱
と言った方がより正確だと思う
0737774ワット発電中さん2022/06/03(金) 00:00:47.75ID:PpJap8mr
確かに
>電気代は100W
だけかもしれんが

>1KW相当の発熱
すれば・・・
電柱、配線、盤が一瞬で火だるまw
0738774ワット発電中さん2022/06/03(金) 04:10:24.33ID:R9JVYE6u
100w分しか発熱せんから、100w分の料金になるわけだがw
0739774ワット発電中さん2022/06/03(金) 05:18:46.83ID:X38CHgcT
伝票にkW/hと記載されている
0740774ワット発電中さん2022/06/03(金) 05:43:24.82ID:R9JVYE6u
分に反応してる?

ふんでなくてぶんね
0741774ワット発電中さん2022/06/03(金) 05:58:15.38ID:R9JVYE6u
ついでに、無効電力の単位ははワットじゃなくて「ボルトアンペア」だったりする
0742774ワット発電中さん2022/06/03(金) 06:03:27.03ID:R9JVYE6u
違った
ボルトアンペアは皮相電力だった失礼
0743774ワット発電中さん2022/06/03(金) 08:50:38.36ID:m8jSHnhQ
>>741
varか

ヴぁーる・か、と言ったんであって、バーカって書いたわけじゃないから変な勘違いしないでよね!!(へQへ///)
0744774ワット発電中さん2022/06/03(金) 13:49:12.26ID:YK/+ZIZA
100[W]で力率0.1の負荷というから誤解が生じるので
1[kVA]で力率0.1の負荷と言えばいいんじゃね
0745774ワット発電中さん2022/06/03(金) 13:50:33.84ID:X38CHgcT
>>744
消費税込みの価格から消費税分はいくらでしょう
みたいな算数か

わからんけど
0746774ワット発電中さん2022/06/03(金) 13:56:46.86ID:EfY0L56Q
>>743
それ、圧力単位
0747774ワット発電中さん2022/06/03(金) 14:00:58.31ID:W5VDJf1/
>>746
var = volt, ampere, reactive power(無効電力)
0748774ワット発電中さん2022/06/03(金) 20:03:44.41ID:9cs8f+5L
wt% か mol% の違いに近いんじゃね
0749774ワット発電中さん2022/06/03(金) 20:10:02.56ID:v0Suen70
>>746
圧力ならbar
0750774ワット発電中さん2022/06/03(金) 20:46:31.88ID:Vl1CHne2
>>732
> 例えば100Wで力率0.1の暖房装置があるとすると
つまらない突っ込みだが、原価が高そうだなw
0751774ワット発電中さん2022/06/04(土) 08:12:27.24ID:wVz6s7kz
I3Cとは何ですか?
0752774ワット発電中さん2022/06/04(土) 09:11:45.06ID:gltotBFA
へええ。I2Cと同じで2線式、最大10Mビット/秒か。
0753774ワット発電中さん2022/06/04(土) 09:23:39.18ID:LVNFtTgB
IIIC
0754774ワット発電中さん2022/06/04(土) 11:19:18.95ID:1pgBm0fF
アイキュピカルシー って読むの?
0755774ワット発電中さん2022/06/04(土) 13:49:11.50ID:LVNFtTgB

@YouTube



@YouTube



@YouTube

0756774ワット発電中さん2022/06/06(月) 13:34:22.78ID:9Q5k8Ybg
>>689
5.1ch用の6色くらいのはあるな
0757774ワット発電中さん2022/06/07(火) 21:25:10.66ID:DYMGkhWY
積分回路の電源切ったときにオペアンプに負電圧がかかり壊れるとかありますか?
次々と壊れるのでコンデンサと直列に抵抗入れたら壊れなくなったんですが
0758774ワット発電中さん2022/06/07(火) 22:57:42.90ID:3PrghvBw
オペアンプの種類とコンデンサの容量、電源の落ちる速さなどの条件で壊れることもあるかも。
0759774ワット発電中さん2022/06/08(水) 06:11:59.84ID:7juPdNjr
あるある、電源切ると壊れる回路
昔はトラ技なんかで特集されてたりしたね
0760774ワット発電中さん2022/06/08(水) 07:46:38.58ID:qA5MuySo
>>757
CMOS入力?
0761774ワット発電中さん2022/06/08(水) 11:46:10.46ID:vDdlUPJp
三端子レギュレータも保護ダイオードがいるやつあるね
0762774ワット発電中さん2022/06/08(水) 20:34:29.73ID:q3GE8Ixj
基本必要なんじゃないの?
入力側が遅れて立ち下がることが確実なら省略できるってだけで
0763774ワット発電中さん2022/06/08(水) 22:50:20.06ID:2N6CP1Ku
新しい設計のレギュレータだとダイオードが入っているものが多くなってる。
0764774ワット発電中さん2022/06/09(木) 18:40:33.73ID:OfVaFQB4
結局ダイオードは必要なんだな。
0765774ワット発電中さん2022/06/09(木) 18:59:28.21ID:eA9p1rjk
三端子の下流より大きな電解コンが上流に入っていれば要らんのでは
0766774ワット発電中さん2022/06/09(木) 19:00:50.76ID:SArBEQS6
>>765
7805とかは入力0vで主力側に電圧描けても壊れない
0767774ワット発電中さん2022/06/09(木) 19:04:44.18ID:2HYpD462
ちょい質問なんだけどAmazonでGreenwaveのEMI dirtyメーターってどんな仕組みで電気の干渉を検知してるんだろうか?
中身分解したブログとか探しまくったけどないしオカルト品なのかな?と思ったけどそうでもないっぽい
朝からずっとモヤモヤしてる。

初心者質問スレ 153 YouTube動画>13本 ->画像>53枚
0768774ワット発電中さん2022/06/09(木) 19:10:46.48ID:SArBEQS6
どういううたい文句になってんのかしらんけど、ただの電界強度計なんじゃないの?
0769774ワット発電中さん2022/06/09(木) 20:06:41.27ID:2HYpD462
>>768
分解された中身を解説してるサイトとかってないですかね?
想像すると中身は単純な回路に3万も払いたくないので作ってみたいんですよね…
0770774ワット発電中さん2022/06/09(木) 20:15:32.10ID:2HYpD462
正確でしっかりしたものだしベストセラー品なんだけど昔くっそ高いもんかって中身開けたら原価1000円も行かないようなしろもんだったからな〜
オヤイデみたいな
0771774ワット発電中さん2022/06/09(木) 20:18:14.05ID:SArBEQS6
>>769
ツべに使ってる所の動画があった
1pW9WTiNuls

電灯線に乗ってきたノイズ計ってるみいたいね
0772774ワット発電中さん2022/06/09(木) 20:34:09.13ID:L+S2HzIw
>>765
例えば12Vから5Vにレギュレータで落とす場合、12V系に負荷が有る場合は電解コンの容量の大小では決まらない
0773774ワット発電中さん2022/06/09(木) 20:56:12.75ID:2HYpD462
一万7千円をオマ国値段で売ってんのきつい
0774774ワット発電中さん2022/06/09(木) 20:58:08.50ID:RQX/IANA
>>773
そういうのこそ自作する価値があるんじゃないの?
0775774ワット発電中さん2022/06/09(木) 21:39:48.03ID:2HYpD462
>>774
ググりまくってんだが分解画像がない
回路図さえあれば作れそうなのに
0776774ワット発電中さん2022/06/09(木) 21:43:12.59ID:8NGU7v1M
1個買って分解して
売る時に同じ結果を出す所を写真に取れば良い
0777774ワット発電中さん2022/06/09(木) 22:14:21.18ID:SArBEQS6
>>775
すぐ殻割しちゃうような人が、興味示さないタイプの商品ってことでしょ
0778774ワット発電中さん2022/06/09(木) 22:41:02.29ID:jPi4zRxc
職人が絶妙に調整した手巻きコイルが盛りだくさんで搭載とかな
0779774ワット発電中さん2022/06/10(金) 13:26:03.23ID:RS4KtO+d
ダイソーの300円スピーカーに付いてるパワーアンプ8002について質問です。

https://www.hmcircuit.jp/daiso300yenusbsp/daiso_usbsp.html

>データシートでは1段目のアンプの利得を1倍として最終利得2倍で組んでありますが、
>ダイソーSP 2017年モデルは1段目を3.24倍とし最終利得約6.5倍、
>2019年モデルでは1段目を約2.2倍とし最終利得4.4倍で組まれています。

と、外部抵抗で利得の設定が出来るんですが、2つのモデルは抵抗値が違って利得も違うんですが、
実用上、消費電力的には同じなんですか?
利得が大きいほうが少電流で鳴らせるものなんですか?
0780774ワット発電中さん2022/06/10(金) 13:28:24.94ID:RS4KtO+d
乾電池で使おうと思ってるのでなるべく電池の持ちを長くしたいです。
0781774ワット発電中さん2022/06/10(金) 13:42:35.52ID:ZfyNxvIt
スピーカーをドライブする電流が大部分だろうから同じ音がでる状況ならほぼ差はでないと思う。
0782774ワット発電中さん2022/06/10(金) 14:10:27.78ID:lhSNQumt
利得というのは信号の倍率みたいなもので
消費する電力は主にスピーカーを震わせる強さなんだろうから
ドライブ先のスピーカーユニットの性能が もしも同じなら という仮定に立てば
利得が大きい方が電池の減りは激しいんじゃね?しらんけど
0783774ワット発電中さん2022/06/10(金) 18:46:15.88ID:RRnhKbb9
同じ音の大きさで聞くなら、有意な差はないのでは。
あえて言うなら、同じ信号ソースで異なるゲインの出力アンプで同じ大きさの音にするなら
高ゲインのアンプはいったん抵抗でレベルを絞っている可能性が高い。その場合はその分ロスが大きいだろね。でも違いはほぼゴミ。
0784774ワット発電中さん2022/06/10(金) 19:33:58.03ID:W4sEJR0b
入力はボリュームで分圧してるだけだから消費電力はほぼ変わらない
このアンプは2段というより一段の反転アンプ出力に1倍の反転アンプを付けて1段目のアンプの出力とその反転でスピーカーをドライブしてる感じか
0785774ワット発電中さん2022/06/10(金) 19:43:25.50ID:07PvPQPv
>>784
入力が無音ならそうだけど、
ソース自体の入力が大きければやはり消費電力は増えるよ
0786774ワット発電中さん2022/06/10(金) 19:46:05.33ID:W4sEJR0b
そりゃ出力がデカくなるから
0787774ワット発電中さん2022/06/10(金) 19:48:00.51ID:W4sEJR0b
消費電力はアンプのゲインじゃなくて出力に依存する普通は
中身が純A級とかなら無音が一番電気食うかも知れないが
0788774ワット発電中さん2022/06/10(金) 20:44:46.84ID:dAQLP+JW
A級は、のべつ幕なし電気食ってるよ、出力あってもなくても同じだけ
0789774ワット発電中さん2022/06/10(金) 21:25:01.58ID:1NYGKMsj
周波数が上がるとインピーダンスが高くなる構成になってれば無音時が一番電気食う極微量
0790774ワット発電中さん2022/06/10(金) 23:17:43.57ID:UPq2ZBX1
誰かこの使い方教えて下さい
AC電源のノイズリダクションみたいなんだけど入出力どのようにすればいいのかわからない…

初心者質問スレ 153 YouTube動画>13本 ->画像>53枚
0791774ワット発電中さん2022/06/10(金) 23:19:33.84ID:UPq2ZBX1
>>790
1と2にac繋げるのか2と3に繋げるのか
0792774ワット発電中さん2022/06/10(金) 23:32:21.97ID:MJeAnTAG
1と2と3にAC繋ぐんだろ
0793774ワット発電中さん2022/06/11(土) 00:00:49.04ID:/Q5ot0+j
キスした
0794774ワット発電中さん2022/06/11(土) 00:30:07.51ID:jyLZlXMg
2がアース、1と3がL/N
0795774ワット発電中さん2022/06/11(土) 01:15:36.78ID:A0hBnk6J
>>794
あざっす!
0796774ワット発電中さん2022/06/11(土) 15:31:50.59ID:U/moWniQ
インピーダンス x 電流 x 電流
または
電圧 x 電圧 ÷ インピーダンス
では
0797774ワット発電中さん2022/06/11(土) 18:43:43.32ID:cFiPIPf6
>>788
>A級は、のべつ幕なし電気食ってるよ、出力あってもなくても同じだけ

じゃあスピーカーが10Wの出力で鳴っているとして
無音の時はその10Wはどこで消費してるの?
0798774ワット発電中さん2022/06/11(土) 18:44:37.07ID:ESfmWmTA
発電所に戻ってる
0799774ワット発電中さん2022/06/11(土) 18:46:29.02ID:ESfmWmTA
正確には、一旦消費したのをまた発電となって
発電所に戻ってる
0800774ワット発電中さん2022/06/11(土) 18:55:26.94ID:itSgAng/
>>797
増幅素子で
A級のイメージは無信号時に素子に10W流しといてスピーカーに流す分をそこから分ける感じ
だから無信号時に一番熱くなる
0801774ワット発電中さん2022/06/11(土) 19:16:25.13ID:ESfmWmTA
>>800
違います。
そういうイメージで例えるなら、一定の消費電力に揺らぎを与える感じです
0802774ワット発電中さん2022/06/11(土) 19:29:10.12ID:BHiMsNn4
最終段のトランジスタや真空管が消費してる説
0803774ワット発電中さん2022/06/11(土) 19:54:05.80ID:Jq34x0w7
>>797
スピーカーが鳴っている電力じゃなくてダナ
なっていようがなっていまいが、出せる最大出力と同じ「だけ終段が消費して熱になってんのよ
0804774ワット発電中さん2022/06/11(土) 20:28:26.18ID:cFiPIPf6
じゃあ無音の時はその10Wはプレートやコレクタ損失になるのか?
0805774ワット発電中さん2022/06/11(土) 20:45:34.31ID:Jq34x0w7
>>804
そうだよ、てか10wから離れろw
0806774ワット発電中さん2022/06/11(土) 20:50:10.89ID:Jq34x0w7
10wの音を出しても、アンプの最大出力が100wなら、ずっと100w食ってるのがA級なのよ
実際にはA級でもフルスイングさせると歪が大きいから、もっと食わしてるはずよ
0807774ワット発電中さん2022/06/11(土) 22:12:16.91ID:itSgAng/
>>801
馬鹿は黙って
0808774ワット発電中さん2022/06/11(土) 22:16:30.18ID:Jq34x0w7
>>807
合ってるけどw
0809774ワット発電中さん2022/06/11(土) 22:22:53.89ID:v5Ffgb+6
>>807
黒ラベル
0810774ワット発電中さん2022/06/12(日) 01:17:49.83ID:z2I0JehW
A級アンプでは
電源「電流」は、負荷への出力によらず一定
その電流を、出力段と負荷で分け合うというのもだいたい正しい(見た目の経路が違う)
電力効率は、(電圧との積の合計なので)最大出力時がもっとも良い50%。それ以外のときは50%未満
0811774ワット発電中さん2022/06/12(日) 10:10:15.91ID:WBX4hWsk
最終段で消費される説明動画

@YouTube

0812774ワット発電中さん2022/06/12(日) 10:15:51.55ID:WBX4hWsk
観たことある人多いと思うけど
2個2個技術部
sm12736646
sm12884261
0813774ワット発電中さん2022/06/12(日) 11:33:49.79ID:xsQUy8Gw
普通にURLで貼っとくれ
0814774ワット発電中さん2022/06/12(日) 14:40:51.12ID:TZcdfv7T
汎用ロジックICの動作時間の計算方法が紹介されている本とかってありませんか?
使い方はググればいっぱい出てきますが時間を評価する方法を書いているサイトが見つけられません
最小値と最大値を見積もる方法を知りたいです
0815774ワット発電中さん2022/06/12(日) 14:42:15.57ID:ZwE0uJLF
ニコ動のURLは昔からNGワーオで直に貼れない
0816774ワット発電中さん2022/06/12(日) 14:47:27.56ID:JqbCs/r7
こういうのが好き
https://nico.ms/sm23955087
0817774ワット発電中さん2022/06/12(日) 14:47:54.35ID:qQ1yfDwA
>>814
データー使徒
0818774ワット発電中さん2022/06/12(日) 15:20:36.09ID:WBX4hWsk
32Hz の低周波に 8Hz で変調掛けてみたら気持ち悪かった
聴こえるというより体感的に気持ち悪い揺れ
0819774ワット発電中さん2022/06/12(日) 15:21:50.30ID:WBX4hWsk
>>814
企画雹
0820774ワット発電中さん2022/06/12(日) 18:36:48.23ID:NcLi/UTD
>>797
熱になるだけ
0821774ワット発電中さん2022/06/15(水) 11:26:36.02ID:Fm5O4WdP
スレ的にここで良いの分からないのですが、一旦質問させてください。

スマホのバッテリーレス運用をしたいと思い、膨らんだバッテリーパックからBMC基板を取り外してBMCの電極部に直接DC4Vを供給しました
このままだとコールドブートができないようで、最初に数秒スマホ本体のUSB端子から給電すると起動できるようになるのですが、一旦BMC電極への給電を停止するとまたコールドブートできなくなります。

こういう保護回路的な動作を回避するためのアイデアはないでしょうか

BMCを飛ばして本体側の端子に直接電源とサーマル端子に対抗を付けただけだと起動できませんでした。バッテリーパックにID端子がついておりバッテリー認証を行ってるようです。
0822774ワット発電中さん2022/06/15(水) 12:11:21.57ID:v1UspyHk
機種名やバッテリーの型番、基板の写真とか出した方が良い。ブートしないのも供給側の過電流保護が働いてるだけかもしれないし、バッテリーの方も普通はサーミスタ監視で認証なんかしてないのでは。
0823774ワット発電中さん2022/06/15(水) 12:37:17.96ID:y7piFkmA
バッテリー外してUSB給電で動かないの?
0824774ワット発電中さん2022/06/15(水) 14:17:03.05ID:Fm5O4WdP
>> 822
失礼しました。機種はHUAWEI Honor9です
初心者質問スレ 153 YouTube動画>13本 ->画像>53枚 バッテリーと基板
初心者質問スレ 153 YouTube動画>13本 ->画像>53枚 バッテリー端子
電源は安定化電源を使用していて、電流制限を5Aに設定しています。
USB側に数秒給電したあと、USBを抜いて安定化電源だけで給電していても起動はできています。
起動中の最大電流も3Aまでは到達していないようです。

>> 823
残念ながらできません。
あと、そのポートを3Dプリンタ接続用に使いたくてこんなことしてます。
この機種はYケーブルやDockを使って給電とデバイスの同時使用ができないみたいなんです。
0825774ワット発電中さん2022/06/15(水) 16:40:29.08ID:tX+KuYgA
こーるどぶーと をググって、電源断状態からの
起動処理をそう呼ぶことを知ったが、検索に
ヒットした候補中に 人肌でゆっくり温めてから起動しろ
というトラブル対策があってわろた そっちのコールドかよ!!
0826774ワット発電中さん2022/06/15(水) 16:52:19.08ID:V3w/XrE5
>>825
寒いところだマジでそういうのあるよw

昔のダンプとかも、寒い朝はエンジンの下で新聞紙燃やして「からエンジンかけてたりしたwww
0827774ワット発電中さん2022/06/15(水) 16:56:15.85ID:nE+s3rdX
>>825
面白英語シリーズ

@YouTube

0828774ワット発電中さん2022/06/15(水) 17:58:10.13ID:XA5AgZOL
いやさすがにグロープラグくらい付いてるだろ
0829774ワット発電中さん2022/06/15(水) 18:02:26.53ID:V3w/XrE5
>>828
それでもかからねえんだな当時のは
下で燃やすっても、実際にはキャブレターをあぶってたんだと思うよ
0830774ワット発電中さん2022/06/15(水) 18:08:49.79ID:G+1jSxsG
ガソリンエンジンのダンプなんてあるのか
軽トラダンプか
0831774ワット発電中さん2022/06/15(水) 18:10:11.62ID:V3w/XrE5
ディーゼルにキャブなんか無えってつっこまれそうだから空気旧入口って言っとこうか
0832774ワット発電中さん2022/06/15(水) 18:11:15.69ID:V3w/XrE5
あら、先に突っ込まれちゃったよw
0833774ワット発電中さん2022/06/15(水) 18:18:43.84ID:V3w/XrE5
当時はガソリントラックは、まだ走ってたけどな
ダンプはどうだったかしらんが
0834774ワット発電中さん2022/06/15(水) 19:46:04.58ID:vbuj0g0A
>>831
今は出口なのか?
0835774ワット発電中さん2022/06/15(水) 19:48:02.26ID:q79PzbaH
二次大戦の戦車兵の逸話で炙って起動というのは何かで読んだ記憶がある。

寅壱というと僕ら的には作業服メーカーだけど
ミリオタに言わせればドイツの名戦車。
とても強力だったと伝説級の扱いだけど
アレを作れたこと以上に、
あんなしろものを まともに扱えた現場の戦車乗りの技術が凄いのだ、という話もある。

二次大戦時は自動車技術者も花形というか憧れの技術者で人材が豊富だったのだろうと推測。
(戦車関係は自動車技術の延長だと思う)、だがウクライナの噂を聞くと戦車の黄金期はもう過去かも知れませぬな。

はてさて、第三次世界大戦では ドローンが比重を増すだろうけれど
近い将来 徴兵されるだろう俺らも、電子工作ぎじつの腕を磨いておかねばならぬのではなかろうか?

♪わかーものよー からだをきたえておけ〜(^p^)
0836774ワット発電中さん2022/06/15(水) 21:44:40.64ID:8rw7ncW7
マイクロバスでスキー場、凍結してプレヒーターも効かず。
宿でお湯もらって、エンジンにかけて融かした。
0837774ワット発電中さん2022/06/15(水) 22:33:22.48ID:BnAd6zGi
>>826
それで車燃やすオチだよな
0838774ワット発電中さん2022/06/15(水) 23:12:46.38ID:V3w/XrE5
燃やせたらそれはそれですごい
0839774ワット発電中さん2022/06/16(木) 00:46:27.11ID:JDwXoghY
>>826
ヤンマーディーゼルだとインマニからガソリン吹いたりね
0840774ワット発電中さん2022/06/16(木) 08:10:27.58ID:7eC//7QE
ディーゼルではガソリン吹かない
0841774ワット発電中さん2022/06/16(木) 08:11:38.74ID:7eC//7QE
焼玉エンジンって、外部加熱で起動させるぞ。
0842774ワット発電中さん2022/06/16(木) 08:21:29.23ID:BSUL+7JR
ディーゼルエンジンってフツーグローブラグでシリンダー温めるんでは
0843774ワット発電中さん2022/06/16(木) 08:36:17.08ID:4SF3OB7K
>>840
CVCCの着火室をディーゼルに置き換えて
軽油ガソリン両方使うエンジン作れないかな(^p^)
CVCCエンジンはメインシリンダーと着火室繋がったままだけど
圧縮比がちげーからそこが面倒そうだけど☆彡

別件、水素ガスは金属をもろくするらしいけど
水素燃料の内燃機関って強度大丈夫なのかしらん・・・;^p^)
0844774ワット発電中さん2022/06/16(木) 11:50:09.08ID:n1Lgn4X7
オシロスコープを買おうと思うのですがアドバイスお願いします。

自分の用途としてはArduinoやラズパイのI2CやSPI信号を2CHで見るくらいなのですが(ロジックアナライザは既に持っています)
SPIが1MHzぐらいなので

https://www.アマゾン.co.jp/dp/B08ZRVH87B/

こちらの40MHzのオシロスコープで充分でしょうか?

また、これの上位機種の70MHzとか100MHzでみる信号はどういったものになりますか?

CPUのような高速信号、もしくはFMラジオの70~80MHzの電波などでしょうか?
0845774ワット発電中さん2022/06/16(木) 12:51:13.00ID:R9jMvJqI
>>844
ロジック回路のような角の立った信号を観測する場合は
信号の周波数の10倍の帯域があればgood
デジタルオシロの場合サンプルレートにも同じことが言える

よくわからんかったら、予算の範囲で可能な限り、周波数帯域の高い奴に汁

帯域足りんと角なまって丸くなる
0846774ワット発電中さん2022/06/16(木) 12:53:41.71ID:tj3YMyOy
ロジックならロジアナで十分だろ。
ってか、2chだと足りなくなるよ。
0847774ワット発電中さん2022/06/16(木) 13:04:57.62ID:R9jMvJqI
ロジアナは波形が見えんから、オシロは持ってて損は無い
0848774ワット発電中さん2022/06/16(木) 13:49:59.59ID:6Ep2d+Ox
ロジアナ買えば、I2CやSPIでどんな通信してるか解る(デコード機能)
なんでロジアナ買った方が楽

オシロも4chとかでまともな奴買えば、それぐらい出来るけど

ってから初心者ならADALM2000とかAnalogdiscveyみたいなオールインワンな奴買った方が手間が無いとは思うが
0849774ワット発電中さん2022/06/16(木) 14:23:16.28ID:HnkDu99r
ありがとうございます。

>>845
出来るだけ周波数帯域の高いものを買おうとすると100MHzのやつになるのですがこれぐらいの周波数を扱う電子工作って何になるんでしょうか?

>>844にも書きましたが、CPUのような高速信号やFMラジオの電波を見るのに使うのでしょうか?
0850774ワット発電中さん2022/06/16(木) 14:27:35.10ID:R9jMvJqI
>>849
だからちゃんと観測できるのは帯域の1/10くらいの周波数の信号までだってば
0851774ワット発電中さん2022/06/16(木) 16:16:22.69ID:ekH6dM48
>>843
今はガソリンエンジンで圧縮点火やれてる時代
そもそも軽油自体不要
0852774ワット発電中さん2022/06/16(木) 16:18:16.93ID:9DfP+mmW
ヘッド面研w
0853774ワット発電中さん2022/06/16(木) 16:21:44.44ID:vBLCxARD
ハイオク仕様ww
0854774ワット発電中さん2022/06/16(木) 16:46:30.45ID:BxB7H98Y
波形ではなく信号を見たい感じだな
0855774ワット発電中さん2022/06/16(木) 17:15:41.06ID:K/dlPNmr
波形は正弦波だけではない。矩形波や三角波、パルス波、何でもあり。
その繰り返し周期の逆数を周波数と言えば周波数だが。フーリエ解析における
直交基底である正弦波の周波数成分でいうと、正弦波以外は多くの高調波を持ち
その帯域は基本周波数の何倍にもおよぶ。なので、それらの波形を正しく観測する
場合は高調波を含んだ周波数までカバーするオシロスコープが必要となる。
単に正弦波波形だけ観測したいならオシロスコープなど必要なく周波数カウンタ
や周波数計で済む・
波形、信号を観測したいからオシロスコープはそれなりの帯域が必要となる。
0856774ワット発電中さん2022/06/16(木) 17:45:58.26ID:MfaORUga
ギガオシロは初心者お断り仕様
0857774ワット発電中さん2022/06/16(木) 19:39:02.28ID:APFjaJqu
>>849
とりあえずだ、まずは安いのでいいからさっさと買え。
で、使っていて不満が出てきたら次のをどうするか考えろ。

ここでぐだぐだ議論するより、なまくらでもいいから、
実際の波形を見るのが一番なんだよ。
0858774ワット発電中さん2022/06/16(木) 20:04:01.01ID:hQOMB9Wp
>>855
君は説明が下手だけど言いたいことは判った
0859774ワット発電中さん2022/06/16(木) 20:25:48.75ID:TqgFZbLs
オシロ使うときは波形を見たいんじゃなく
遅延を見たいとかHの電圧見たいとかだな。
0860774ワット発電中さん2022/06/16(木) 20:32:06.31ID:o0/Lak+w
オロシ使うときは焼き魚だけじゃなく
脂っこい焼肉や鍋料理にポン酢と一緒に食べたいとかだな。
0861774ワット発電中さん2022/06/16(木) 20:33:03.39ID:R9jMvJqI
それをまとめて波形と言う
0862774ワット発電中さん2022/06/16(木) 20:38:07.87ID:TqgFZbLs
豚骨ラーメンも波形に入りまつか?><
0863774ワット発電中さん2022/06/16(木) 20:39:33.09ID:R9jMvJqI
豚骨は、はかたです
0864774ワット発電中さん2022/06/16(木) 21:16:46.33ID:LPqUCxez
>>857
正解です。
0865774ワット発電中さん2022/06/16(木) 21:32:49.56ID:LPqUCxez
>>849

>出来るだけ周波数帯域の高いものを買おうとすると100MHzのやつになるのですが
>これぐらいの周波数を扱う電子工作って何になるんでしょうか?
ラジオとか高速なマイコンの信号とかを見るために使用します。

例えば、ラズベリーパイのポートからの細いパルス(短い時間のパルス)を見た時、
100MHzのオシロだと0←→3.3Vの矩形波が、ビシッビシッと直角に0←→3.3Vで見えるけど
10MHzのオシロだと角がダラ〜っと丸まって見える。
でも、デジタルの測定では、高さは0←→3.3Vと決まっているので、時間軸の情報のほうが
重要なことが多いです。だから、そういうときなら10MHzのオシロでも使えます。

100MHzのオシロが欲しいけど予算が無いというときは、
新品を買わずに中古のをヤフオクで買うとよいです。
ヤフオクで10MHzの中古を安く買って、あれこれ使って波形を見てみて、
実際にどんな時に不満が出るのか、知ってから、100MHzを買えば良いです。
中華のオシロを新品で買うくらいなら、ヤフオクで国産かテクトロの中古を買った方がいいです。
同じUSA製でも、アジレントよりテクトロの方が使いやすいと近所の奥さんにも評判です。
0866774ワット発電中さん2022/06/16(木) 21:35:33.88ID:LPqUCxez
>>849
使いにくいのは、レクロイのオシロ、横河のオシロでしょうか。
まあ普通というのがアジレントのオシロ。
使いやすいのは、ダントツ テクトロです。
0867774ワット発電中さん2022/06/16(木) 22:58:57.23ID:/bCQ4f5H
圧着工具はゲンミツなのに、オシロの話になるとすげえ適当で、自分趣味の一般化するんだな。
0868774ワット発電中さん2022/06/17(金) 00:01:10.63ID:yl4brQjm
>>844,849
初心者質問スレ 153 YouTube動画>13本 ->画像>53枚 (横長過ぎたゴメン)
オシロの帯域と見れるだろう波形の比較 (信号にリンギングが発生してると仮定)
owonのHDS200はサブ機なら良いけどメインにはちょっと・・・

アーニャおしろにすみたい\(^o^)/
0869774ワット発電中さん2022/06/17(金) 03:58:57.04ID:K8dWc2vF
>>867
電線の太さは、圧着器の成功率のために選ぶんじゃなくて、
装置の配線に最適な太さを選ぶのが正しいですよね。
正規の圧着器は、太さに関係なく綺麗にしっかりと圧着できます。
お小遣いを貯めて、XH用だけ買えば済むことです。
なぜそれができないのか、不思議ですよねえ
08708492022/06/17(金) 07:20:03.38ID:trXps1AU
ありがとうございます、自分の用途にはオーバースペックな気がしますがとりあえず100MHzのオシロを買おうかと思います。
0871774ワット発電中さん2022/06/17(金) 07:43:08.62ID:cMB5qi3q
SIGLENT SDS1104X-Eを持っていて先日OWON HDS2102Sを買ったけど
基本的な使い勝手は圧倒的に前者の方が良いな。性能・機能差はもちろん
操作性も良いし、画面もでかくて情報量が多く見やすい
一台目としてHDS200シリーズはお薦めしづらい
サブとして小型であることやバッテリー駆動可能であることに価値を見いだすならあり

HDS200は一応2chオシロだけどオシロのプローブは1ch分しか付いていないし
2ch使用時にはサンプリングレート半減するしHDS2102Sでも2ch使用時の帯域は25MHzくらいでは
0872774ワット発電中さん2022/06/17(金) 08:13:17.19ID:GklNDW9t
>>869
>正規の圧着器は、太さに関係なく綺麗にしっかりと圧着できます。
ワイヤーの太さは指定されてるだろ。
圧着器使ったことないのがバレバレ。
0873774ワット発電中さん2022/06/17(金) 08:45:45.21ID:M4JEU9s7
便乗して話のネタを提供すると、正規の圧着工具は
施工不良防ぐために正規締め込み角度にならないと開かないラチェットが付いてるタイプもあるから
太すぎる電線でチャレンジすると開かなくなる危険がある。
仮に運よく銅線部は何とかなっても、
XHに、適用外の太めのAWG20とかを使っちゃうと
だと被覆の厚さが隣と干渉してハウジングに入らないかもしれないリスクとかもあったりするんよ・・・
そういう意味でも適用電線の確認は確実にね(^p^;
0874774ワット発電中さん2022/06/17(金) 11:51:23.67ID:trXps1AU
>>871
HDS2102を買おうかと

https://www.dropbox.com/s/uyxufdcd1q1wzga/HANMATEK%20HO52.zip?dl=0&file_subpath=%2FHANMATEK+HO52%2FHO52+%E3%83%87%E3%83%A5%E3%82%A2%E3%83%AB%E3%83%81%E3%83%A3%E3%83%B3%E3%83%8D%E3%83%AB%E3%82%B7%E3%83%AA%EF%BF%BD%60%E3%82%BA%E3%83%8F%E3%83%B3%E3%83%87%E3%82%A3%E5%A1%80%E3%82%AA%E3%82%B7%E3%83%AD%E3%82%B9%E3%82%B3%EF%BF%BD%60%E3%83%97%E3%83%A6%EF%BF%BD%60%E3%82%B6%EF%BF%BD%60%E3%83%9E%E3%83%8B%E3%83%A5%E3%82%A2%E3%83%AB_V1.0.0-%E6%99%A9%E4%BA%91%EF%BF%BDZ.pdf

にあるマニュアルを見てみると測定できる項目が

周期、周波数、幅、ピーク値、最大値、最小値、平均値

しかなく、Duty比もRMS(実効値)も測れないようなのですがこれってオシロに当然ある機能と思っていましたがないのですね。
自分の持っているアマゾンの4000円の格安オシロでも正確性はおいといてもDuty比とRMSは表示されます。
しかも2chオシロなのにプローブが1つしか付いてないとは自分にとってHDS2102は完全に地雷でした。

そこで

OWON ハイコストパフォーマンスデジタルオシロスコープ 1Gs/s 100MHz帯域 薄型軽量 SDS1102【3年保証】【国内正規品】
https://www.アマゾン.co.jp/OWON-%E3%83%87%E3%82%B8%E3%82%BF%E3%83%AB%E3%82%AA%E3%82%B7%E3%83%AD%E3%82%B9%E3%82%B3%E3%83%BC%E3%83%97SDS1000%E3%82%B7%E3%83%AA%E3%83%BC%E3%82%BA-100Ms-20%EF%BC%8D200MHz%E5%B8%AF%E5%9F%9F-%E3%83%8F%E3%82%A4%E3%82%B3%E3%82%B9%E3%83%88%E3%83%91%E3%83%95%E3%82%A9%E3%83%BC%E3%83%9E%E3%83%B3%E3%82%B9%E3%80%90%E5%9B%BD%E5%86%85%E6%AD%A3%E8%A6%8F%E5%93%81%E3%80%91%E3%80%903%E5%B9%B4%E4%BF%9D%E8%A8%BC%E3%80%91/dp/B0784FHXYR?th=1

このオシロにしようかと思うのですが何か落とし穴のようなものはないでしょうか?
0875774ワット発電中さん2022/06/17(金) 12:34:02.88ID:5ehxeTz7
オシロ初めて使うのかな?
入力の耐圧が意外と低いので
プローブの1:1は使わないほうが良いとか?
0876774ワット発電中さん2022/06/17(金) 12:37:55.95ID:VY/z2YU5
携帯型のオシロってスペック高いくせに拡大つまみがないから腹立つ
0877774ワット発電中さん2022/06/17(金) 13:00:46.25ID:5ehxeTz7
プローブが2つ以上ある場合
アースが共通なの忘れて
一緒にしてはいけないところを「つないでしまって
プローブ燃やしたり、機器を破壊したりも初心者アルアル
0878774ワット発電中さん2022/06/17(金) 13:01:36.78ID:CMwJAjQQ
>>877
???
0879774ワット発電中さん2022/06/17(金) 13:05:24.80ID:5ehxeTz7
>>878
ん?なに?わかんない?
0880774ワット発電中さん2022/06/17(金) 13:06:50.37ID:cU9O8PYQ
>>877
それでフローティング電源焼いたわ
こんがりと
0881774ワット発電中さん2022/06/17(金) 13:25:12.42ID:6sw82sqI
>>874
その「OWONメーカー直営店」を覗いたら
FNIRSIのスペック詐欺オシロ取り扱ってる
ヤバい気配しかしない\(^o^)/

「OWONメーカー直営店」<-ホントかよ!!

オシロのOWONとは関係ないOWONという名前の会社がOWONのオシロを売ってるだけかも
0882774ワット発電中さん2022/06/17(金) 13:33:12.88ID:cpuGubrs
迫りくる顔文字
加速するよだれ
虚飾と妄想が交錯する
買うのはどっちだ!!
0883774ワット発電中さん2022/06/17(金) 13:41:56.36ID:6sw82sqI
新品の安価なオシロでまともなの買うなら(&まともな売主)
RIGOL JAPANからRIGOL買うか
ADWINからSiglent買うかの2択だと思うの
※異論は認める\(^o^)/
0884774ワット発電中さん2022/06/17(金) 13:50:03.91ID:549Bq1mx
>>879
そんな馬鹿がいるとは思わなかった
0885774ワット発電中さん2022/06/17(金) 13:54:06.50ID:5ehxeTz7
ディスするだけで中身ないのは、容赦なくアボ~ンしてくのであしからず
0886774ワット発電中さん2022/06/17(金) 13:56:45.67ID:28oy/Ec8
片側のプローブのアース線外しとけばいいだけ
0887774ワット発電中さん2022/06/17(金) 14:54:08.59ID:V8ZHkJqS
>>884
ElectroBOOMさんにも同じ事家んのかお前
0888774ワット発電中さん2022/06/17(金) 15:05:39.43ID:PC+sxaxh
一般論として、とりあえず測定器として使えるラインがオシロの場合RIGOLやSIGLENTあたりから。つまり4~5万~
テクトロ等のブランドメーカー品でも最安が5~10万~だしエントリーグレードがこのあたり
それ未満のOWONやHantekとかはホビーグレードとでも呼べばいいのかな。エントリーグレードと比べると完成度が
低かったり、精度が怪しかったり、機能がショボかったりする
それを承知の上で割り切って買うならともかく、そうでないなら安物買いの何とやらである
0889774ワット発電中さん2022/06/17(金) 15:13:40.88ID:5ehxeTz7
三相インバーターをいじるんで3現象行けるのがほしくて
ヤフオクで4現象ストレージオシロの古~いのを買った数千円で

もとはいくらくらいのだろうと調べてみたら3桁万円だと、うひょ~
ブラウン管でいいなら、格安だね
0890774ワット発電中さん2022/06/17(金) 16:13:51.88ID:YXYY0PM/
私はOWONを机の上に出しっぱなしで(電源もプローブも挿しっぱなし、ビニールを被せて)
AVR電子工作などでテスターみたいに使っている。2台目の高機能オシロの出番は殆ど無い。
そりゃ高ければ高いほど性能が良いだろうけど、1台目はowonの2、3万円程度で良いのでは?
0891774ワット発電中さん2022/06/17(金) 16:18:50.31ID:YXYY0PM/
OWONとの差額分でロジックアナライザを買うとか?w
0892774ワット発電中さん2022/06/17(金) 16:41:21.63ID:5ehxeTz7
ロジアナはすでに持ってるって言ってたw
0893774ワット発電中さん2022/06/17(金) 18:06:10.66ID:dGfU8znL
>>890
例えば上で名前が出ているSDS1102だと2ch+Trigですらなくて本当に2chしかないし
プロトコルデコーダもないように見えるけどマイコン工作で足りる?
CS#でトリガ掛けてSPI見るとか出来ないよね
OWON SDS1000シリーズってスペック表もガバガバに見えるし
サンプリングレートはちゃんと書いていないしメモリ長も不明だし・・・
0894774ワット発電中さん2022/06/17(金) 18:21:42.74ID:7UWSpQS0
もうオシロスレでやれよ。
0895774ワット発電中さん2022/06/17(金) 21:43:33.97ID:D+0bVg+x
>>893
CS#とSCKの関係がタイミング条件を満たしていることを確認する。
CS#とSCKのANDを外部回路で作って、それでトリガをかけて、MOSIとMISOを別々に見る。

これから買うなら4chあるほうがいいのは間違いない。
0896774ワット発電中さん2022/06/17(金) 22:03:43.53ID:7yPDE2PA
>>881
OWON JAPAN はOWONの日本法人じゃないからね
SIGLENT JAPAN も同じく
本家のHPの日本語訳で
あたかも日本法人のように装っているが
08978712022/06/17(金) 22:07:17.28ID:5NUIC42B
>>874
オシロのプローブが1本なのは不便なのでAmazonでP6100の2本セットを買った
HDS2102Sはテクニカルサポートがマシそうな秋月電子で購入
Amazonの方がちょっと安いけどトラブったとき面倒になるのは避けたかったし

>SDS1102
他の人も心配しているけど、ch数やメモリは足りるの?
10Kサンプルしかないしデジタルはロジアナに任せて波形の確認だけ出来れば
かまわないくらいの割り切りは必要そう

>>881
名称もだしテクニカルサポートが期待出来そうにないんだよね
ググると上位に出てくることが多いウェーブクレストも自称国内正規販売代理店の可能性が高いし
結局HDS2102Sは秋月で買った。OWON製品買うなら秋月が無難かなと思う
0898774ワット発電中さん2022/06/18(土) 10:38:57.17ID:QQNUwVi3
俺も>>893みたいにグダグダ考えてた頃があった。
OWONを買って>>890のように雑にテスター代わりに使いだしてから
テクトロを使う機会が激減した。
たとえ低スペックだろうが有ると無いとじゃ大違い。
0899774ワット発電中さん2022/06/18(土) 12:51:43.37ID:w/W16bgu
AmazonでRIGOL DS1102Z-Eが約3.4万でセールになっているね
3万くらいでOWONやHantek買うならこっちの方が良さそう
トリガ入力も付いているしロングメモリだし
あとHantek DSO5000Pシリーズ(DSO5072Pとか)はトリガ入力付いているみたい
価格優先ならSDS1102よりこっちの方が良さそうだけど使っている人いる?
0900774ワット発電中さん2022/06/18(土) 19:39:10.03ID:fL6kF0Hd
LM386を使ったギターアンプを作成したいと思い、ネットでいろいろな作例を見て勉強しています。

https://www.etsy.com/listing/713721358/diy-qrome-guitar-amplifier-blank-circuit
このURLの製品のコンデンサC6 470pFは、どこにつながっていてどのような役割なのでしょうか?

基板の表と裏の画像を見るとC4とC5が電源のパスコン?の気がしますが、さらにC6にまで線がつながっているように見えます。
3重にもコンデンサをつける作例は見たことないので多分違うと思いますが、そうなると余計にわかりません。
どなたか、よろしくお願いします。
0901774ワット発電中さん2022/06/18(土) 20:21:39.36ID:VnDiFNSl
回路図作ってUPしてくれ
0902774ワット発電中さん2022/06/18(土) 20:42:31.48ID:o252Vu1I
>>900
3枚目の写真だとC6は両足ともショートしているように見えるが
0903774ワット発電中さん2022/06/18(土) 20:59:04.12ID:v+T0pxU5
コンデンサは並列にするほど音がよくなます
初心者質問スレ 153 YouTube動画>13本 ->画像>53枚
0904774ワット発電中さん2022/06/18(土) 21:06:40.45ID:9evSk49w
なましたらダメだろ。
0905774ワット発電中さん2022/06/18(土) 21:51:14.30ID:WuE9e2By
また現代アートか
0906774ワット発電中さん2022/06/19(日) 01:43:44.34ID:FmFPq1t6
>>903
そう言えば、何でオーディオ機器の電源回路に付いてる大容量ケミコンは、1つだけなんでしょうね。
少し小さな容量で、並列につないでも良さそうなのに。
0907774ワット発電中さん2022/06/19(日) 03:07:49.11ID:VzQhxdly
>>906
kwsk
0908774ワット発電中さん2022/06/19(日) 05:28:31.49ID:vXu8lS7R
>>906
複数付いてる物なんていくらでもあると思うが
0909774ワット発電中さん2022/06/19(日) 07:20:29.84ID:+A9FwMpt
初心者質問スレ 153 YouTube動画>13本 ->画像>53枚
0910774ワット発電中さん2022/06/19(日) 07:24:01.45ID:+A9FwMpt
アンプを駆動するには直流電源が必要で、電荷は電源コンデンサーに蓄えられていて、そこから供給されます。
普通、この電源コンデンサー容量が十分でないために、アンプが正しく働いていない、即ち歪むと考えています。
そのため、電源のコンデンサーと並列に、数十倍、数百倍の大容量コンデンサーを接続してやると、高音質の再
生ができます。
0911774ワット発電中さん2022/06/19(日) 07:53:38.97ID:pDHTAlBX
>>909
四角い点々がこれみたい

@YouTube

0912774ワット発電中さん2022/06/19(日) 07:54:24.10ID:pDHTAlBX
>>909
ナイフスイッテ懐かしぶりにみた
0913774ワット発電中さん2022/06/19(日) 07:58:35.28ID:gw8c1fmo
>>911
音はリアルじゃないシンセで作ってる
0914774ワット発電中さん2022/06/19(日) 08:08:49.84ID:KIFvJQXM
せめてこれくらいで

https://www.jp.onkyo.com/support/pcaudio/discontinued_products/se200pci/index.htm
初心者質問スレ 153 YouTube動画>13本 ->画像>53枚

https://ascii.jp/elem/000/000/343/343411/
0915774ワット発電中さん2022/06/19(日) 12:14:49.81ID:q5NwM8SN
コンデンサをいたずらに増やしても仕方がないよ。
初心者の人は、やったらたくさんのコンデンサを付けるのはネタとして受けておいて。
0916774ワット発電中さん2022/06/19(日) 14:52:01.29ID:vXu8lS7R
>大容量コンデンサーを接続してやると、高音質の再生ができます。

これはちょっと語弊があるな
0917774ワット発電中さん2022/06/19(日) 14:53:43.17ID:Ypc4dpUx
ブラシーボ効果にたいへん貢献します
0918774ワット発電中さん2022/06/19(日) 16:57:36.97ID:XMe/PH5y
スパシーボ
0919774ワット発電中さん2022/06/19(日) 17:14:58.69ID:zGgtDe4v
あと、あと、1uF追加すれば全く別次元の音になったのにと。
0920774ワット発電中さん2022/06/19(日) 17:33:04.17ID:KUAQ+wwr
コンデンサの妖怪じゃん
0921774ワット発電中さん2022/06/19(日) 18:42:45.84ID:Vm+luhkq
>>915
オーディオやってる人、コンデンサ変えたがるよね。違いもわからないのに。
0922774ワット発電中さん2022/06/19(日) 18:53:46.16ID:tCzthE68
オールフィルムコンじゃないのか
0923774ワット発電中さん2022/06/19(日) 18:56:32.23ID:JzY8caPf
そういえばオーディオの人って
コンデンサはデカい容量で、フィルムコンデンサしか認めない
とか結構似たようなこと言うけど、
元々は何処が発祥の情報なんだろう
0924774ワット発電中さん2022/06/19(日) 19:09:26.48ID:tCzthE68
発祥は知らんが特性が良いから
スピーカーネットワークなんてフィルムコンしか見たことないし
0925774ワット発電中さん2022/06/19(日) 19:18:44.58ID:Ypc4dpUx
ほとんど宗教みたいなもんだから、教祖的な奴が雑誌とかで吹いて回って広めたんじゃないの?

俺もスピーカーネットワーク有害論に染まって長年フルレンジ最強とか思ってたけど、ぜんぜんそんなことないのな
0926774ワット発電中さん2022/06/19(日) 19:56:04.15ID:rRknAkKH
>>921
わならないのはお前でしょ
0927774ワット発電中さん2022/06/19(日) 19:57:13.06ID:fxUbiaAW
今やほぼデジタル処理から逃げられないのにアナログのみ注視している次点でお察しじゃね
0928774ワット発電中さん2022/06/19(日) 20:01:11.13ID:JzY8caPf
フィルムコンが聴きやすくなるのはわかるのだけど
電解とフィルムで違う容量を使うとか言い出す人までいるし・・・
どういう理屈なんだろう

セラコンも異常に嫌われるよね
0929774ワット発電中さん2022/06/19(日) 20:28:09.52ID:/itMkF46
帯域によって容量変わるからとか?
知らんけど
0930774ワット発電中さん2022/06/19(日) 21:18:26.19ID:2JV3rIxb
>>928
セラコンはB特とか避ける為だね

フィルムは、セラコンの変わりに使われる場合が多い(0.1uFとかは特に)からじゃない?
0931774ワット発電中さん2022/06/19(日) 21:21:00.41ID:q5NwM8SN
>>928
セラコンに関しては、オーディオマニアでなくても、要注意です。

高誘電率タイプを信号ラインに使う場合ですが
・電圧やかける周波数によって静電容量が変わります。静電容量が問題になる領域で使う場合、信号に影響を与えます。
・振動でノイズを発生します。(といっても基板を叩くようなことをしない限り、大きいものではないですが)
0932774ワット発電中さん2022/06/19(日) 21:36:34.25ID:JzY8caPf
それってオーディオの人が「セラコン」でひとくくりにしてるからおかしな事になってるだけじゃないの?
それぞれ目的別に特性をわけてるんだから、合わせたものを使えば問題にならないような気がするけど
0933774ワット発電中さん2022/06/19(日) 22:02:35.94ID:q5NwM8SN
高誘電率タイプを結合コンデンサに使っているケースもあるしね。
0934774ワット発電中さん2022/06/20(月) 00:03:15.23ID:lgtjTn8V
>フィルムコンが聴きやすくなるのはわかるのだけど
「聴きやすい」の意味するところと、どういう条件や理屈で有意な違いが生まれるのかが「わかる」なら教えて。
0935774ワット発電中さん2022/06/20(月) 00:28:39.08ID:XrrsCiGz
>>933
>高誘電率タイプを結合コンデンサに使っているケースもあるしね。
なんでダメなの?
0936774ワット発電中さん2022/06/20(月) 01:25:39.76ID:7tGRILX8
もうオーディオスレでやれ
0937774ワット発電中さん2022/06/20(月) 05:59:20.01ID:zgRPzqhN
>>927
デジタル信号も結局はアナログだっていうことが
大人になると解ってくるよ。
0938774ワット発電中さん2022/06/20(月) 06:29:09.95ID:yP/14ydv
>>935
誘電損が大きいのと、温度特性が悪いからじゃない?
0939774ワット発電中さん2022/06/20(月) 07:27:37.91ID:lgtjTn8V
>>935
ダメということではなくて、ダメなこともあるし大丈夫なこともある。合目的ならいい。
条件をちゃんと面倒くさく考えずにダメかそうでないかを分けることがダメ。
高誘電率タイプの大き目なマイナス点は>>931に書いた通り。
でも、フィルムコンデンサ、電解コンデンサに比べてメリットもあるしね。
0940774ワット発電中さん2022/06/20(月) 07:30:34.77ID:H+u9kKNf
たぶん差し替えてブラインドテストしてもわからないし
よくないとされるCの方がいいとか言い出す可能性も
0941774ワット発電中さん2022/06/20(月) 07:32:49.10ID:H+u9kKNf
>>931
バリキャップのように変調かけるテクニックもありますね
定義されてないような特性をわざわざ採用したりしないだろうけど
ホビー的にはアリかと
0942774ワット発電中さん2022/06/20(月) 07:34:15.33ID:H+u9kKNf
ちなみに初めて作ったFMワイヤレスマイクは
マイクで拾う音よりもコイルが拾う音の方が
リバーブかかって官能的でした
0943774ワット発電中さん2022/06/20(月) 07:36:52.63ID:lgtjTn8V
>>973
>デジタル信号も結局はアナログだっていうことが
極論に導けばそうなるけれど、その場その場でどの範囲が有意にデジタルなのかアナログなのかは
大筋で把握できることが多いと思う。
およそデジタル信号で考えて良い領域なのかアナログ信号として考えて良い領域なのか、事案ごとに
変わってくるだろうから、具体例を出した上で客観的事実を積み上げて考えるべき。
0944774ワット発電中さん2022/06/20(月) 07:47:48.15ID:lgtjTn8V
アンカーまちがった。
>>937 だった
>デジタル信号も結局はアナログだっていうことが

>>941
自分ではやったことがないけれど、たとえばそれで周波数変調でもかけるとして
pFオーダーの高誘電率系が欲しくなるような気がした。そんなものあるのかな?
と思ったら、100pF以下の高誘電率系ってあるんだね。
0945774ワット発電中さん2022/06/20(月) 07:49:58.33ID:yP/14ydv
>>944
一部のマニアしか欲しがらないから売ってない
どうしても欲しいなら高誘電率タイプを直列にすればいい
0946774ワット発電中さん2022/06/20(月) 07:57:44.01ID:lgtjTn8V
>>945
ふつうにDigikeyにいっぱい並んでるよ。(つまりマルツで買える)
その事実を俺が知らなかっただけ。
0947774ワット発電中さん2022/06/20(月) 08:18:40.20ID:DxmkgPIr
>>909
タイル爺さん、まだやってんのかね
0948774ワット発電中さん2022/06/20(月) 08:24:20.75ID:yP/14ydv
>>946
なるほど
勉強になったな
0949774ワット発電中さん2022/06/20(月) 10:15:48.00ID:ejw0uG3Y
>>938
誘電損が大きかったり、温特が悪いと、
何でオーディオの段間結合に使ってはダメなの?
100歩譲って温特はそうとしても、誘電損がまずい理由は何でしょうか?
0950774ワット発電中さん2022/06/20(月) 10:56:19.41ID:H+u9kKNf
イメージが悪い
つまりイメージが音を奏でているのです

例えば今鳴ってるアンプが無名ともいえるようなOEM元と著名なブランドでは
ブランドロゴ以外は同じものでも音は変わりませんが
奏でるイメージが変わるのです
電源となる発電所しかり

電解コンデンサなどは、巻いてる収縮樹脂の模様が
いちばん作用しているのかもしれません
0951774ワット発電中さん2022/06/20(月) 10:59:23.11ID:lgtjTn8V
ステレオタイプな「バカにされるオーディオマニア」を演じて貶める必要もないと思うんだよな。
0952774ワット発電中さん2022/06/20(月) 11:04:40.74ID:6++uG4cb
大昔ピュアオーディオ板の住人だった俺としてはこの流れ興味深い
0953774ワット発電中さん2022/06/20(月) 11:29:19.22ID:yP/14ydv
>>949
温度特性が悪かったらその日の温度によって音が変わる気がするでしょ
誘電損が大きかったらなんか気になるでしょ
そういうことよ
0954774ワット発電中さん2022/06/20(月) 11:32:44.29ID:g0KG1udB
>>953
体調や環境状態による耳の特性や感覚の変化のほうが著しいとは思いますがね
何で判断するんだろう
標準器みたいなのがあるのだろうか

絶対音感みたいに絶対特性みたいなの体得してるのだろうか
0955774ワット発電中さん2022/06/20(月) 12:26:56.09ID:5HBFxtd8
オーディオマニアは測定器では検出できないほどの小さな違いを判別できる人たち()なので彼らと標準や測定での議論はできない
彼らにとっては主観に影響を与える要因こそ大切ともいえる
実際には何も変化していなくても心理的に何らかの影響があればOK
0956774ワット発電中さん2022/06/20(月) 12:30:47.02ID:rG9HVYWd
見た目で音が決まるのは事実だけどな

https://www.audiostyle.net/archives/51365361.html
0957774ワット発電中さん2022/06/20(月) 12:34:30.78ID:H+u9kKNf
>>955
それを心の変化という
0958774ワット発電中さん2022/06/20(月) 12:59:21.86ID:SP7G7R8v
音質を追求するのは程々にして、楽器を覚えて演奏する側になれよ。楽しいぜ?
音楽とは本来そういうものだろ?
0959774ワット発電中さん2022/06/20(月) 13:03:28.44ID:bx90HAud
>>954
そのくらいじゃないとオーオタなんて名乗れない
0960774ワット発電中さん2022/06/20(月) 13:04:01.77ID:CuBwnoxy
楽器弾くからエレキギターのピックアップに無酸素銅を使いたい
0961774ワット発電中さん2022/06/20(月) 13:13:42.83ID:599ckz7b
楽器演奏する側(とくにエレキとか)は音を作るがわなので、オーディオ屋がダメって言うことをやりほうだい
だがなぜか、オーディオ屋と同じことを言う人が多いのが謎だ
0962774ワット発電中さん2022/06/20(月) 13:33:56.97ID:8X1M0bBA
>>958
聴きたいのは音楽じゃ無いんだよ、音なんだ
0963774ワット発電中さん2022/06/20(月) 13:42:31.28ID:IpEoA58k
>>961
いかに心地よく歪ませるか考えて回路組むくらいだしな
0964774ワット発電中さん2022/06/20(月) 13:50:08.34ID:zgRPzqhN
まあ、やりたきゃなんでもやりゃ良いのさ。

昔、アンプとスピーカーの組み合わせを変えながら
色々な音楽ソースを聞いてみた時に、たしかに
相性ってあるものだなあと感じたこともあったから、
オーヲタを全否定はしないけどね。
0965774ワット発電中さん2022/06/20(月) 13:52:30.70ID:ejw0uG3Y
オーディオをなめてはいけない、奥が深いんですね。
電源が50Hzと60Hzでも違う、と言われそうですね。
0966774ワット発電中さん2022/06/20(月) 13:57:34.75ID:599ckz7b
演奏側
ドラムの音が気に入らね~
俺が求めてるのは、そんな音じゃない
スタジオにあったいろんな物を叩きだす
「これだ」「この音だ」
その時たたいていたのはイショウケース
無事レコーディング完了しましたとさ


オーディオ屋
イショウケースをすぴーかーぼっくすにしたら、とっても気に入った音が出たんだが
こんなもの認めるわけにいかね~、なかったことにしようw
0967774ワット発電中さん2022/06/20(月) 15:53:49.54ID:X50ur5YT
デジタルアンプの上に真空管鎮座させて
ヒーター光らせとけばええ感じの音な気がしない?
0968774ワット発電中さん2022/06/20(月) 15:59:17.15ID:599ckz7b
>>967
暑いからヤダ
0969774ワット発電中さん2022/06/20(月) 16:30:51.14ID:9xZGWJuB
テスト
0970774ワット発電中さん2022/06/20(月) 16:42:09.13ID:9xZGWJuB
M5STACKを使ったBlackmagicのタリーランプが作れるとの記事を見たので
それを作れるように勉強したいと思っているのですが、素人がそのようなものを作るとき
電子工作ではどういう本を読むと作れるようになるのでしょうか?

ご教示いただければ幸いです。
0971774ワット発電中さん2022/06/20(月) 16:54:18.37ID:599ckz7b
質問が漠然としすぎtwるから、アドバイスしようにも・・・・

製作者がどの程度のレベルに居るかによっても勧める本は違うし
そのタリーとやらの必要要件もわからんので、どんな技術を要するかがわからん
0972774ワット発電中さん2022/06/20(月) 16:54:33.96ID:iVWNpQ2O
>>970
コードまで公開されてるなら部品さえありゃ出来る。記事とかいわれても詳しい内容も参照先も無しじゃ知らんがなって感じだが。
0973774ワット発電中さん2022/06/20(月) 17:04:29.26ID:T3fXga3M
>>970
とりあえずM5Stackを買ってその記事を真似るか、「M5 stack Lチカ」でググって基本から
M5Stackである必要もないけど
0974774ワット発電中さん2022/06/20(月) 17:08:21.78ID:9xZGWJuB
ご回答ありがとうございます。

タリーランプはスイッチャーと言われる映像を切り替えたのがわかるランプで
数字が表示されフリー(緑)オンエア(赤)その他(黒) とかで表示できればなと思っています。と
数字のバックの色が変わればいいなと思っています。

https://github.com/oneguyoneblog/tally-light-esp32-for-blackmagic-atem-switcher/blob/master/tally-light-esp32-for-blackmagic-atem-switcher.ino

こんなサイトを見つけました。
私は全然の素人です。
よろしくお願いいたします。
0975774ワット発電中さん2022/06/20(月) 17:21:28.02ID:9xZGWJuB
>>974
#define GRAY 0x0020 // 8 8 8
#define GREEN 0x0200 // 0 64 0
#define RED 0xF800 // 255 0 0

ここの意味は解読できました。
多分ですが 
#define GRAY 0x0020
で色の定義をしてるんですね。
// 8 8 8はPCの色のコードですよね?

プログラムするとすると

#define GRAY 0x0020
と書き込むってことですかね?
0976774ワット発電中さん2022/06/20(月) 17:22:35.31ID:599ckz7b
詳細はこちらのリンクが切れてて、なんもわからんがw
製作記事になってて、ソースコードまであるなら、背景にある技術とかはいらないから
はんだ付けができてm5が使えりゃokって、もしかして、M5に備わってるインターフェイスでぜんぶできちゃうのかな?
それだとはんだずけもいらないが
0977774ワット発電中さん2022/06/20(月) 17:24:48.61ID:T3fXga3M
>>975
ソースコードなんて後で読めばいい
とりあえず買って、arduino ide インストールして、そいつをM5に書き込んで動けばOK
動かなかったらソースコード読んでIPアドレスとか適切に設定したりするまで
0978774ワット発電中さん2022/06/20(月) 17:27:09.53ID:9xZGWJuB
>>977
これ間にURLの記述溶かされてますけど
これコピペでいいんですか?
0979774ワット発電中さん2022/06/20(月) 17:32:03.44ID:IDxJXkoi
これってwifiルーターにスイッチャーがWiFiで繋がっててそれに対してesp32が
WiFiでそのネットワークに接続して
スイッチャーの状態を読みに行ってるかんじなのかな?

ライブラリー作ってくれてるから頑張ったら行けそう
>>978
IPアドレスとかwifiのssidとパスワードを環境に合わせたらいいんじゃないのかな
0980774ワット発電中さん2022/06/20(月) 17:39:51.03ID:uh4EDTU6
>>978
コピペで良いよ
ちゃんとライブラリ入れたりしないとコンパイルエラーになると思うけど、それはその時解決すれば良いよ
あとハードはM5stickCだね。
0981774ワット発電中さん2022/06/20(月) 17:43:49.56ID:N3ahW+AS
素人かどうかは関係無い
馬鹿には無理
0982774ワット発電中さん2022/06/20(月) 17:50:46.36ID:9xZGWJuB
>>979
>>980
ありがとうございます。

そうです。
スイッチャーがLANでルーターに接続されてて
それをM5stickが読み取るという感じです。
これは現行モデルのM5Stack Basic V2.6などでは使えないのでしょうか?

おなじesp32ベースなのでこのまま行けると思うのですが、基本的にはそれは難しいものなのでしょうか?

ちょっと買って試したいと思うのでハードの面よろしくおねがいいたします。
これが出来て、ソースを解読していけば組む題材としては頑張れる気がします。
0983774ワット発電中さん2022/06/20(月) 17:59:48.56ID:IDxJXkoi
>>982
使ってるライブラリーとか関数は
M5stick固有の製品使ってないっぽいから
多分ベーシックでもcore2でもこれは実行できるはず
このプログラムでは状態表示にLED使ってるけどディスプレイで代用できるし

極端な話、WiFi扱えるモデルなら何でもいけそう
0984774ワット発電中さん2022/06/21(火) 01:00:15.74ID:jKE77eal
>>983
ありがとうございます。
早速1台買ってみて試そうと思います!
0985774ワット発電中さん2022/06/21(火) 17:16:56.86ID:yeKVsyKo
あれ?短いIDだな?と一瞬思ってしまったw >>983
0986774ワット発電中さん2022/06/21(火) 18:24:24.68ID:jhu2IAmX
100V/100V 10VAの絶縁トランスを購入したのですが、
試しに直流抵抗を計ると二次の方が5割ほど大きいのです
よく観察すると一次二次で巻線の太さが違うのですが、
これはどういう意図があるのでしょう?
0987774ワット発電中さん2022/06/21(火) 18:46:24.80ID:s65k/NCr
>>986
特殊なもんじゃなけりゃ、一層目巻いた上に二層目って重ねるから

同じ太さで同じ回数巻いても線長は異なるので直流抵抗値は異なって当然
0988774ワット発電中さん2022/06/21(火) 18:58:34.12ID:jhu2IAmX
なるほど。納得できました。
0989774ワット発電中さん2022/06/21(火) 19:16:41.91ID:mfDV+hxv
再放送はいつですか?

https://www.nhk.jp/p/ts/DN23LL75QJ/episode/te/X7VGRZRR9V/
「光通信に賭けた男 〜独創の科学者・西澤潤一〜」
初回放送日: 2022年6月21日
今日の高度情報化社会を支える光通信。
それは、入口で電気信号を光に変える半導体レーザー、その光を通す光ファイバー、
出口で光を再び電気信号に変えるpinダイオードから成り立っている。
この三要素を世界に先駆けて開発、提唱したのが、東北大学の西澤潤一教授であった。
しかし、その独創性ゆえに国内では異端視され、彼の発明の多くは海外で認められた。
西澤教授に密着し、画期的な発明を生む思想と行動を描く。
0990774ワット発電中さん2022/06/21(火) 19:20:54.95ID:QuC6KoJh
>しかし、その独創性ゆえに国内では異端視され、彼の発明の多くは海外で認められた。
そのパターンばっかりだよなぁ。
マイコンしかり、3Dプリンタしかり、AIしかり。

きっと、今も未来のトンデモナイお宝が眠っているに違いない。

俺は余命が大して残ってないが、みんな頑張れよ。
0991774ワット発電中さん2022/06/22(水) 07:12:22.50ID:PAW4PeZN
西澤先生ほんますごいと思うわ
0992774ワット発電中さん2022/06/22(水) 08:02:01.01ID:b+Bw6JJ+
電子立国にも登場してた
0993774ワット発電中さん2022/06/22(水) 08:04:10.30ID:b+Bw6JJ+
NHKも契約の強要するばかりでなくいい番組を作ればいいだけなのだ
つまらない低俗番組だからゆえ、お金払わずに見ようという人の割合が多くなるのだ
北風と太陽なのだ
0994774ワット発電中さん2022/06/22(水) 12:07:12.71ID:Sz7q5BQt
>>989
pinフォトダイオードの発明って、ソニーのエンジニアじゃ無いの?
確か、今はどっかの大学の先生になってる
0995774ワット発電中さん2022/06/22(水) 12:52:46.36ID:Naqxa6nH
特公昭30-8969号
高抵抗薄層領域を有する半導体光電変換器
発明者 西沢潤一 渡辺寧
0996774ワット発電中さん2022/06/22(水) 12:54:50.15ID:b+Bw6JJ+
>>994
東北大学学長
0997774ワット発電中さん2022/06/22(水) 13:01:38.22ID:VhEBh250
0998774ワット発電中さん2022/06/22(水) 13:32:41.87ID:XZzlXTnS
誰か次スレおねしゃす
0999774ワット発電中さん2022/06/22(水) 13:38:07.05ID:KKtUjyAn
>>990
赤の西澤
青の中村
1000774ワット発電中さん2022/06/22(水) 13:40:32.73ID:KKtUjyAn
初心者質問スレ 154
http://2chb.net/r/denki/1655872805/
10011001Over 1000Thread
このスレッドは1000を超えました。
新しいスレッドを立ててください。
life time: 56日 1時間 3分 21秒
10021002Over 1000Thread
5ちゃんねるの運営はプレミアム会員の皆さまに支えられています。
運営にご協力お願いいたします。


───────────────────
《プレミアム会員の主な特典》
★ 5ちゃんねる専用ブラウザからの広告除去
★ 5ちゃんねるの過去ログを取得
★ 書き込み規制の緩和
───────────────────

会員登録には個人情報は一切必要ありません。
月300円から匿名でご購入いただけます。

▼ プレミアム会員登録はこちら ▼
https://premium.5ch.net/

▼ 浪人ログインはこちら ▼
https://login.5ch.net/login.php

lud20240528043425ca
このスレへの固定リンク: http://5chb.net/r/denki/1651030631/
ヒント:5chスレのurlに http://xxxx.5chb.net/xxxx のようにbを入れるだけでここでスレ保存、閲覧できます。

TOPへ TOPへ  

このエントリをはてなブックマークに追加現在登録者数177 ブックマークへ


全掲示板一覧 この掲示板へ 人気スレ | Youtube 動画 >50 >100 >200 >300 >500 >1000枚 新着画像

 ↓「初心者質問スレ 153 YouTube動画>13本 ->画像>53枚 」を見た人も見ています:
初心者質問スレ 154
ポケGOPvP初心者質問スレ 1戦目
MTB初心者質問スレ part103
ロード初心者質問スレ part443
初心者質問スレ その123 [無断転載禁止]
Tasker 初心者質問スレ 16 [無断転載禁止]
【海外通販】AliExpress初心者質問スレ 15
【海外通販】AliExpress初心者質問スレ 19
【海外通販】AliExpress初心者質問スレ 17
【海外通販】AliExpress初心者質問スレ 12
Cakewalk by BandLab 初心者質問スレ Part 18
【MobA】vainglory初心者質問スレ ★Tier13 [無断転載禁止]
【FEH】ファイアーエムブレム ヒーローズ 初心者質問スレ Part.13
禁煙初心者質問スレ
初心者質問スレ その122
初心者質問スレ その135
初心者質問スレ その136
初心者質問スレ その125
初心者質問スレ その129
初心者質問スレ その124
MTB初心者質問スレ part114
ボウリング初心者質問スレ
MTB初心者質問スレ part118
MTB初心者質問スレ part126
MTB初心者質問スレ part115
MTB初心者質問スレ part106
ロード初心者質問スレ part395
信長の野望・大志 初心者質問スレ
ロード初心者質問スレ part464
ロード初心者質問スレ part444
ロード初心者質問スレ part 469
ロード初心者質問スレ part445
ロード初心者質問スレ part426
ロード初心者質問スレ part457
ロード初心者質問スレ part458
婚活パーティー初心者質問スレ
ロード初心者質問スレ part424
Google Chrome 初心者質問スレ 4
Google Chrome 初心者質問スレ 2
クロスバイク初心者質問スレ Part49
クロスバイク初心者質問スレ Part51
クロスバイク初心者質問スレ part9
クロスバイク初心者質問スレ part10
【海外通販】Aliexpress初心者質問スレ 2
【海外通販】AliExpress初心者質問スレ 8
【海外通販】AliExpress初心者質問スレ 21
【海外通販】AliExpress初心者質問スレ 20
【MobA】vainglory初心者質問スレ ★Tier6
【海外通販】AliExpress初心者質問スレ 26
ロード初心者質問スレ part397 [無断転載禁止]
【MobA】vainglory初心者質問スレ ★Tier7
ロード初心者質問スレ part408 [無断転載禁止]
ロード初心者質問スレ part356 [無断転載禁止]
ロード初心者質問スレ part394 [無断転載禁止]
初心者質問スレ その118 &#169;2ch.net
ロード初心者質問スレ part401 [無断転載禁止]
ロード初心者質問スレ part407 [無断転載禁止]
【海外通販】AliExpress初心者質問スレ Part2
【MacOSX】 Apache初心者質問スレ 【専門】
【Switch】ARMS 初心者質問スレ [無断転載禁止]
初心者質問スレ その122 ※aitendo利用者出入禁止
出前館 初心者質問スレ ブートキャンプ ★2日目
クロスバイク初心者質問スレ part6 [無断転載禁止]
.pid .lock ソケットファイル /run /tmp関連初心者質問スレ
【海外通販】AliExpress初心者質問スレ 6 【ワッチョイ】

人気検索: ショタ 中学 つぼみ 女子小学生のパンツ 繝√い繧ャ繝シ繝ォ逋ス繧「繝ウ繧ケ繧ウ 女子小学生マンコ画像 小学生盗撮 画像 ショタ Child nude little girls 女子小学生 少女スレ
10:28:50 up 44 days, 6:28, 0 users, load average: 4.63, 4.68, 4.34

in 0.070167064666748 sec @0.070167064666748@1c3 on 060823